829925
The
SAT®
Practice
Test #
2
Make time to take the practice test.
It
is
one
of
the
best
ways
to
get
ready
for
the
SAT.
After you have taken the practice test, score
it
right away
at
sat.org/scoring.
This version
of
the
SAT
Practice
Test
is
for students
who
will
be
taking
the digital
SAT
in
nondigital format.
11111111111111111111111
I I
Ill
t1
(:}
CollegeBoard
6TSL03
2
829925
6TSL03
Test begins on the next page.
Module
1
Reading and Writing
33
QUESTIONS
DIRECTIONS
The questions in this section address a
number
of
important
reading and
writing
skills.
Each
question includes one
or
more passages, which may include a table or graph.
Read
each passage
and question carefully, and then choose
the
best answer
to
the
question based on
the
passage(s).
All questions in this section are
multiple
-choice
with
four
answer choices.
Each
question
has
a
single best answer.
---------~ ---------~
1
As Mexico’s first president from an Indigenous
community, Benito Juarez
became one of the most
______ figures in his country’s history: among the
many significant accomplishments
of his long tenure
in office (1858–1872), Juarez consolidated the
authority of the national government and advanced
the rights of Indigenous peoples.
Which choice completes the
text with the most
logical and precise word or phrase?
A) unpredictable
B) important
C) secretive
D) ordinary
......................................................................................................................................................................................................
2
Due to their often strange images, highly
experimental
syntax, and opaque subject matter,
many of John Ashbery’s poems can be quite difficult
to
______ and thus are the object of heated debate
among scholars.
Which choice completes the text with the most
logical and precise word or phrase?
A) delegate
B) compose
C) interpret
D) renounce
Unauthorized copying or reuse of any part of this page is illegal.
2
CONTINUE
Module
1
...............................................................................................................................................................................................................................................................................................................
3
The Cambrian explosion gets its name from the
sudden appearance and rapid diversification
of
animal remains in the fossil record about 541 million
years ago, during the Cambrian period. Some
scientists argue that this ______ change in the fossil
record might be because
of a shift in many organisms
to body types that were more likely to be preserved.
Which choice completes the text
with the most
logical and precise word or phrase?
A) catastrophic
B) elusive
C) abrupt
D) imminent
4
During a 2014 archaeological
dig in Spain,
Vicente
Lull and his team uncovered the skeleton of
a woman from El Algar, an Early Bronze Age society,
buried with valuable objects signaling a high position
of power. This finding may persuade researchers who
have argued that Bronze Age societies were ruled by
men to ______ that women may have also held
leadership roles.
Which choice completes
the text
with the most
logical and precise word or phrase?
A) waive
B) concede
C) refute
D) require
5
Within baleen whale
species, some individuals
develop
an accessory spleen—a seemingly
functionless formation of splenetic tissue outside
the normal spleen. Given the formation’s greater
prevalence among whales known to make deeper
dives, some researchers hypothesize that its role isn’t
______; rather, the accessory spleen may actively
support diving mechanisms.
Which choice
completes the text
with the most
logical and precise word or phrase?
A) replicable
B) predetermined
C) operative
D) latent
6
According to a US tax policy expert, state taxes are
______
other factors when considering
an interstate
move. Even significant differences in state taxation
have almost no effect on most people’s decisions,
while differences in employment opportunities,
housing availability, and climate are strong
influences.
Which choice completes the text
with the most
logical and precise word or phrase?
A) consistent with
B) representative of
C) overshadowed by
D) irrelevant to
Unauthorized copying or reuse of any part of this page is illegal.
3
CONTINUE
Module
1
7
The author’s claim about the relationship between
Neanderthals and Homo sapiens is ______, as it
fails to account
for several recent archaeological
discoveries. To be convincing, his argument would
need to address recent finds of additional hominid
fossils, such as the latest Denisovan specimens and
Homo longi.
Which choice completes the text with the most
logical and precise word or phrase?
A) disorienting
B) tenuous
C) nuanced
D) unoriginal
8
The following text is from
Georgia Douglas
Johnson’s
1922 poem “Benediction.”
Go forth, my son,
Winged by my heart’s desire!
Great reaches, yet unknown,
Await
For your possession.
I may not, if I would,
Retrace the way with you,
My pilgrimage is through,
But life is calling you!
Which choice best states the main purpose of the
text?
A) To express hope that a child will have the same
accomplishments as his parent did
B) To suggest that raising a child involves many
struggles
C) To warn a child that he will face many challenges
throughout his life
D) To encourage a child to embrace the experiences
life will offer
...............................................................................................................................................................................................................................................................................................................
9
The following text is adapted from Indian
Boyhood,
a 1902 memoir by Ohiyesa (Charles A. Eastman), a
Santee Dakota writer. In the text, Ohiyesa recalls how
the women in his tribe harvested maple syrup during
his childhood.
Now the women began to test the trees—moving
leisurely among them, axe in hand, and striking
a single quick blow, to see if the sap would
appear. The trees, like people, have their
individual characters; some were ready to yield
up their life-blood, while others were more
reluctant. Now one of the birchen basins was set
under each tree, and a hardwood chip driven
deep into the cut which the axe had made. From
the corners of this chip—at first drop by drop,
then more freely—the sap trickled into the little
dishes.
Which choice best describes the function of the
underlined sentence in the text as a whole?
A) It portrays the range of personality traits
displayed by the women as they work.
B) It foregrounds the beneficial relationship
between humans and maple trees.
C) It demonstrates how human behavior can be
influenced by the natural environment.
D) It elaborates on an aspect of the maple trees that
the women evaluate.
Unauthorized copying or reuse of any part of this page is illegal.
4
CONTINUE
Module
1
10
Text 1
Ecologists have long wondered how thousands of
microscopic phytoplankton species can live together
near ocean surfaces competing for the same
resources. According to conventional wisdom, one
species should emerge after outcompeting the rest.
So why do so many species remain? Ecologists’ many
efforts to explain this phenomenon still haven’t
uncovered a satisfactory explanation.
Text 2
Ecologist Michael Behrenfeld and colleagues have
connected phytoplankton’s diversity to their
microscopic size. Because these organisms are so
tiny, they are spaced relatively far apart from each
other in ocean water and, moreover, experience that
water as a relatively dense substance. This in turn
makes it hard for them to move around and interact
with one another. Therefore, says Behrenfeld’s team,
direct competition among phytoplankton probably
happens much less than previously thought.
Based on the texts, how would Behrenfeld and
colleagues (Text 2) most likely respond to the
“conventional wisdom” discussed in Text 1?
A) By arguing that it is based on a misconception
about phytoplankton species competing with
one another
B) By asserting that it fails to recognize that routine
replenishment of ocean nutrients prevents
competition between phytoplankton species
C) By suggesting that their own findings help clarify
how phytoplankton species are able to compete
with larger organisms
D) By recommending that more ecologists focus
their research on how competition among
phytoplankton species is increased with water
density
...............................................................................................................................................................................................................................................................................................................
11
In 2014, Amelia Quon and her team at NASA set out
to
build a helicopter capable of flying on Mars.
Because Mars’s atmosphere is only one percent as
dense as Earth’s, the air of Mars would not provide
enough resistance to the rotating blades of a standard
helicopter for the aircraft to stay aloft. For five years,
Quon’s team tested designs in a lab that mimicked
Mars’s atmospheric conditions. The craft the team
ultimately designed can fly on Mars because its
blades are longer and rotate faster than those of a
helicopter of the same size built for Earth.
According to the text, why would a helicopter built
for Earth be unable to fly on Mars?
A) Because Mars and Earth have different
atmospheric conditions
B) Because the blades of helicopters built for Earth
are too large to work on Mars
C) Because the gravity of Mars is much weaker than
the gravity of Earth
D) Because helicopters built for Earth are too small
to handle the conditions on Mars
Unauthorized copying or reuse of any part of this page is illegal.
5
CONTINUE
Module
1
12
In West Africa, jalis have traditionally been keepers
of information about family histories and records of
important events. They have often served as teachers
and advisers, too. New technologies may have
changed some aspects of the role today, but jalis
continue to be valued for knowing and protecting
their peoples’ stories.
Which choice best states the main idea of the text?
A) Even though there have been some changes in
their role, jalis continue to preserve their
communities’ histories.
B) Although jalis have many roles, many of them
like teaching best.
C) Jalis have been entertaining the people within
their communities for centuries.
D) Technology can now do some of the things jalis
used to be responsible for.
...............................................................................................................................................................................................................................................................................................................
13
In 1934 physicist Eugene Wigner posited the
existence
of a crystal consisting entirely of electrons
in a honeycomb-like structure. The so-called Wigner
crystal remained largely conjecture, however, until
Feng Wang and colleagues announced in 2021 that
they had captured an image of one. The researchers
trapped electrons between two semiconductors and
then cooled the apparatus, causing the electrons to
settle into a crystalline structure. By inserting an
ultrathin sheet of graphene above the crystal, the
researchers obtained an impression—the first visual
confirmation of the Wigner crystal.
Which choice best states the main idea of the text?
A) Researchers have obtained the most definitive
evidence to date of the existence of the Wigner
crystal.
B) Researchers have identified an innovative new
method for working with unusual crystalline
structures.
C) Graphene is the most important of the
components required to capture an image of a
Wigner crystal.
D) It’s difficult to acquire an image of a Wigner
crystal because of the crystal’s honeycomb
structure.
Unauthorized copying or reuse of any part of this page is illegal.
6
CONTINUE
-
--
7
6
5
4
3
2
1
0
Average
Number
oflndividuals
Reporting Directly
to
CEOs
,f,<-,
󰸔
,o,°''
Years
󰲢
managers
󰲡
department
leaders
Module
1
14
Considering a large sample of companies,
economics experts Maria Guadalupe, Julie Wulf, and
Raghuram Rajan assessed the number of managers
and leaders from different departments who reported
directly to a chief executive officer (CEO). According
to the researchers, the findings suggest that across
the years analyzed, there was a growing interest
among CEOs in connecting with more departments
in their companies.
...............................................................................................................................................................................................................................................................................................................
Which choice best describes data from the graph that
support the researchers’ conclusion?
A) The average numbers of managers and
department leaders reporting directly to their
CEO didn’t fluctuate from the 1991–1995 period
to the 2001–2008 period.
B) The average number of managers reporting
directly to their CEO was highest in the
1996–2001 period.
C) The average number of department leaders
reporting directly to their CEO was greater than
the average number of managers reporting
directly to their CEO in each of the three periods
studied.
D) The average number of department leaders
reporting directly to their CEO rose over the
three periods studied.
Unauthorized copying or reuse of any part of this page is illegal.
7
CONTINUE
Module
1
15
When digging for clams, their primary food, sea
otters damage the roots of eelgrass plants growing
on the seafloor. Near Vancouver Island in Canada,
the otter population is large and well established,
yet the eelgrass meadows are healthier than those
found elsewhere off Canada’s coast. To explain
this, conservation scientist Erin Foster and
colleagues compared the Vancouver Island
meadows to meadows where otters are absent or
were reintroduced only recently. Finding that the
Vancouver Island meadows have a more diverse gene
pool than the others do, Foster hypothesized that
damage to eelgrass roots increases the plant’s rate of
sexual reproduction; this, in turn, boosts genetic
diversity, which benefits the meadow’s health overall.
Which finding, if true, would most directly
undermine Foster’s hypothesis?
A) At some sites in the study, eelgrass meadows are
found near otter populations that are small and
have only recently been reintroduced.
B) At several sites not included in the study, there
are large, well-established sea otter populations
but no eelgrass meadows.
C) At several sites not included in the study,
eelgrass meadows’ health correlates negatively
with the length of residence and size of otter
populations.
D) At some sites in the study, the health of plants
unrelated to eelgrass correlates negatively with
the length of residence and size of otter
populations.
...............................................................................................................................................................................................................................................................................................................
16
Scholars have noted that F. Scott Fitzgerald’s writings
were
likely influenced in part by his marriage to
Zelda Fitzgerald, but many don’t recognize Zelda as a
writer in her own right. Indeed, Zelda authored
several works herself, such as the novel Save Me the
Waltz and numerous short stories. Thus, those who
primarily view Zelda as an inspiration for F. Scott’s
writings ______
Which choice most logically completes the text?
A) overlook the many other factors that motivated
F. Scott to write.
B) risk misrepresenting the full range of Zelda’s
contributions to literature.
C) may draw inaccurate conclusions about how
F. Scott and Zelda viewed each other’s works.
D) tend to read the works of F. Scott and Zelda in an
overly autobiographical light.
Unauthorized copying or reuse of any part of this page is illegal.
8
CONTINUE
Module
1
17
Among social animals that care for their young,
such as chickens, macaque monkeys, and humans,
newborns appear to show an innate attraction to
faces and face-like stimuli. Elisabetta Versace and
her colleagues used an image of three black dots
arranged in the shape of eyes and a nose or mouth
to test whether this trait also occurs in Testudo
tortoises, which live alone and do not engage in
parental care. They found that tortoise hatchlings
showed a significant preference for the image,
suggesting that ______
Which choice most logically completes the text?
A) face-like stimuli are likely perceived as harmless
by newborns of social species that practice
parental care but as threatening by newborns of
solitary species without parental care.
B) researchers should not assume that an innate
attraction to face-like stimuli is necessarily an
adaptation related to social interaction or
parental care.
C) researchers can assume that the attraction to
face-like stimuli that is seen in social species that
practice parental care is learned rather than
innate.
D) newly hatched Testudo tortoises show a stronger
preference for face-like stimuli than adult
Testudo tortoises do.
...............................................................................................................................................................................................................................................................................................................
18
Compiled in the late 1500s largely through the
efforts
of Indigenous scribes, Cantares Mexicanos is
the most important collection of poetry in Classical
Nahuatl, the principal language of the Aztec Empire.
The poems portray Aztec society before the
occupation of the empire by the army of Spain, and
marginal notes in Cantares Mexicanos indicate that
much of the collection’s content predates the initial
invasion. Nonetheless, some of the poems contain
inarguable references to beliefs and customs
common in Spain during this era. Thus, some
scholars have concluded that ______
Which choice most logically completes the text?
A) while its content largely predates the invasion,
Cantares Mexicanos also contains additions
made after the invasion.
B) although those who compiled Cantares
Mexicanos were fluent in Nahuatl, they had
limited knowledge of the Spanish language.
C) before the invasion by Spain, the poets of the
Aztec Empire borrowed from the literary
traditions of other societies.
D) the references to beliefs and customs in
Spain should be attributed to a coincidental
resemblance between the societies of Spain
and the Aztec Empire.
Unauthorized copying or reuse of any part of this page is illegal.
9
CONTINUE
Module
1
-
----
-
----
...............................................................................................................................................................................................................................................................................................................
19
In a study of the cognitive abilities of white-faced
capuchin monkeys (Cebus imitator), researchers
neglected to control for the physical difficulty of the
tasks they used to evaluate the monkeys. The
cognitive abilities of monkeys given problems
requiring little dexterity, such as sliding a panel to
retrieve food, were judged by the same criteria as
were those of monkeys given physically demanding
problems, such as unscrewing a bottle and inserting a
straw. The results of the study, therefore, ______
Which choice most logically completes the text?
A) could suggest that there are differences in
cognitive ability among the monkeys even
though such differences may not actually exist.
B) are useful for identifying tasks that the monkeys
lack the cognitive capacity to perform but not for
identifying tasks that the monkeys can perform.
C) should not be taken as indicative of the cognitive
abilities of any monkey species other than
C. imitator.
D) reveal more about the monkeys’ cognitive
abilities when solving artificial problems than
when solving problems encountered in the wild.
20
To survive when water is scarce,
embryos
inside
African turquoise killifish eggs ______ a
dormant state known as diapause. In this state,
embryonic development is paused for as long as two
years—longer than the life span of an adult killifish.
Which choice completes the text so that it conforms
to the conventions of Standard English?
A) enter
B) to enter
C) having entered
D) entering
21
Formed in 1967 to foster
political and economic
stability
within the Asia-Pacific region, the
Association of Southeast Asian Nations was
originally made up of five members: Thailand, the
Philippines, Singapore, Malaysia, and Indonesia. By
the end of the 1990s, the organization ______ its
initial membership.
Which choice completes
the text
so that it conforms
to the conventions of Standard English?
A) has doubled
B) had doubled
C) doubles
D) will double
22
The intense pressure found in the deep ocean can
affect
the structure of proteins in fish’s cells,
distorting the proteins’ shape. The chemical
trimethylamine N-oxide (TMAO) counters this
effect, ensuring that proteins retain their original
______ is found in high concentrations in the cells of
the deepest-dwelling fish.
Which choice
completes the text
so that it conforms
to the conventions of Standard English?
A) configurations. TMAO
B) configurations TMAO
C) configurations, TMAO
D) configurations and TMAO
Unauthorized copying or reuse of any part of this page is illegal.
10
CONTINUE
Module
1
...............................................................................................................................................................................................................................................................................................................
23
Food and the sensation of taste are central to
Monique Truong’s novels. In The Book of Salt, for
example, the exiled character of Bình connects to his
native Saigon through the food he prepares, while in
Bitter in the Mouth, the character of Linda ______ a
form of
synesthesia whereby the words she hears
evoke tastes.
Which choice completes
the text so that it conforms
to the conventions of Standard English?
A) experienced
B) had experienced
C) experiences
D) will be experiencing
24
Inventor John Friedman created
a prototype of the
first
flexible straw by inserting a screw into a paper
straw and, using dental floss, binding the straw
tightly around the ______ When the floss and screw
were removed,
the resulting corrugations in the
paper allowed the straw to bend easily over the edge
of a glass.
Which choice completes
the text so that it conforms
to the conventions of Standard English?
A) screw’s thread’s.
B) screws’ threads.
C) screw’s threads.
D) screws threads’.
25
In her analysis of Edith Wharton’s The
House of
Mirth (1905), scholar Candace Waid observes that
the novel depicts the upper classes of New York
society as “consumed by the appetite of a soulless
______ an apt assessment given that The House
of
Mirth is set during the Gilded Age, a period marked
by rapid industrialization, economic greed, and
widening wealth disparities.
Which choice completes
the text so that it conforms
to the conventions of Standard English?
A) materialism”; and
B) materialism” and
C) materialism,”
D) materialism”
26
To humans, it does not appear that the golden
orb-weaver
spider uses camouflage to capture its
______ the brightly colored arachnid seems to wait
conspicuously in
the center of its large circular web
for insects to approach. Researcher Po Peng of the
University of Melbourne has explained that the
spider’s distinctive coloration may in fact be part of
its appeal.
Which choice completes the text so that it conforms
to the conventions of Standard English?
A) prey, rather,
B) prey rather,
C) prey, rather;
D) prey; rather,
Unauthorized copying or reuse of any part of this page is illegal.
11
CONTINUE
Module
1
27
In Death Valley National Park’s Racetrack Playa, a
flat, dry lakebed, are 162 rocks—some weighing less
than a pound but others almost 700 pounds—that
move periodically from place to place, seemingly of
their own volition. Racetrack-like trails in the ______
mysterious migration.
Which
choice completes
the text so that it conforms
to the conventions of Standard English?
A) playas sediment mark the rock’s
B) playa’s sediment mark the rocks
C) playa’s sediment mark the rocks’
D) playas’ sediment mark the rocks’
28
In crafting her fantasy fiction, Nigerian-born British
author
Helen Oyeyemi has drawn inspiration from
the classic nineteenth-century fairy tales of the
Brothers Grimm. Her 2014 novel Boy, Snow, Bird,
for instance, is a complex retelling of the story of
Snow White, while her 2019 novel ______ offers a
delicious twist
on the classic tale of Hansel and
Gretel.
Which choice completes
the text so that it conforms
to the conventions of Standard English?
A) Gingerbread
B) Gingerbread,
C) Gingerbread
D) Gingerbread:
...............................................................................................................................................................................................................................................................................................................
29
While researching a topic, a student has taken the
following
notes:
NASA uses rovers, large remote vehicles with
wheels, to explore the surface of Mars.
NASA’s rovers can’t explore regions inaccessible
to wheeled vehicles.
Rovers are also heavy, making them difficult to
land on the planet’s surface.
Microprobes, robotic probes that weigh as little as
50 milligrams, could be deployed virtually
anywhere on the surface of Mars.
Microprobes have been proposed as an alternative
to rovers.
The student wants to explain an advantage of
microprobes. Which choice most effectively uses
relevant information from the notes to accomplish
this goal?
A) Despite being heavy, NASA’s rovers can land
successfully on the surface of Mars.
B) Microprobes, which weigh as little as
50 milligrams, could explore areas of Mars that
are inaccessible to NASA’s heavy, wheeled
rovers.
C) NASA currently uses its rovers on Mars, but
microprobes have been proposed as an
alternative.
D) Though they are different sizes, both
microprobes and rovers can be used to explore
the surface of Mars.
Unauthorized copying or reuse of any part of this page is illegal.
12
CONTINUE
Module
1
30
While researching a topic, a student has taken the
following notes:
Abdulrazak Gurnah was awarded the 2021 Nobel
Prize in Literature.
Gurnah was born in Zanzibar in East Africa and
currently lives in the United Kingdom.
Many readers have singled out Gurnah’s 1994
book Paradise for praise.
Paradise is a historical novel about events that
occurred in colonial East Africa.
The student wants to introduce Paradise to an
audience unfamiliar with the novel and its author.
Which choice most effectively uses relevant
information from the notes to accomplish this goal?
A) Abdulrazak Gurnah, who wrote Paradise and
later was awarded the Nobel Prize in Literature,
was born in Zanzibar in East Africa and
currently lives in the United Kingdom.
B) Many readers have singled out Abdulrazak
Gurnah’s 1994 book Paradise, a historical novel
about colonial East Africa, for praise.
C) A much-praised historical novel about colonial
East Africa, Paradise (1994) was written by
Abdulrazak Gurnah, winner of the 2021 Nobel
Prize in Literature.
D) Paradise is a historical novel about events that
occurred in colonial East Africa, Abdulrazak
Gurnah’s homeland.
...............................................................................................................................................................................................................................................................................................................
31
While researching a topic, a student has taken the
following
notes:
Ulaanbaatar is the capital of Mongolia.
The city’s population is 907,802.
Ulaanbaatar contains 31.98 percent of Mongolia’s
population.
Hanoi is the capital of Vietnam.
The city’s population is 7,781,631.
Hanoi contains 8.14 percent of
Vietnam’s population.
Which choice most effectively uses information from
the given sentences to emphasize the relative sizes of
the two capitals’ populations?
A) Mongolia’s capital is Ulaanbaatar, which has
907,802 people, and Vietnam’s capital is Hanoi,
which has 7,781,631 people.
B) The populations of the capitals of Mongolia and
Vietnam are 907,802 (Ulaanbaatar) and
7,781,631 (Hanoi), respectively.
C) Even though Hanoi (population 7,781,631) is
larger than Ulaanbaatar (population 907,802),
Ulaanbaatar accounts for more of its
country’s population.
D) Comparing Vietnam and Mongolia, 7,781,631 is
8.14 percent of Vietnam’s population, and
907,802 is 31.98 percent of Mongolia’s.
Unauthorized copying or reuse of any part of this page is illegal.
13
CONTINUE
Module
1
32
While researching a topic, a student has taken the
following notes:
One of history’s greatest libraries was the House of
Wisdom in Baghdad, Iraq.
It was founded in the eighth century with the goal
of preserving all the world’s knowledge.
Scholars at the House of Wisdom collected
ancient and contemporary texts from Greece,
India, and elsewhere and translated them into
Arabic.
Writings included those of the Greek philosopher
Aristotle and the Indian mathematician
Aryabhata.
The House of Wisdom used Chinese papermaking
technology to create paper versions to be studied
and shared.
The student wants to explain how the House of
Wisdom preserved the world’s knowledge. Which
choice most effectively uses relevant information
from the notes to accomplish this goal?
A) The House of Wisdom was known for bringing
together knowledge from around the world,
including from Greece, India, and China.
B) Founded in Iraq in the eighth century, the House
of Wisdom employed many scholars as
translators.
C) Writings from the Greek philosopher Aristotle
and the Indian mathematician Aryabhata were
preserved at the House of Wisdom.
D) The House of Wisdom collected writings from
different countries and created paper versions in
Arabic to be studied and shared.
33
...............................................................................................................................................................................................................................................
While researching a topic, a student has taken the
following
notes:
British musicians John Lennon and
Paul McCartney shared writing credit for
numerous Beatles songs.
Many Lennon-McCartney songs were actually
written by either Lennon or McCartney, not by
both.
The exact authorship of specific parts of many
Beatles songs, such as the verse for “In My Life,” is
disputed.
Mark Glickman, Jason Brown, and Ryan Song
used statistical methods to analyze the musical
content of Beatles songs.
They concluded that there is
18.9% probability
that McCartney wrote the verse for “In My Life,”
stating that the verse is “consistent with Lennon’s
songwriting style.”
The student
wants to make a generalization about the
kind of study conducted by Glickman, Brown, and
Song. Which choice most effectively uses relevant
information from the notes to accomplish this goal?
A) Based on statistical analysis, Glickman, Brown,
and Song claim that John Lennon wrote the
verse of “In My Life.”
B)
There is only an 18.9% probability that
Paul McCartney wrote the verse for “In My Life”;
John Lennon is the more likely author.
C) It
is likely that John Lennon, not
Paul McCartney, wrote the verse for “In My
Life.”
D) Researchers have used statistical methods to
address questions of authorship within the field
of music.
STOP
If you finish before time is called, you may check your work on this module only.
Do not turn to any other module in the test.
Unauthorized copying or reuse of any part of this page is illegal.
14
No Test Material On This Page
Module
2
Reading and Writing
33
QUESTIONS
DIRECTIONS
The questions in this section address a
number
of
important
reading and
writing
skills.
Each
question includes one
or
more passages, which may include a table or graph.
Read
each passage
and question carefully, and then choose
the
best answer
to
the
question based on
the
passage(s).
All questions in this section are
multiple
-choice
with
four
answer choices.
Each
question
has
a
single best answer.
---------~ ---------~
1
The Mule Bone, a 1930 play written by Zora Neale
Hurston and Langston Hughes, is perhaps the
best-known of the few examples of
______ in
literature. Most writers
prefer working alone, and
given that working together cost Hurston and
Hughes their friendship, it is not hard to see why.
Which choice completes the
text with the most
logical and precise word or phrase?
A) characterization
B) interpretation
C) collaboration
D) commercialization
......................................................................................................................................................................................................
2
The process of mechanically recycling plastics
is
often considered
______ because of the
environmental impact and
the loss of material
quality that often occurs. But chemist Takunda
Chazovachii has helped develop a cleaner process of
chemical recycling that converts superabsorbent
polymers from diapers into a desirable reusable
adhesive.
Which choice completes the
text with the most
logical and precise word or phrase?
A) resilient
B) inadequate
C) dynamic
D) satisfactory
Unauthorized copying or reuse of any part of this page is illegal.
16
CONTINUE
Module
2
3
Interruptions in the supply chain for microchips
used in personal electronics have challenged an
economist’s assertion that retailers can expect robust
growth in sales of those devices in the coming
months. The delays are unlikely to ______ her
projection entirely
but will almost certainly extend
its time frame.
Which choice completes
the text with the most
logical and precise word or phrase?
A) dispute
B) withdraw
C) underscore
D) invalidate
4
For her 2021 art installation Anthem,
Wu Tsang
joined forces with singer and composer Beverly
Glenn-Copeland to produce a piece that critics
found truly ______: they praised Tsang for
creatively transforming
a museum rotunda into a
dynamic exhibit by projecting filmed images of
Glenn-Copeland onto a massive 84-foot curtain and
filling the space with the sounds of his and other
voices singing.
Which choice completes
the text with the most
logical and precise word or phrase?
A) restrained
B) inventive
C) inexplicable
D) mystifying
...............................................................................................................................................................................................................................................................................................................
5
Some scientists have suggested that mammals in the
Mesozoic
era were not a very ______ group, but
paleontologist Zhe-Xi
Luo’s research suggests that
early mammals living in the shadow of dinosaurs
weren’t all ground-dwelling insectivores. Fossils of
various plant-eating mammals have been found in
China, including species like Vilevolodon diplomylos,
which Luo says could glide like a flying squirrel.
Which choice completes
the text with the most
logical and precise word or phrase?
A) predatory
B) obscure
C) diverse
D) localized
Unauthorized copying or reuse of any part of this page is illegal.
17
CONTINUE
Module
2
6
The following text is adapted from Gwendolyn
Bennett’s 1926 poem “Street Lamps in Early Spring.”
Night wears a garment
All velvet soft, all violet blue...
And over her face she draws a veil
As shimmering fine as floating dew...
And here and there
In the black of her hair
The subtle hands of Night
Move slowly with their gem-starred light.
Which choice best describes the overall structure of
the text?
A) It presents alternating descriptions of night in a
rural area and in a city.
B) It sketches an image of nightfall, then an image
of sunrise.
C) It makes an extended comparison of night to a
human being.
D) It portrays how night changes from one season
of the year to the next.
...............................................................................................................................................................................................................................................................................................................
7
According to historian Vicki L. Ruiz, Mexican
American
women made crucial contributions to the
labor movement during World War II. At the time,
food processing companies entered into contracts to
supply United States armed forces with canned
goods. Increased production quotas conferred
greater bargaining power on the companies’
employees, many of whom were Mexican American
women: employees insisted on more favorable
benefits, and employers, who were anxious to fulfill
the contracts, complied. Thus, labor activism became
a platform for Mexican American women to assert
their agency.
Which choice best describes the function of the
underlined portion in the text as a whole?
A) It elaborates on a claim about labor relations in a
particular industry made earlier in the text.
B) It offers an example of a trend in the World
War II–era economy discussed earlier in the text.
C) It notes a possible exception to the historical
narrative of labor activism sketched earlier in the
text.
D) It provides further details about the identities of
the workers discussed earlier in the text.
Unauthorized copying or reuse of any part of this page is illegal.
18
CONTINUE
Module
2
-
-----~
-
-----~
8
The following text is adapted from Zora Neale
Hurston’s 1921 short story “John Redding Goes to
Sea.” John is a child who lives in a town in the woods.
Perhaps ten-year-old John was puzzling to the
folk there in the Florida woods for he was an
imaginative child and fond of day-dreams. The
St. John River flowed a scarce three hundred feet
from his back door. On its banks at this point
grow numerous palms, luxuriant magnolias
and bay trees. On the bosom of the stream
float millions of delicately colored hyacinths.
[John Redding] loved to wander down to the
water’s edge, and, casting in dry twigs, watch
them sail away down stream to Jacksonville, the
sea, the wide world and [he] wanted to follow
them.
Which choice best describes the function of the
underlined sentence in the text as a whole?
A) It provides an extended description of a location
that John likes to visit.
B) It reveals that some residents of John’s town are
confused by his behavior.
C) It illustrates the uniqueness of John’s
imagination compared to the imaginations
of other children.
D) It suggests that John longs to experience a larger
life outside the Florida woods.
...............................................................................................................................................................................................................................................................................................................
9
The following text is adapted from Oscar Wilde’s
1891
novel The Picture of Dorian Gray. Dorian Gray
is taking his first look at a portrait that Hallward has
painted of him.
Dorian passed listlessly in front of his picture
and turned towards it. When he saw it he drew
back, and his cheeks flushed for a moment with
pleasure. A look of joy came into his eyes, as if he
had recognized himself for the first time. He
stood there motionless and in wonder, dimly
conscious that Hallward was speaking to him,
but not catching the meaning of his words. The
sense of his own beauty came on him like a
revelation. He had never felt it before.
According to the text, what is true about Dorian?
A) He wants to know Hallward’s opinion of the
portrait.
B) He is delighted by what he sees in the portrait.
C) He prefers portraits to other types of paintings.
D) He is uncertain of Hallward’s talent as an artist.
Unauthorized copying or reuse of any part of this page is illegal.
19
CONTINUE
-
--
100
90
80
70
60
50
40
30
20
10
0
Voters' Political Orientation, Level
of
Political Information, and Probability
of
Voting
-
-
1
- - -
f--
f-- f--
- - -
f--
f-- f--
2 3 4 5 6
Voters' political orientation
( 1
= strong Democrat/liberal;
4
= independent;
>-
>-
7
7 = strong Republican/conservative)
󰲢
low information
󰲡high information
Module
2
10
Economists Kerwin Kofi Charles and Melvin
Stephens Jr. investigated a variety of factors that
influence voter turnout in the United States. Using
survey data that revealed whether respondents voted
in national elections and how knowledgeable
respondents are about politics, Charles and Stephens
claim that the likelihood of voting is driven in part by
potential voters’ confidence in their assessments of
candidates—essentially, the more informed voters
are about politics, the more confident they are at
evaluating whether candidates share their views, and
thus the more likely they are to vote.
...............................................................................................................................................................................................................................................................................................................
Which choice best describes data in the graph that
support Charles and Stephens’s claim?
A) At each point on the political orientation scale,
high-information voters were more likely than
low-information voters to vote.
B)
Only low-information voters who identify
as independents had a voting probability
below
50%.
C) The
closer that low-information voters are to the
ends of the political orientation scale, the more
likely they were to vote.
D) High-information voters were more likely to
identify as strong Democrats or strong
Republicans than low-information voters were.
Unauthorized copying or reuse of any part of this page is illegal.
20
CO N T I N U E
-
---
Spider Population
Count
90~-------------
..,
i:::
;::l
8
80
70
60
50
t 40
"O
;f
30
20
10
0
20
Day
of
experiment
30
I D
no
lizards
󰲡
with
lizards
Module
2
----------
11
To investigate the effect of lizard predation on spider
populations, a student in a biology class placed
spiders in two enclosures, one with lizards and one
without, and tracked the number of spiders in the
enclosures for 30 days. The student concluded that
the reduction in the spider population count in the
enclosure with lizards by day 30 was entirely
attributable to the presence of the lizards.
Which choice best describes data from the graph that
weaken the student’s conclusion?
A) The spider population count was the same in
both enclosures on day 1.
B) The spider population count also substantially
declined by day 30 in the enclosure without
lizards.
C) The largest decline in spider population count in
the enclosure with lizards occurred from day 1 to
day 10.
D) The spider population count on day 30 was
lower in the enclosure with lizards than in the
enclosure without lizards.
...............................................................................................................................................................................................................................................................................................................
12
Archaeologist Petra Vaiglova, anthropologist
Xinyi
Liu, and their colleagues investigated the
domestication of farm animals in China during the
Bronze Age (approximately 2000 to 1000 BCE). By
analyzing the chemical composition of the bones of
sheep, goats, and cattle from this era, the team
determined that wild plants made up the bulk of
sheep’s and goats’ diets, while the cattle’s diet
consisted largely of millet, a crop cultivated by
humans. The team concluded that cattle were likely
raised closer to human settlements, whereas sheep
and goats were allowed to roam farther away.
Which finding, if true, would most strongly support
the team’s conclusion?
A) Analysis of the animal bones showed that the
cattle’s diet also consisted of wheat, which
humans widely cultivated in China during the
Bronze Age.
B) Further investigation of sheep and goat bones
revealed that their diets consisted of small
portions of millet as well.
C) Cattle’s diets generally require larger amounts of
food and a greater variety of nutrients than do
sheep’s and goats’ diets.
D) The diets of sheep, goats, and cattle were found
to vary based on what the farmers in each
Bronze Age settlement could grow.
Unauthorized copying or reuse of any part of this page is illegal.
21
CO N T I N U E
Module
2
13
Mosasaurs were large marine reptiles that lived
in the Late Cretaceous period, approximately
100 million to 66 million years ago. Celina Suarez,
Alberto Pérez-Huerta, and T. Lynn Harrell Jr.
examined oxygen-18 isotopes in mosasaur tooth
enamel in order to calculate likely mosasaur body
temperatures and determined that mosasaurs were
endothermic—that is, they used internal metabolic
processes to maintain a stable body temperature
in a variety of ambient temperatures. Suarez,
Pérez-Huerta, and Harrell claim that endothermy
would have enabled mosasaurs to include relatively
cold polar waters in their range.
Which finding, if true, would most directly support
Suarez, Pérez-Huerta, and Harrell’s claim?
A) Mosasaurs’ likely body temperatures are easier
to determine from tooth enamel oxygen-18
isotope data than the body temperatures of
nonendothermic Late Cretaceous marine
reptiles are.
B) Fossils of both mosasaurs and nonendothermic
marine reptiles have been found in roughly equal
numbers in regions known to be near the poles
during the Late Cretaceous, though in lower
concentrations than elsewhere.
C) Several mosasaur fossils have been found in
regions known to be near the poles during the
Late Cretaceous, while relatively few fossils of
nonendothermic marine reptiles have been
found in those locations.
D) During the Late Cretaceous, seawater
temperatures were likely higher throughout
mosasaurs’ range, including near the poles, than
seawater temperatures at those same latitudes are
today.
...............................................................................................................................................................................................................................................................................................................
14
Researchers hypothesized that a decline in the
population
of dusky sharks near the mid-Atlantic
coast of North America led to a decline in the
population of eastern oysters in the region. Dusky
sharks do not typically consume eastern oysters but
do consume cownose rays, which are the main
predators of the oysters.
Which finding, if true, would most directly support
the researchers’ hypothesis?
A) Declines in the regional abundance of dusky
sharks’ prey other than cownose rays are
associated with regional declines in dusky shark
abundance.
B) Eastern oyster abundance tends to be greater in
areas with both dusky sharks and cownose rays
than in areas with only dusky sharks.
C) Consumption of eastern oysters by cownose rays
in the region substantially increased before the
regional decline in dusky shark abundance
began.
D) Cownose rays have increased in regional
abundance as dusky sharks have decreased in
regional abundance.
Unauthorized copying or reuse of any part of this page is illegal.
22
CO N T I N U E
Module
2
15
Political scientists who favor the traditional view of
voter behavior claim that voting in an election does
not change a voter’s attitude toward the candidates
in that election. Focusing on each US presidential
election from 1976 to 1996, Ebonya Washington
and Sendhil Mullainathan tested this claim by
distinguishing between subjects who had just become
old enough to vote (around half of whom actually
voted) and otherwise similar subjects who were
slightly too young to vote (and thus none of whom
voted). Washington and Mullainathan compared the
attitudes of the groups of subjects toward the
winning candidate two years after each election.
Which finding from Washington and Mullainathan’s
study, if true, would most directly weaken the claim
made by people who favor the traditional view of
voter behavior?
A) Subjects’ attitudes toward the winning
candidate two years after a given election were
strongly predicted by subjects’ general political
orientation, regardless of whether subjects were
old enough to vote at the time of the election.
B) Subjects who were not old enough to vote in a
given election held significantly more positive
attitudes towards the winning candidate
two years later than they held at the time of the
election.
C) Subjects who voted in a given election held
significantly more polarized attitudes toward
the winning candidate two years later than did
subjects who were not old enough to vote in that
election.
D) Two years after a given election, subjects who
voted and subjects who were not old enough to
vote were significantly more likely to express
negative attitudes than positive attitudes toward
the winning candidate in that election.
...............................................................................................................................................................................................................................................................................................................
Unauthorized copying or reuse of any part of this page is illegal.
23
CO N T I N U E
---
Power Conversion Efficiency
of
Lowest and Highest Performing
Spin-coated and Spray-coated
Electron
Transport
Layers
18~-------------
16--i---------,
14
12
10
8
6
4
2
o~-~-
Thickness
I
󰲢
spray coating
󰲡
spin coating
Module
2
16
Perovskite solar cells convert light into electricity
more efficiently than earlier kinds of solar cells, and
manufacturing advances have recently made them
commercially attractive. One limitation of the cells,
however, has to do with their electron transport layer
(ETL), through which absorbed electrons must pass.
Often the ETL is applied through a process called
spin coating, but such ETLs are fairly inefficient
at converting input power to output power.
André Taylor and colleagues tested a novel spray
coating method for applying the ETL. The team
produced ETLs of various thicknesses and concluded
that spray coating holds promise for improving the
power conversion efficiency of ETLs in perovskite
solar cells.
...............................................................................................................................................................................................................................................................................................................
Which choice best describes data from the graph that
support Taylor and colleagues’ conclusion?
A) Both the ETL applied through spin coating and
the
ETL applied through spray coating showed a
power conversion efficiency greater than
10% at
their lowest performing thickness.
B) The lowest
performing ETL applied through
spray coating had a higher power conversion
efficiency than the highest performing ETL
applied through spin coating.
C)
The highest performing ETL applied through
spray coating showed a power conversion
efficiency of approximately
13%, while the
highest performing ETL applied through spin
coating showed a power conversion efficiency of
approximately
11%.
D) There
was a substantial difference in power
conversion efficiency between the lowest and
highest performing ETLs applied through spray
coating.
Unauthorized copying or reuse of any part of this page is illegal.
24
CO N T I N U E
Module
2
17
...............................................................................................................................................................................................................................................................................................................
While attending school in New York City in the
1980s, Okwui Enwezor encountered few works by
African artists in exhibitions, despite New York’s
reputation as one of the best places to view
contemporary art from around the world. According
to an arts journalist, later in his career as a renowned
curator and art historian, Enwezor sought to remedy
this deficiency, not by focusing solely on modern
African artists, but by showing how their work fits
into the larger context of global modern art and art
history.
Which finding, if true, would most directly support
the journalist’s claim?
A) As curator of the Haus der Kunst in Munich,
Germany, Enwezor organized a retrospective of
Ghanaian sculptor El Anatsui’s work entitled El
Anatsui: Triumphant Scale, one of the largest art
exhibitions devoted to a Black artist in Europe’s
history.
B) In the exhibition Postwar: Art Between the Pacific
and the Atlantic, 1945–1965, Enwezor and
cocurator Katy Siegel brought works by African
artists such as Malangatana Ngwenya together
with pieces by major figures from other
countries, like US artist Andy Warhol and
Mexico’s David Siqueiros.
C) Enwezor’s work as curator of the 2001 exhibition
The Short Century: Independence and Liberation
Movements in Africa, 1945–1994 showed how
African movements for independence from
European colonial powers following the Second
World War profoundly influenced work by
African artists of the period, such as Kamala
Ibrahim Ishaq and Thomas Mukarobgwa.
D) Enwezor organized the exhibition In/sight:
African Photographers, 1940 to the Present not to
emphasize a particular aesthetic trend but to
demonstrate the broad range of ways in which
African artists have approached the medium of
photography.
18
For thousands of
years, people in the Americas
______
the bottle gourd, a large bitter fruit with a
thick rind, to make bottles, other types of containers,
and even musical instruments. Oddly, there is no
evidence that any type of bottle gourd is native to the
Western Hemisphere; either the fruit or its seeds
must have somehow been carried from Asia or
Africa.
Which choice completes the text so that it conforms
to the conventions of Standard English?
A) to use
B) have used
C) having used
D) using
19
While many video game creators strive to make their
graphics
ever more ______ others look to the past,
developing titles
with visuals inspired by the “8-bit”
games of the 1980s and 1990s. (The term “8-bit”
refers to a console whose processor could only
handle eight bits of data at once.)
Which choice completes
the text so that it conforms
to the conventions of Standard English?
A) lifelike but
B) lifelike
C) lifelike,
D) lifelike, but
Unauthorized copying or reuse of any part of this page is illegal.
25
CO N T I N U E
Module
2
...............................................................................................................................................................................................................................................................................................................
20
In the 1950s, a man named Joseph McVicker was
struggling to keep his business afloat when his sister-
in-law Kay Zufall advised him to repurpose the
company’s product, a nontoxic, clay-like substance
for removing soot from wallpaper, as a modeling
putty for kids. In addition, Zufall ______ selling the
product under a child-friendly name:
Play-Doh.
Which choice completes the text so
that it conforms
to the conventions of Standard English?
A) suggested
B) suggests
C) had suggested
D) was suggesting
21
Beatrix Potter is perhaps best known for writing
and
illustrating children’s books such as The Tale of
Peter Rabbit (1902), but she also dedicated herself
to mycology, the study of ______ more than
350 paintings of the fungal
species she observed in
nature and submitting her research on spore
germination to the Linnean Society of London.
Which choice completes the text so
that it conforms
to the conventions of Standard English?
A) fungi; producing
B) fungi. Producing
C) fungi producing
D) fungi, producing
22
In assessing the films of Japanese director
Akira
Kurosawa, ______ have missed his equally
deep engagement with Japanese artistic traditions
such as Noh theater.
Which choice completes the text so that it conforms
to the conventions of Standard English?
A) many critics have focused on Kurosawa’s use of
Western literary sources but
B) Kurosawa’s use of Western literary sources has
been the focus of many critics, who
C) there are many critics who have focused on
Kurosawa’s use of Western literary sources, but
they
D) the focus of many critics has been on Kurosawa’s
use of Western literary sources; they
23
Joshua Hinson, director of the language revitalization
program
of the Chickasaw Nation in Oklahoma,
helped produce the world’s first Indigenous-language
instructional app, Chickasaw ______ Chickasaw TV,
in 2010; and a Rosetta
Stone language course in
Chickasaw, in 2015.
Which choice completes the text so
that it conforms
to the conventions of Standard English?
A) Basic; in 2009, an online television network;
B) Basic; in 2009, an online television network,
C) Basic, in 2009; an online television network,
D) Basic, in 2009, an online television network,
Unauthorized copying or reuse of any part of this page is illegal.
26
CO N T I N U E
Module
2
...............................................................................................................................................................................................................................................................................................................
24
The forty-seven geothermal springs of Arkansas’ Hot
Springs National Park are sourced via a process
known as natural groundwater recharge, in which
rainwater percolates downward through the
earth—in this case, the porous rocks of the hills
around Hot ______ collect in a subterranean basin.
Which choice completes the text so that it conforms
to the conventions of Standard English?
A) Springs to
B) Springs: to
C) Springs—to
D) Springs, to
25
Over twenty years ago, in a landmark experiment in
the
psychology of choice, professor Sheena Iyengar
set up a jam-tasting booth at a grocery store. The
number of jams available for tasting ______ some
shoppers had twenty-four different options,
others
only six. Interestingly, the shoppers with fewer jams
to choose from purchased more jam.
Which choice completes the text so
that it conforms
to the conventions of Standard English?
A) varied:
B) varied,
C) varied, while
D) varied while
26
Nigerian author Buchi Emecheta’s celebrated literary
oeuvre
includes The Joys of Motherhood, a novel
about the changing roles of women in 1950s ______
a television play about the
private struggles of a
newlywed couple in Nigeria; and Head Above Water,
her autobiography.
Which choice completes the text so
that it conforms
to the conventions of Standard English?
A) Lagos, A Kind of Marriage,
B) Lagos; A Kind of Marriage,
C) Lagos, A Kind of Marriage:
D) Lagos; A Kind of Marriage
27
Chimamanda Ngozi Adichie’s 2013 novel
Americanah chronicles
the divergent experiences of
Ifemelu and Obinze, a young Nigerian couple, after
high school. Ifemelu moves to the United States to
attend a prestigious university. ______ Obinze
travels to London, hoping to
start a career there.
However, frustrated with the lack of opportunities,
he soon returns to Nigeria.
Which choice completes the text with
the most
logical transition?
A) Meanwhile,
B) Nevertheless,
C) Secondly,
D) In fact,
Unauthorized copying or reuse of any part of this page is illegal.
27
CO N T I N U E
Module
2
...............................................................................................................................................................................................................................................................................................................
28
Organisms have evolved a number of surprising
adaptations to ensure their survival in adverse
conditions. Tadpole shrimp (Triops longicaudatus)
embryos, ______ can pause development for over
ten years during extended periods of
drought.
Which choice completes the text with the
most
logical transition?
A) in contrast,
B) for example,
C) meanwhile,
D) consequently,
29
In 1933, the Twentieth Amendment to the US
Constitution
was ratified. The amendment
mandates that presidential inaugurations be held
on January 20, approximately ten weeks after the
November election. ______ this amendment requires
newly elected US senators and representatives
to be
sworn into their respective offices on January 3.
Which choice completes the text with the
most
logical transition?
A) Instead,
B) For instance,
C) Specifically,
D) In addition,
30
In her poetry collection Thomas
and Beulah, Rita
Dove
interweaves the titular characters’ personal
stories with broader historical narratives. She places
Thomas’s journey from the American South to the
Midwest in the early 1900s within the larger context
of the Great Migration. ______ Dove sets events
from Beulah’s personal life against the
backdrop of
the US Civil Rights Movement.
Which choice completes the text with the
most
logical transition?
A) Specifically,
B) Thus,
C) Regardless,
D) Similarly,
31
While researching a topic, a student has taken the
following
notes:
The Philadelphia and Lancaster Turnpike was a
road built between 1792 and 1794.
It was the first private turnpike in the
United States.
It connected the cities of Philadelphia and
Lancaster in the state of Pennsylvania.
It was sixty-two miles long.
The student wants to emphasize the distance covered
by the Philadelphia and Lancaster Turnpike. Which
choice most effectively uses relevant information
from the notes to accomplish this goal?
A) The sixty-two-mile-long Philadelphia and
Lancaster Turnpike connected the Pennsylvania
cities of Philadelphia and Lancaster.
B) The Philadelphia and Lancaster Turnpike was
the first private turnpike in the United States.
C) The Philadelphia and Lancaster Turnpike, which
connected two Pennsylvania cities, was built
between 1792 and 1794.
D) A historic Pennsylvania road, the Philadelphia
and Lancaster Turnpike was completed in 1794.
Unauthorized copying or reuse of any part of this page is illegal.
28
CO N T I N U E
Module
2
32
While researching a topic, a student has taken the
following notes:
Most, but not all, of the Moon’s oxygen comes
from the Sun, via solar wind.
Cosmochemist Kentaro Terada from
Osaka University wondered if some of the
unaccounted-for oxygen could be coming
from Earth.
In 2008, he analyzed data from the Japanese
satellite Kaguya.
Kaguya gathered data about gases and particles it
encountered while orbiting the Moon.
Based on the Kaguya data, Terada confirmed his
suspicion that Earth is sending oxygen to the
Moon.
The student wants to emphasize the aim of the
research study. Which choice most effectively uses
relevant information from the notes to accomplish
this goal?
A) As it orbited the Moon, the Kaguya satellite
collected data that was later analyzed by
cosmochemist Kentaro Terada.
B) Before 2008, Kentaro Terada wondered if the
Moon was receiving some of its oxygen from
Earth.
C) Cosmochemist Kentaro Terada set out to
determine whether some of the Moon’s oxygen
was coming from Earth.
D) Kentaro Terada’s study determined that Earth is
sending a small amount of oxygen to the Moon.
33
...............................................................................................................................................................................................................................................
While researching a topic, a student has taken the
following
notes:
Ducklings expend up to 62.8% less energy when
swimming in a line behind their mother than
when swimming alone.
The physics behind this energy savings hasn’t
always been well understood.
Naval architect Zhiming Yuan used computer
simulations to study the effect of the mother
duck’s wake.
The study revealed that ducklings are pushed in a
forward direction by the wake’s waves.
Yuan determined this push reduces the effect of
wave drag on the ducklings by
158%.
The
student wants to present the study and its
methodology. Which choice most effectively uses
relevant information from the notes to accomplish
this goal?
A)
A study revealed that ducklings, which expend
up to
62.8% less energy when swimming in a line
behind their mother, also experience
158% less
drag.
B) Seeking
to understand how ducklings swimming
in a line behind their mother save energy,
Zhiming Yuan used computer simulations to
study the effect of the mother duck’s wake.
C) Zhiming Yuan studied the physics behind the
fact that by being pushed in a forward direction
by waves, ducklings save energy.
D) Naval architect Zhiming Yuan discovered that
ducklings are pushed in a forward direction by
the waves of their mother’s wake, reducing the
effect of drag by
158%.
STOP
If you finish before time is called, you may check your work on this module only.
Do not turn to any other module in the test.
Unauthorized copying or reuse of any part of this page is illegal.
29
Module
1
Math
27
QUESTIONS
DIRECTIONS
The questions in this section address a
number
of
important
math skills.
Use
of
a calculator
is
permitted for all questions.
,a-nH
NOTES
Unless otherwise indicated:
All variables and expressions represent
real numbers.
Figures provided are drawn
to
scale.
All figures lie in a plane.
The domain
of
a given function
f
is
the
set
of
all real numbers x
for
which f(x)
is
a real number.
REFERENCE
Unauthorized copying or reuse of any part of this page is illegal.
30
CONTINUE
li04ii1Ut31
G
£

b~
CJw
~x
0
b
a
x--../3
'~
45°
s
A
=nr
2
C=2nr
A=
Rw
A =_!bh
2
c2
=
a2
+
b2
Special Right Triangles
V=
R
wh
V=
nr
2
h
V
=
i,.r3
3
V=½nr
2
h
1
V =
3
£
wh
62h
E}
£
The
number
of
degrees
of
arc in a circle
is
360.
The
number
of
radians
of
arc in a circle
is
2n.
@
4
The sum
of
the
measures in degrees
of
the
angles
of
a triangle
is
180.
£
Module
1
For multiple-choice questions, solve each problem, choose the correct
answer from
the
choices provided, and then circle your answer in this book.
Circle only one answer for each question. If you change your mind, completely
erase the circle.
You
will
not
get
credit
for
questions
with
more than one
answer circled, or
for
questions
with
no answers circled.
For student-produced response questions, solve each problem and write
your answer
next
to
or
under
the
question in
the
test
book
as
described below.
Once you've written your answer, circle
it
clearly.
You
will
not
receive credit
for
anything
written
outside
the
circle, or
for
any questions
with
more than
one circled answer.
If
you find more than one correct answer, write and circle only one answer.
Your answer
can
be
up
to
5 characters
for
a positive answer and up
to
6 characters (including the negative sign) for a negative answer,
but
no more.
If
your answer
is
a fraction
that
is
too
long (over 5 characters
for
positive,
6 characters
for
negative), write
the
decimal equivalent.
If
your answer
is
a decimal
that
is
too
long (over 5 characters for positive,
6 characters
for
negative), truncate
it
or round at the
fourth
digit.
If
your answer
is
a mixed number (such
as
3
.!.
2
),
write
it
as
an
improper
fraction (7/2) or its decimal equivalent
(3
.
5)
.
Don't
include symbols such
as
a percent sign, comma, or dollar sign in
your circled answer.
Unauthorized copying or reuse of any part of this page is illegal.
31
CONTINUE
Module
---------~
"'
'"'

.....
15%

5%
r:::
Q)
I:!
Q)
11..
'
'
'
\.
'
-
-
I
I
'
I
Model year
---------~
1
-
t
m n
33
1
The line graph shows the percent of cars for sale at a
used car lot on a given day by model year.
For what model year is the percent of cars for sale the
smallest?
A) 2012
B) 2013
C) 2014
D) 2015
2
For a particular machine that produces beads,
29 out
of
every
100 beads it produces have a defect. A bead
produced by the machine will be selected at random.
What is the probability of selecting a bead that has a
defect?
A)
1
2,900
B)
1
29
C)
29
100
D)
29
10
...............................................................................................................................................................................................................................................................................................................
3
Note: Figure
not
drawn to scale.
In the figure, line m is parallel to line n, and line t
intersects both lines. What is the value of x ?
A) 33
B) 57
C) 123
D) 147
Unauthorized copying or reuse of any part of this page is illegal.
32
CONTINUE
-----
y
114
11
19
8

y
__
.....
6
l
l
f
1
T
- - - -
-10
8 6 4 2
u
.
I
I
I
X
246810
Module
1
----------
----------
----------
----------
...............................................................................................................................................................................................................................................................................................................
4
What is the y-intercept of the graph shown?
A)
(−8, 0)
B)
(−6, 0)
C)
(0, 6)
D)
(0, 8)
5
The total cost
fx()
, in dollars, to lease a car for
36 months from a particular car dealership is given
by
fx()= 36x + 1,000
, where x
is the monthly
payment, in dollars. What is the total cost to lease a
car when the monthly payment is $400?
A) $13,400
B) $13,000
C) $15,400
D) $37,400
6
Each side of a square has a length of 45. What
is the
perimeter
of this square?
7
55
=
x
x
+6
What is the positive solution to the given equation?
8
An object travels at a constant speed of
12
centimeters per second. At this speed, what is the
time, in seconds, that it would take for the object to
travel 108 centimeters?
A) 9
B) 96
C) 120
D) 972
Unauthorized copying or reuse of any part of this page is illegal.
33
CONTINUE
Module
1
-
------~
-
------~
-
------~
-
------~
-
------~
-
------~
9
Data set X: 5, 9, 9, 13
Data set Y: 5, 9, 9, 13, 27
The lists give the values in data sets X and Y. Which
statement correctly compares the mean of data set X
and the mean of data set Y?
A) The mean of data set X is greater than the mean
of data set Y.
B) The mean of data set X is less than the mean of
data set Y.
C) The means of data set X and data set Y are equal.
D) There is not enough information to compare the
means.
10
A rocket contained 467,000 kilograms (kg) of
propellant
before launch. Exactly 21 seconds after
launch, 362,105 kg of this propellant remained. On
average, approximately how much propellant, in kg,
did the rocket burn each second after launch?
A) 4,995
B) 17,243
C) 39,481
D) 104,895
11
If
4+x 2 =12
, what is the value of
16x + 8
?
A) 40
B) 48
C) 56
D) 60
...............................................................................................................................................................................................................................................................................................................
12
An object is kicked from a platform. The equation
h
represents this situation, where
h is the height of the object above the ground, in
meters, t
seconds after it is kicked.
Which number
represents the height, in meters, from which the
object was kicked?
A) 0
B) 4.9
C) 7
D) 9
=−4.9t
2
+7t +9
13
fx()= 4x
2
50x + 126
The given equation defines the function f. For what
value of x
does
fx()
reach its minimum?
14
A small business owner budgets $2,200 to purchase
candles.
The owner must purchase a minimum of
200 candles to maintain the discounted pricing. If the
owner pays $4.90 per candle to purchase small
candles and $11.60 per candle to purchase large
candles, what is the maximum number of large
candles the owner can purchase to stay within the
budget and maintain the discounted pricing?
Unauthorized copying or reuse of any part of this page is illegal.
34
CONTINUE
Module
1
----------
----
----------
----------
...............................................................................................................................................................................................................................................................................................................
15
In the linear function f,
f ()0=8
and
f ()1= 12
.
Which equation defines f ?
A)
fx()=12x +8
B)
fx()=4x
C)
fx()=4x + 12
D)
fx()=4x +8
16
The function
fw()=6w
2
gives the area of a
rectangle, in square feet
(ft
2
)
, if its width is w ft and
its length is 6 times its width. Which of the following
is the best interpretation of
f () 14 = 1,176
?
A) If the width of the rectangle
is 14 ft, then the area
of the rectangle is 1,176
ft
2
.
B) If the width of the rectangle is 14 ft, then the
length of the rectangle is 1,176 ft.
C) If the width of the rectangle is 1,176 ft, then the
length of the rectangle is 14 ft.
D) If the width of the rectangle is 1,176 ft, then the
area of the rectangle is 14
ft
2
.
17
Note: Figure
not
drawn to scale.
The circle shown has center O, circumference
144π
,
and diameters
PR
and
QS
. The length of arc
PS
is
twice the length of arc
PQ
. What is the length of
arc
QR
?
A)
24π
B)
48π
C)
72π
D)
96π
18
A company that provides whale-watching tours takes
groups of 21 people at a time. The company’s
revenue is 80 dollars per adult and 60 dollars per
child. If the company’s revenue for one group
consisting of adults and children was 1,440 dollars,
how many people in the group were children?
A) 3
B) 9
C) 12
D) 18
Unauthorized copying or reuse of any part of this page is illegal.
35
CONTINUE
Module
1
-
------~
-
------~
-
------~
-
------~
-
------~
...............................................................................................................................................................................................................................................................................................................
19
The function h is defined by
hx()=4x + 28
. The
graph of xy
y = h(x)
in the -plane has an x-intercept
at
(,a 0)
and
a y-intercept at
(0, b)
, where a
and b
are constants. What is the value of
a
+ b
?
A) 21
B) 28
C) 32
D) 35
20
One of the factors of
2x
3
+ 42x
2
+ 208x
is
x
+ b
,
where b is a positive constant. What is the smallest
possible value of b ?
21
y =−1.5
y = x
2
+8x + a
In the given system of equations, a is a positive
constant. The system has exactly one distinct real
solution. What is the value of a ?
22
f(x)= (x +6)(x +5)(x 4)
The function f
is given. Which table of values
represents
y = f()x3
?
A)
x y
−6 −9
−5
−8
4 1
B)
x
y
−6 −3
−5
−3
4
−3
C)
x y
−6 −3
−5
−2
4
7
D)
x y
−6 3
−5
3
4
3
23
For the function q, the value of
qx()
decreases by
45% for every increase in the value of x
by 1. If
q()0=14
, which equation defines q ?
A)
qx()=0.55 (14 )
x
B)
qx()=1.45(14 )
x
C)
qx()=14(0.55)
x
D)
qx()=14(1.45)
x
Unauthorized copying or reuse of any part of this page is illegal.
36
CONTINUE
24
Module
1
-----
----------
y
10
8
6
....
4
--
--
,__
--
10
-8
-6
-4
-2-Ll
->-2-
4-
-6-
>-8-
~)
:2
X
14
I
I 6
I
lg
I
'10
The graph of
y = f ()x + 14
is shown. Which
equation defines function f ?
A)
1
fx()=− x −12
4
B)
1
fx() =− x +16
4
C)
1
fx() =− x +2
4
D)
1
fx() =− x −14
4
...............................................................................................................................................................................................................................................................................................................
25
RS =20
ST =48
TR =52
The side lengths of right triangle
RST
are given.
Triangle
RST
is similar to triangle
UVW
, where S
corresponds to V and T corresponds to W. What is
the value of
tan W
?
A)
5
13
B)
5
12
C)
12
13
D)
12
5
Unauthorized copying or reuse of any part of this page is illegal.
37
CONTINUE
Module
1
-
-----~
-
-----~
26
...............................................................................................................................................................................................................................................
One gallon of paint will cover 220 square feet of a
surface. A room has a total wall area of w square feet.
Which equation represents the total amount of
paint P, in gallons, needed to paint the walls of the
room twice?
A)
w
P =
110
B)
P = 440w
C)
w
P =
220
D)
P = 220w
27
The number a is 110% greater than the number b.
The number b is 90% less than 47. What is the
value of a ?
STOP
If you finish before time is called, you may check your work on this module only.
Do not turn to any other module in the test.
Unauthorized copying or reuse of any part of this page is illegal.
38
No Test Material On This Page
Module
2
Math
27
QUESTIONS
DIRECTIONS
The questions in this section address a
number
of
important
math skills.
Use
of
a calculator
is
permitted for all questions.
NOTES
,a-nH
Unless otherwise indicated:
All variables and expressions represent
real numbers.
Figures provided are drawn
to
scale.
All figures lie in a plane.
The domain
of
a given function
f
is
the
set
of
all real numbers x
for
which f(x)
is
a real number.
REFERENCE
li04ii1Ut31
G
£

b~
CJw
~x
0
b
a
x--../3
'~
45°
s
A
=nr
2
C=2nr
A=
Rw
A =_!bh
2
c2
=
a2
+
b2
Special Right Triangles
V=
R
wh
V=
nr
2
h
V
=
i,.r3
3
V=½nr
2
h
1
V =
3
£
wh
62h
E}
£
The
number
of
degrees
of
arc in a circle
is
360.
The
number
of
radians
of
arc in a circle
is
2n.
@
4
The sum
of
the
measures in degrees
of
the
angles
of
a triangle
is
180.
£
Unauthorized copying or reuse of any part of this page is illegal.
40
CONTINUE
Module
2
For multiple-choice questions, solve each problem, choose the correct
answer from
the
choices provided, and then circle your answer in this book.
Circle only one answer for each question. If you change your mind, completely
erase the circle.
You
will
not
get
credit
for
questions
with
more than one
answer circled, or
for
questions
with
no answers circled.
For student-produced response questions, solve each problem and write
your answer
next
to
or
under
the
question in
the
test
book
as
described below.
Once you've written your answer, circle
it
clearly.
You
will
not
receive credit
for
anything
written
outside
the
circle, or
for
any questions
with
more than
one circled answer.
If
you find more than one correct answer, write and circle only one answer.
Your answer
can
be
up
to
5 characters
for
a positive answer and up
to
6 characters (including the negative sign) for a negative answer,
but
no more.
If
your answer
is
a fraction
that
is
too
long (over 5 characters
for
positive,
6 characters
for
negative), write
the
decimal equivalent.
If
your answer
is
a decimal
that
is
too
long (over 5 characters for positive,
6 characters
for
negative), truncate
it
or round at the
fourth
digit.
If
your answer
is
a mixed number (such
as
3
.!.
2
),
write
it
as
an
improper
fraction (7/2) or its decimal equivalent
(3
.
5)
.
Don't
include symbols such
as
a percent sign, comma, or dollar sign in
your circled answer.
Unauthorized copying or reuse of any part of this page is illegal.
41
CONTINUE
1
There are 55 students in Spanish club. A sample of
the Spanish club students was selected at random
and
asked whether they intend to enroll in a new
study program. Of those surveyed,
20%
responded
that they intend to enroll in the study program.
Based on this survey, which of the following is the
best estimate of the total number of Spanish club
students who intend to enroll in the study program?
A) 11
B) 20
C) 44
D) 55
2
Jay walks at a speed of 3 miles per hour and runs at a
speed of 5 miles per hour. He walks for w hours and
runs for r hours for a combined total of 14 miles.
Which equation represents this situation?
A)
w =3+5r 14
B)
1
1
w
+
r
=14
3
5
C)
1
1
w
+
r
=
112
3
5
D)
3w + 5r = 112
...............................................................................................................................................................................................................................................................................................................
3
The scatterplot shows the relationship between
two variables, x and y. A line of best fit is also
shown.
Module
2
---------~ ---------~
---------~
16
14
12
10
8
6
4
2
y
7
u
,;
/
::
L
V -
/
,,
-
z
/
/
/
v_
7

X
1 2 3 4 5 6 7 8
Which of the following equations best represents the
line of best fit shown?
A)
y = 2.8 + 1.7x
B)
y = 2.8 1.7x
C)
y = −2.8 + 1.7x
D)
y = −2.8 1.7x
Unauthorized copying or reuse of any part of this page is illegal.
42
CONTINUE
Module
2
-----
----------
y
8
7
I
6
I
5
I
4
,.
L;'
-z'
;i
8-.=6-.=4-.=2
_
1~
>-
_
z_
_ 4 _
_6
_
f--8-
X
2
,.
3
,,
4
I
-~--------
5
6
I
7
I
8
----------
----------
4
The graph of
y f x= ( )
is shown in the xy-plane.
What is the value of
f 0( )
?
A)
−3
B)
0
C)
3
5
D)
3
5
Which expression is equivalent to
(m q z )(mq z )
4 4 −1 5 3
,
where m, q, and z are positive?
A)
m q z
4 20 −3
B)
m q z
5 9
2
C)
m q z
6 8 −1
D)
m q z
20 12 −2
...............................................................................................................................................................................................................................................................................................................
6
73, 74, 75, 77, 79, 82, 84, 85, 91
What is the median of the data shown?
7
x + 40 = 95
What value of x is the solution to the given
equation?
8
5x = 15
−4x + y = −2
The solution to the given system of equations is
x y( , )
. What is the value of
x y+
?
A)
−17
B)
−13
C)
13
D)
17
Unauthorized copying or reuse of any part of this page is illegal.
CO N T I N U E
43
Module
2
-
-----~
-
-----~
-
-----~
-
-----~
9
gm) = −0.05m( + 12.1
The given function g models the number of gallons
of gasoline that remains from a full gas tank in a car
after driving m miles. According to the model, about
how many gallons of gasoline are used to drive
each mile?
A) 0.05
B) 12.1
C) 20
D) 242.0
10
1 11x
=
7b y
The given equation relates the positive numbers b, x,
and y. Which equation correctly expresses x in terms
of b and y ?
A)
11
x =
7by
B)
x =−77by
C)
x
=
y
77b
D)
x =77by
...............................................................................................................................................................................................................................................................................................................
11
y =76
y = x
2
5
The graphs of the given equations in the xy-plane
intersect at the point
(,xy)
. What is a possible value
of x ?
A)
76
5
B)
−9
C)
5
D)
76
12
y >14
4+ xy < 18
The point
(x, 53)
is a solution to the system of
inequalities in the xy-plane. Which of the following
could be the value
of x ?
A)
−9
B)
−5
C)
5
D)
9
Unauthorized copying or reuse of any part of this page is illegal.
44
CONTINUE
Module
2
-
------~
-
------~
-
------~
-
------~
-
------~
13
Out of 300 seeds that were planted,
80%
sprouted.
How many of these seeds sprouted?
14
The function f
is defined by
fx()=4x
. For what
value of x
does
fx()=8
?
15
Which expression is equivalent to
8xx( −7)−3(x −7)
2−14 x
, where
x >7
?
A)
x −7
5
B)
x8−3
2
C)
2
8−3x x 14
x2−14
D)
2
8−3x x 77
x2−14
...............................................................................................................................................................................................................................................................................................................
16
Line p
is defined by
2+y 18x =9
. Line r
is
perpendicular to line p in the xy-plane. What is the
slope of line r
?
A)
−9
B)
1
9
C)
1
9
D)
9
17
ft( ) = 8,000(0.65)
t
The given function f
models the number of coupons
a company sent to their customers at the end of each
year, where t
represents the number of years since
the end of
1998, and
0≤ t ≤5
.If
y =(ft )
is graphed
in the
ty-plane, which of the following is the best
interpretation of the y-intercept of the graph in this
context?
A) The minimum estimated number
of coupons the
company sent to their customers during the
5 years was 1,428.
B) The minimum estimated number of coupons the
company sent to their customers during the
5 years was 8,000.
C) The estimated number of coupons the company
sent to their customers at the end of 1998
was 1,428.
D) The estimated number of coupons the company
sent to their customers at the end of 1998
was 8,000.
Unauthorized copying or reuse of any part of this page is illegal.
45
CONTINUE
Module
2
-
------~
-
------~
-
------~
-
------~
-
------~
-
------~
18
...............................................................................................................................................................................................................................................................................................................
Triangle
XYZ
is similar to triangle
RST
such that X,
Y, and Z
correspond to R, S, and T, respectively.
The measure of
Z
is
20°
and
2XY = RS
. What is
the measure of
T
?
A)
B)
10°
C)
20°
D)
40°
19
y =6x + 18
One of the equations in a system of two linear
equations is given. The system has no solution.
Which equation could be the second equation in the
system?
A)
−6xy+ = 18
B)
−6xy+ = 22
C)
−12xy+ = 36
D)
−12xy+ = 18
20
What is the area, in square centimeters, of a rectangle
with a length of 34 centimeters (cm) and a width of
29 cm?
21
y =4x +1
4=y 15x 8
The solution to the given system of equations is
(,xy)
. What is the value of
xy
?
22
2
5x + 10x +16 =0
How many distinct real solutions does the given
equation have?
A) Exactly one
B) Exactly two
C) Infinitely many
D) Zero
23
A certain park has an area of
11,863,808
square
yards. What is the area, in square miles, of this park?
(
1 mile = 1,760 yards
)
A) 1.96
B) 3.83
C) 3,444.39
D) 6,740.8
Unauthorized copying or reuse of any part of this page is illegal.
46
CONTINUE
Module
2
----------
-
----------
12
10

8
i 6

4
µ..
2
0
Data Set A
H
I
10 20 30 40 50 60
Integer
Data Set B
12~-----
l0-+------

8+------1
i
6+---l

4-+--~--t
µ..
2
0-+---+---+---+---+----+-
10 20 30 40 50
60
Integer
----------
24
Which of the following equations represents a circle
in the xy-plane that intersects the y-axis at exactly
one point?
A)
(x 8)
2
+ (y 8)
2
= 16
B)
(x 8)
2
+ (y 4)
2
= 16
C)
(x 4)
2
+ (y 9)
2
= 16
D)
x
2
+(y 9)
2
= 16
25
In triangles
ABC
and
DEF
, angles B and E each
have measure
27°
and angles C and F each have
measure
41°
. Which additional piece of information
is sufficient to determine whether triangle
ABC
is
congruent to triangle
DEF
?
A) The measure of angle A
B) The length of side
AB
C) The lengths of sides
BC
and
EF
D) No additional information is necessary.
.................................................................................................................................................................................................................................................
26
Two data sets of 23 integers each are summarized in
the histograms shown. For each of the histograms,
the first interval represents the frequency of integers
greater than or equal to 10, but less than 20. The
second interval represents the frequency of integers
greater than or equal to 20, but less than 30, and so
on. What is the smallest possible difference between
the mean of data set A and the mean of data set B?
A) 0
B) 1
C) 10
D) 23
27
A right triangle has legs with lengths of
24
centimeters and 21 centimeters. If the
length of
this triangle’s hypotenuse, in centimeters, can be
written in the form
3 d
, where d is an integer, what
is the value of d ?
STOP
If you finish before time is called, you may check your work on this module only.
Do not turn to any other module in the test.
Unauthorized copying or reuse of any part of this page is illegal.
47
No Test Material On This Page
No Test Material On This Page
No Test Material On This Page
No Test Material On This Page
No Test Material On This Page
No Test Material On This Page
The
SAT.
The
SAT
®
GENERAL
DIRECTIONS
You
may work on only one module
at
a time.
If you finish a module before time is called,
check
your
work
on
that
module only.
You
may
NOT
turn
to
any
other
module.
MARKING
YOUR ANSWERS
Be
sure
to
answer your questions properly in this
book
.
Circle only one answer
to
each question. If you change your mind, completely
erase the circle.
You
will
not
get
credit
for
questions with more than one answer
circled,
or
for
questions with no answers circled.
USING
YOUR
TEST
BOOK
You
may use
the
test
book
for
scratch work.
You
may
not
fold
or
remove pages
or
portions
of
a page from this book,
or
take the
book
from
the
testing room.
II
I
II
II
I
II
I
IIIII
II
I
111111111111111
© 2022 College Board. College Board, SAT, and the acorn logo are registered trademarks of College Board.
WF2P0007
The SAT
®
Practice
Test
#
2
ANSWER EXPLANATIONS
These answer explanations are for students taking the
digital SAT in nondigital format.
© 2022 College Board. College Board, SAT, and the acorn logo are registered trademarks of College Board.
SAT ANSWER EXPLANATIONS
n
READING AND WRITING: MODULE 1
2 SAT PRACTICE TEST #2 ANSWER EXPLANATIONS
Reading and Writing
Module 1
(33 questions)
QUESTION 1
Choice B is the best answer because it most logically completes the text’s
discussion of Juarez. In this context, “important” means marked by significant
work or consequence. The text indicates that Juarez, who was the first president
of Mexico from an Indigenous community, became a certain kind of figure in
Mexico’s history. It then supports that claim by describing some of the “many
significant accomplishments” from Juarez’s long tenure in office. This context
conveys that Juarez is a significant and consequential figure in Mexico’s history.
Choice A is incorrect because the text focuses on Juarez’s role as the first
president of Mexico from an Indigenous community and on his many major
accomplishments during his lengthy time in office; nothing in the text suggests
that Juarez was “unpredictable,” or tended to behave in ways that couldn’t be
predicted. Choice C is incorrect because nothing in the text suggests that Juarez
was a particularly “secretive” figure, or that he tended to keep things private or
hidden from others. Instead, the text focuses on things that are known about
Juarez: that he was the first president of Mexico from an Indigenous community,
that he had a lengthy tenure, and that his many major accomplishments included
consolidating the national government’s authority and advancing Indigenous
rights. Choice D is incorrect because the text focuses on the idea that Juarez,
who was the first president of Mexico from an Indigenous community, had many
major accomplishments during his lengthy time in office. Rather than suggesting
that Juarez was an “ordinary,” or common and typical, figure in Mexico’s history,
this context conveys that Juarez was instead a notable figure.
SAT ANSWER EXPLANATIONS
n
READING AND WRITING: MODULE 1
3 SAT PRACTICE TEST #2 ANSWER EXPLANATIONS
QUESTION 2
Choice C is the best answer because it most logically completes the text’s
discussion of John Ashbery’s poems. As used in this context, “interpret” would
mean decipher the meaning of. The text indicates that Ashbery’s poems have
many unusual features, that it’s difficult to tell what exactly the poems’ subject
matter is, and that scholars strongly disagree about the poems. This context
conveys the idea that it’s difficult to interpret Ashbery’s poems.
Choice A is incorrect because “delegate” means to assign someone as a
representative of another person or to entrust something to someone else,
neither of which would make sense in context. The text is focused only on the
difficulty that readers have interpreting Ashbery’s poems due to their many
unusual features; it doesn’t suggest anything about the poems being difficult to
delegate. Choice B is incorrect because describing Ashbery’s poems as difficult
to “compose,” or put together or produce, would make sense only if the text were
about Ashbery’s experience of writing the poems. It could be true that it was
difficult for Ashbery to compose his poems, but the text doesn’t address this; it
instead discusses how readers interpret and engage with the poems. Choice D is
incorrect because describing Ashbery’s poems as being difficult to “renounce,”
or give up or refuse, wouldn’t make sense in context. The text focuses on the idea
that features of Ashbery’s poems are odd or unclear and have caused heated
scholarly debate. This context suggests that the poems are difficult to interpret,
not that the poems are difficult to renounce.
QUESTION 3
Choice C is the best answer because it most logically and precisely completes
the text’s discussion of the fossil record from the Cambrian period. In this context,
“abrupt” means sudden. The text explains that the fossil record reflects the
unexpected appearance and rapid diversification, or increase in variety, of animal
remains during the Cambrian period. This context establishes that these remains’
entry into the fossil record was sudden.
Choice A is incorrect. Although the word “explosion” appears in the name of
the event marked by the fossil record change, the text never suggests that the
change was “catastrophic,” or disastrous. In context, “explosion” refers to the
rapid diversification, or the swift increase in variety, of animal remains in the fossil
record—a phenomenon that the text presents in a relatively neutral manner,
without commenting on whether it was negative or positive. Choice B is incorrect
because the text never suggests that the change toward greater diversification is
elusive,” or difficult to locate, in the fossil record. Rather, the text notes that the
change occurred about 541 million years ago, suggesting that scientists have
indeed been able to locate it. Choice D is incorrect because it wouldn’t make
sense in context to describe the change in the fossil record as “imminent,” or
about to occur, since the text indicates that the change already occurred millions
of years ago.
SAT ANSWER EXPLANATIONS
n
READING AND WRITING: MODULE 1
4 SAT PRACTICE TEST #2 ANSWER EXPLANATIONS
QUESTION 4
Choice B is the best answer because it most logically completes the text’s
discussion of the significance of the 2014 archaeological finding at El Algar. In
this context, “concede” means to admit something is true after first resisting
that admission. The text indicates that some researchers believe “Bronze Age
societies were ruled by men.” But the Bronze Age burial of a woman at El Algar
included “valuable objects signaling a high position of power,” which would raise
the possibility that “women may have also held leadership roles.” Thus, the text is
calling into question the notion that only men were leaders in these societies and
speculating that people holding this view may reconsider their opinion.
Choice A is incorrect because “waive” means to refrain from insisting that
something, such as a right or a requirement, be observed; the word isn’t used,
however, in contexts where someone acknowledges that an opinion they hold
may be invalid, as is the case in the text. Choice C is incorrect. According to the
text, the finding from the El Algar burial site undermines the view that Bronze Age
societies were exclusively ruled by men. However, “refute” means to demonstrate
that something is false and would not make sense in context. Lull and team’s
finding supports the view that women may have also held leadership roles, not
that they did not participate in such roles. Choice D is incorrect because in this
context, “require” means to demand or specify as mandatory. However, it would
not make sense for contemporary researchers to demand that Bronze Age
“women may have also held leadership roles.”
QUESTION 5
Choice D is the best answer because it most logically completes the text’s
discussion of baleen whale accessory spleens. In this context, “latent” means
dormant or functionless. The text sets up a contrast between the idea that
baleen whale accessory spleens appear not to have a function and the research
indicating that the accessory spleen may actually have a role in supporting the
whales’ diving mechanisms. This context therefore conveys the idea that the
assumption that baleen whale accessory spleens are latent may be incorrect.
Choice A is incorrect because it wouldn’t make sense to say that the role of
the accessory spleen is “replicable,” or capable of being reproduced. The text
indicates that the role of the accessory spleen seems to have no function, but
some researchers think it does have a role; the text doesn’t address whether
the role of the accessory spleen could or couldn’t be reproduced. ChoiceB
is incorrect because suggesting that the role of the accessory spleen is
“predetermined,” or decided in advance, wouldn’t make sense in context.
Although the researchers may agree that the role of the accessory spleen or
any other organ hasn’t been determined in advance, the text focuses on the idea
that the accessory spleen was thought to have been functionless but may in
fact serve an active role for baleen whales. Choice C is incorrect because it’s the
opposite of what the context of the text is conveying. The second sentence of the
text indicates that baleen whale accessory spleens may not be useless, not that
they aren’t “operative,” or functional.
SAT ANSWER EXPLANATIONS
n
READING AND WRITING: MODULE 1
5 SAT PRACTICE TEST #2 ANSWER EXPLANATIONS
QUESTION 6
Choice C is the best answer because it most logically completes the text’s
discussion of the factors that influence peoples’ decisions to move to a different
state. As used in this context, “overshadowed by” means to be surpassed by
or caused to seem less important than other factors affecting a move. The text
indicates that, according to a US tax policy expert, when people think about an
interstate move, state taxes have little effect on their decisions, while employment
opportunities, housing availability, and climate have a very strong effect. This
context suggests that people consider these other factors to be more important
than state taxes.
Choice A is incorrect because the text indicates that state taxes aren’t as
important a consideration as other factors when people are thinking of moving to
another state. The context doesn’t suggest that state taxes are “consistent with,”
or in agreement with these other factors. Choice B is incorrect because it wouldn’t
make sense in context to say that state taxes are “representative of,” or typical
of, other factors. Taxes aren’t an example of employment opportunities, housing
availability, and climate, which are the other factors listed in the text. Choice D is
incorrect because it wouldn’t make sense in context to say that state taxes are
“irrelevant to,” or unconnected or unimportant to other factors. State taxes are
irrelevant to peoples’ decisions, not to other factors. In other words, although the
text suggests that state taxes may be irrelevant to people considering a move
to another state, the other factors mentioned in the text, such as employment
opportunities, are unable to have an opinion about state taxes. Furthermore, the
text indicates that significant differences in state taxes have almost no effect on
peoples’ choices to move, but they aren’t completely unimportant.
QUESTION 7
Choice B is the best answer because it most logically completes the text’s
discussion of the author’s claim about the relationship between Neanderthals
and Homo sapiens. As used in this context, “tenuous” means lacking substance
or strength. The text states that the author’s claim isn’t convincing because it
doesn’t consider certain pieces of evidence—relevant recent discoveries. The
context conveys the idea that the author’s claim is weak.
Choice A is incorrect because the text doesn’t suggest that the author’s claim
is “disorienting,” or confusing; rather than indicating that the claim is hard
to grasp, the text focuses on the idea that it has a weakness that makes it
unconvincing. Choice C is incorrect because describing the claim as “nuanced,”
or subtle, wouldn’t make sense in context. The text emphasizes that the claim
is unconvincing because it didn’t consider certain key archaeological finds; it
doesn’t suggest that what’s in the claim seems subtle. Choice D is incorrect
because the text faults the claim for not considering certain key archaeological
finds; it doesn’t suggest that the author’s claim is “unoriginal,” or imitative and
lacking originality.
SAT ANSWER EXPLANATIONS
n
READING AND WRITING: MODULE 1
6 SAT PRACTICE TEST #2 ANSWER EXPLANATIONS
QUESTION 8
Choice D is the best answer because it accurately states the text’s main
purpose. The poem begins with the speaker urging a child to “go forth” with
her encouragement (“my heart’s desire”). The speaker goes on to suggest that
new experiences (“Great reaches, yet unknown”) lie ahead for the son that “life
is calling” him to seek out. Thus, the main purpose is to encourage a child to
embrace the experiences available to him in his life.
Choice A is incorrect because the speaker encourages the child to pursue new
experiences (“Great reaches”) without knowing exactly what those experiences
will be (“yet unknown”) or suggesting that they should match the speaker’s own
accomplishments. Choice B is incorrect because the speaker focuses on positive
possibilities for her son (“Great reaches, yet unknown”) and her enthusiastic
encouragement to embrace those possibilities (“life is calling you!”), while there
is no mention of raising a child or associated struggles. Choice C is incorrect
because the speaker frames the possibilities for her son in a positive light when
she says that “great reaches, yet unknown” are waiting for him, and this positive
outlook for the son is consistent throughout the text.
QUESTION 9
Choice D is the best answer because it best describes the function of the
underlined sentence in the text’s overall portrayal of how the women in Ohiyesa’s
tribe harvested maple syrup. The text states that the women used an axe to
strike the maple trees in order to find out which ones would produce sap. The
underlined sentence compares the trees to people, with the sap described as the
trees’ “life-blood.” Some of the trees are ready to give out their sap, while others
are unwilling to do so. Using personification, the sentence provides greater detail
about the aspect of the maple trees—their potential to give sap—that the women
are evaluating.
Choice A is incorrect because the personalities of the women are not discussed in
the text. Although the underlined sentence does mention “individual characters,”
this reference is not to the women in the text but rather to the maple trees, which
the sentence compares to people with individual character traits. Choice B is
incorrect because the underlined sentence focuses on the trees’ willingness or
refusal to yield sap, not on the beneficial relationship between the women and the
trees. Additionally, although the text does suggest that the women and their tribe
benefit from the maple trees since the trees allow the women to harvest syrup,
there is nothing in the text to suggest that the trees benefit from this relationship
in turn. Choice C is incorrect because the underlined sentence is comparing
maple trees to humans, not addressing the influence of the natural environment
on how the actual humans in the text, the women, behave.
SAT ANSWER EXPLANATIONS
n
READING AND WRITING: MODULE 1
7 SAT PRACTICE TEST #2 ANSWER EXPLANATIONS
QUESTION 10
Choice A is the best answer because based on Text 2, it represents how
Behrenfeld and colleagues would most likely respond to the “conventional
wisdom” discussed in Text 1. The conventional wisdom cited holds the opinion
that when there is species diversity within a phytoplankton population, “one
species should emerge after outcompeting the rest”—that is, after being so
successful in competing for resources that the other species vanish from the
population. However, Text 2 explains that according to Behrenfeld and colleagues,
phytoplankton are so small and spaced so far apart in the water that there is
“much less” direct competition for resources within phytoplankton populations
than scientists had previously thought.
Choice B is incorrect because Text 2 never discusses whether routine
replenishment of ocean nutrients affects competition between phytoplankton
species. Choice C is incorrect because the interspecies competition discussed
in both texts is specifically between phytoplankton species, and neither
text considers whether phytoplankton compete for resources with larger
nonphytoplankton species. Choice D is incorrect because according to Text 2,
Behrenfeld and colleagues argue that water density decreases, not increases,
competition between phytoplankton species.
QUESTION 11
Choice A is the best answer because it presents an explanation about a
helicopter that is directly supported by the text. The text states that Mars’s
atmosphere is much less dense than Earth’s, and as a result, the air on Mars
doesn’t provide the resistance required to support the blades of a helicopter
built for Earth and to keep the helicopter aloft. In other words, a helicopter
built for Earth can’t fly on Mars because of the differences in the two planets’
atmospheres.
Choice B is incorrect because instead of stating that the blades of helicopters
built for Earth are too large to work on Mars, the text indicates that the helicopter
built to fly on Mars actually has even longer blades than a helicopter built for
Earth. Choice C is incorrect because the text never addresses the role of gravity
on Mars or on Earth; instead, it focuses on atmospheric conditions. Choice D is
incorrect because the text doesn’t indicate that helicopters built for Earth are too
small to operate in the conditions on Mars. In fact, the text states that the size of
the helicopter built to fly on Mars is the same size as a helicopter built for Earth,
even though it has longer blades that rotate faster.
SAT ANSWER EXPLANATIONS
n
READING AND WRITING: MODULE 1
8 SAT PRACTICE TEST #2 ANSWER EXPLANATIONS
QUESTION 12
Choice A is the best answer because it best states the main idea of the text.
According to the text, jalis’ traditional role has been to maintain information about
families’ histories and significant events. The text goes on to say that although
technological changes have altered jalis’ role somewhat, jalis are still valued for
preserving the histories of their communities.
Choice B is incorrect because the text says nothing about jalis’ views of the
various tasks they perform. There is no information to support the idea that many
jalis prefer teaching to other tasks. Choice C is incorrect because the text doesn’t
describe jalis as being sources of entertainment. Rather, jalis are presented as
valued sources of knowledge. Additionally, the text gives no indication of how
long jalis have been serving their communities. Choice D is incorrect because the
main focus of the text is on jalis’ role and their continued value despite the effects
of technology, not on what technology can now do. Although the text indicates
that jalis’ role has changed as a result of technological changes, the text doesn’t
present any specific information about technology performing tasks that jalis
once performed.
QUESTION 13
Choice A is the best answer because it most accurately states the main idea
of the text. According to the text, Eugene Wigner hypothesized that a crystal
could exist that would be composed of electrons and have a honeycomb-like
shape. The text goes on to say that the existence of the Wigner crystal remained
unconfirmed until Feng Wang and colleagues were able to make an impression of
one using two semiconductors and an ultrathin sheet of graphene. Thus, the main
idea is that researchers have obtained the most definitive evidence to date of the
existence of the Wigner crystal.
Choice B is incorrect because the text focuses on one kind of crystal—the Wigner
crystal—and doesn’t discuss crystalline structures in general. And although the
text conveys that Wang and colleagues figured out a way to capture an image of
a Wigner crystal, it doesn’t address the idea of applying this approach to other
types of crystals. Choice C is incorrect because the text describes in general the
process Wang and colleagues followed to obtain an impression of the Wigner
crystal; it doesn’t address the relative importance of each component in that
process. Choice D is incorrect because the text doesn’t state that researchers
had a hard time getting an impression of the Wigner crystal because of its
honeycomb structure. Nothing in the text indicates why it took so long to prove
the existence of this crystal or take an impression of it.
SAT ANSWER EXPLANATIONS
n
READING AND WRITING: MODULE 1
9 SAT PRACTICE TEST #2 ANSWER EXPLANATIONS
QUESTION 14
Choice D is the best answer because it describes data from the graph that
support the researchers’ conclusion that there is a growing interest among CEOs
in connecting with more departments in their companies. The graph shows the
average number of individuals reporting directly to CEOs during three different
time periods: the individuals are divided into managers and department leaders.
The average number of department leaders directly reporting to their CEO during
the 1991–1995 period was slightly more than three, during the 1996–2001 period
it was four, and during the 2001–2008 period it was almost seven. Thus, the
average number of department leaders reporting directly to their CEO rose over
the three periods studied, which suggests that CEOs were connecting with more
departments.
Choice A is incorrect because the average number of managers and department
leaders reporting directly to their CEO rose for both categories between the
1991–1995 and 2001–2008 periods; thus, it isn’t true that the average numbers
didn’t fluctuate. Choice B is incorrect because the average number of managers
reporting directly to their CEO was highest in the 2001–2008 period, not in the
1996–2001 period. Choice C is incorrect. Although it correctly describes a feature
of the graph, the observation that more department leaders than managers are
reporting to CEOs does not by itself address the question of whether CEOs are
connecting with more departments over time—to address that question, one
needs to know whether the number of department leaders reporting to CEOs is
increasing over time.
SAT ANSWER EXPLANATIONS
n
READING AND WRITING: MODULE 1
10 SAT PRACTICE TEST #2 ANSWER EXPLANATIONS
QUESTION 15
Choice C is the best answer because it presents a finding that, if true, would
weaken Foster’s hypothesis that damage to eelgrass roots improves the health
of eelgrass meadows by boosting genetic diversity. The text indicates that sea
otters damage eelgrass roots but that eelgrass meadows near Vancouver Island,
where there’s a large otter population, are comparatively healthy. When Foster
and her colleagues compared the Vancouver Island eelgrass meadows to those
that don’t have established otter populations, the researchers found that the
Vancouver Island meadows are more genetically diverse than the other meadows
are. This finding led Foster to hypothesize that damage to the eelgrass roots
encourages eelgrass reproduction, thereby improving genetic diversity and the
health of the meadows. If, however, other meadows not included in the study are
less healthy the larger the local otter population is and the longer the otters have
been in residence, that would suggest that damage to the eelgrass roots, which
would be expected to increase with the size and residential duration of the otter
population, isn’t leading meadows to be healthier. Such a finding would therefore
weaken Foster’s hypothesis.
Choice A is incorrect because finding that small, recently introduced otter
populations are near other eelgrass meadows in the study wouldn’t weaken
Foster’s hypothesis. If otter populations were small and only recently established,
they wouldn’t be expected to have caused much damage to eelgrass roots, so
even if those eelgrass meadows were less healthy than the Vancouver Island
meadows, that wouldn’t undermine Foster’s hypothesis. In fact, it would be
consistent with Foster’s hypothesis since it would suggest that the greater
damage caused by larger, more established otter populations is associated with
healthier meadows. Choice B is incorrect because the existence of areas with
otters but without eelgrass meadows wouldn’t reveal anything about whether
the damage that otters cause to eelgrass roots ultimately benefits eelgrass
meadows. Choice D is incorrect because the health of plants other than eelgrass
would have no bearing on Foster’s hypothesis that damage to eelgrass roots
leads to greater genetic diversity and meadow health. It would be possible for
otters to have a negative effect on other plants while nevertheless improving the
health of eelgrass meadows by damaging eelgrass roots.
SAT ANSWER EXPLANATIONS
n
READING AND WRITING: MODULE 1
11 SAT PRACTICE TEST #2 ANSWER EXPLANATIONS
QUESTION 16
Choice B is the best answer because it most logically completes the text’s
discussion of Zelda Fitzgerald’s contributions to literature. The text begins by
saying that many scholars view Zelda mainly in terms of her marriage to F. Scott
Fitzgerald and “don’t recognize Zelda as a writer in her own right.” The text then
mentions a novel and “numerous short stories” that she wrote and that such
scholars tend to ignore. Therefore, those scholars who focus on Zelda only as
an inspiration for F. Scott’s writings risk misrepresenting the full range of Zelda’s
contributions to literature.
Choice A is incorrect. Although the text does mention that Zelda Fitzgerald
“likely influenced” her husband’s literary work, its focus is on Zelda’s own writing,
not on her husband’s writing or factors that might have influenced it. Choice C
is incorrect because the text does not discuss F. Scott and Zelda Fitzgerald’s
opinions of each other’s works. Choice D is incorrect. Although the text does
suggest that F. Scott Fitzgerald’s works were “likely influenced in part” by his
marriage to Zelda, it does not discuss autobiographical interpretations of the
works of either F. Scott or Zelda.
QUESTION 17
Choice B is the best answer because it presents the conclusion that most
logically completes the text’s discussion of the study by Versace and colleagues.
The text indicates that newborn animals of some species are attracted to
faces and to stimuli that resemble faces. These species, the text says, share
two characteristics: they’re social and they practice parental care, meaning
that parents care for their young. The text goes on to describe Versace and
colleagues’ experiment, which showed that although Testudo tortoises aren’t
social and don’t practice parental care, tortoise hatchlings were attracted to a
stimulus that resembles a face. Since Versace and colleagues have shown that
a species that isn’t social and doesn’t practice parental care nevertheless has
the innate characteristic of being attracted to face-like stimuli, it follows that
this characteristic shouldn’t be assumed to be an adaptation related to social
interaction or parental care.
Choice A is incorrect because the text indicates that the tortoise hatchlings were
attracted to the face-like stimuli (even though their species is solitary and doesn’t
practice parental care), not that they perceived the stimuli as threatening. ChoiceC
is incorrect because the phenomenon discussed in the text is an attraction to
faces and face-like stimuli on the part of newborn animals, which can’t show any
learned characteristics since they were just born. Additionally, the text tells us
that the tortoises Versace and colleagues studied aren’t social and don’t practice
parental care, so any findings about those tortoises wouldn’t be relevant to the
question of whether an attraction to faces in social species that practice parental
care is innate or learned. Choice D is incorrect because the text gives no indication
that adult tortoises were presented with face-like stimuli and, if adults were in fact
tested, no information about how they responded is provided. Since no information
about adult tortoises’ responses is provided, no conclusion comparing those
responses to the responses of newly hatched tortoises can be supported.
SAT ANSWER EXPLANATIONS
n
READING AND WRITING: MODULE 1
12 SAT PRACTICE TEST #2 ANSWER EXPLANATIONS
QUESTION 18
Choice A is the best answer because it most logically completes the text. The
text explains that the Cantares Mexicanos contains poems about the Aztec
Empire from before the Spanish invasion. Furthermore, it indicates that notes in
the collection attest that some of these poems predate the Spanish invasion,
while some customs depicted are likely Spanish in origin. The implication is that
some poems were composed before the invasion but the references to Spanish
customs could have come about only after the invasion, and thus that the
collection includes content that predates the invasion and also content from after
the invasion.
Choice B is incorrect because the text clearly indicates that the collection is in
Nahuatl, not Spanish, so the compilers’ unfamiliarity with Spanish is irrelevant to
whether the collection contains material composed after the Spanish invasion.
Choice C is incorrect because the text mentions only the Aztec Empire and Spain:
there is no information about the relationship of Aztec literature to any traditions
other than its own or Spain’s. Choice D is incorrect because the text states that
some of the poems make “inarguable references” to common Spanish customs,
which conflicts with the idea that these references can reasonably be attributed
to mere coincidence.
QUESTION 19
Choice A is the best answer because it presents the conclusion that most
logically follows from the text’s discussion of the study of capuchin monkeys’
cognitive abilities. The text explains that the study failed to distinguish between
outcomes for the tasks performed by the capuchin monkeys, such that simpler
tasks requiring less dexterity, or skill, were judged by the same criteria as
tasks that demanded more dexterity. Because the study didn’t account for this
discrepancy, the researchers might have assumed that observed differences in
performance were due to the abilities of the monkeys rather than the complexity
of the tasks. In other words, the results may suggest cognitive differences among
the monkeys even though such differences may not really exist.
Choice B is incorrect because the text focuses on the fact that the tasks assigned
to the capuchin monkeys in the study varied in difficulty and that the variety
wasn’t taken into consideration. The text doesn’t suggest that the capuchin
monkeys couldn’t perform certain tasks, just that some tasks were more difficult
to do. Choice C is incorrect because the text doesn’t suggest that the study’s
results are indicative of the abilities of capuchin monkeys but not of other monkey
species; in fact, the text suggests that the results may not even be an accurate
reflection of capuchin monkeys’ abilities. Choice D is incorrect because the text
doesn’t indicate that the researchers compared results for artificial tasks with
those for tasks encountered in the wild, although the tasks described in the text
sliding a panel and putting a straw in a bottle—are presumably artificial.
SAT ANSWER EXPLANATIONS
n
READING AND WRITING: MODULE 1
13 SAT PRACTICE TEST #2 ANSWER EXPLANATIONS
QUESTION 20
Choice A is the best answer. The convention being tested is finite and nonfinite
verb forms within a sentence. A main clause requires a finite verb to perform the
action of the subject (in this case, “embryos”), and this choice supplies the clause
with the finite present tense verb “enter” to indicate how the embryos achieve
diapause.
Choice B is incorrect because the nonfinite to-infinitive “to enter” doesn’t supply
the main clause with a finite verb. Choice C is incorrect because the nonfinite
participle “having entered” doesn’t supply the main clause with a finite verb.
Choice D is incorrect because the nonfinite participle “entering” doesn’t supply
the main clause with a finite verb.
QUESTION 21
Choice B is the best answer. The convention being tested is the use of verbs
to express tense. In this choice, the past perfect verb “had doubled” properly
indicates that the doubling of the organization’s initial membership occurred
during a specific period before the present (between the organization’s founding
in 1967 and the end of the 1990s).
Choice A is incorrect because the present perfect verb “has doubled” doesn’t
indicate that the organization’s doubling of its initial membership occurred
during a specific period in the past. Choice C is incorrect because the present
tense verb “doubles” doesn’t indicate that the organization’s doubling of its initial
membership occurred during a specific period in the past. Choice D is incorrect
because the future tense verb “will double” doesn’t indicate that the organization’s
doubling of its initial membership occurred during a specific period in the past.
QUESTION 22
Choice A is the best answer. The convention being tested is punctuation
use between sentences. In this choice, the period after “configurations” is
used correctly to mark the boundary between one sentence (“The intense
configurations”) and another (“TMAO…fish”). The supplementary phrase
(“ensuring…configurations”) modifies the main clause of the first sentence (“The
chemical…effect”), and “TMAO” is the subject of the second sentence.
Choice B is incorrect because it results in a run-on sentence. The sentences
(“The intense…configurations” and “TMAO…fish”) are fused without punctuation
and/or a conjunction. Choice C is incorrect because it results in a comma splice.
A comma can’t be used in this way to mark the boundary between sentences.
Choice D is incorrect. Without a comma preceding it, the conjunction “and” can’t
be used in this way to join sentences.
SAT ANSWER EXPLANATIONS
n
READING AND WRITING: MODULE 1
14 SAT PRACTICE TEST #2 ANSWER EXPLANATIONS
QUESTION 23
Choice C is the best answer. The convention being tested is the use of verbs to
express tense. In this choice, the present tense verb “experiences” is consistent
with the other present tense verbs (e.g., “connects” and “prepares”) used to
describe the events in Truong’s novels. Furthermore, it’s conventional to use the
present tense when discussing a literary work.
Choice A is incorrect because the past tense verb “experienced” isn’t consistent
with the other present tense verbs used to describe the events in Truong’s novels.
Choice B is incorrect because the past perfect tense verb “had experienced”
isn’t consistent with the other present tense verbs used to describe the events in
Truong’s novels. Choice D is incorrect because the future progressive tense verb
“will be experiencing” isn’t consistent with the other present tense verbs used to
describe the events in Truong’s novels.
QUESTION 24
Choice C is the best answer. The convention being tested is the use of plural and
possessive nouns. The singular possessive noun “screw’s” and the plural noun
“threads” correctly indicate that there is only one screw and it has multiple threads.
Choice A is incorrect because the context requires the plural noun “threads,”
not the singular possessive noun “thread’s.” Choice B is incorrect because the
context requires the singular possessive noun “screw’s,” not the plural possessive
noun “screws’.” Choice D is incorrect because the context requires the singular
possessive noun “screw’s” and the plural noun “threads,” not the plural noun
“screws” or the plural possessive noun “threads.’”
QUESTION 25
Choice C is the best answer. The convention being tested is punctuation
between a main clause and a supplementary noun phrase. This choice correctly
uses a comma to mark the boundary between the main clause (“scholar
materialism”) and the supplementary noun phrase (“an apt assessment”)
that describes Waid’s observation about how The House of Mirth depicts the
upper classes of New York society.
Choice A is incorrect because a semicolon and the conjunction “and” can’t
be used in this way to mark the boundary between a main clause and a
supplementary noun phrase. Choice B is incorrect. Joining the main clause
(“scholar…materialism”) and the following noun phrase with the conjunction “and”
results in a confusing and illogical sentence that suggests that the novel depicts
the upper classes of New York society as “an apt assessment,” which doesn’t
make sense in this context. Choice D is incorrect because it fails to mark the
boundary between the main clause and the supplementary noun phrase with
appropriate punctuation.
SAT ANSWER EXPLANATIONS
n
READING AND WRITING: MODULE 1
15 SAT PRACTICE TEST #2 ANSWER EXPLANATIONS
QUESTION 26
Choice D is the best answer. The convention being tested is the coordination
of main clauses within a sentence. The semicolon is correctly used to join the
first main clause (“To humans…prey”) and the second main clause (“rather
approach”). Further, the comma after the adverb “rather” is correctly used to
separate the adverb from the main clause (“the brightly…approach”) it modifies,
logically indicating that the information in this clause (how the spider’s behavior
appears to humans) is contrary to the information in the previous clause (how the
spider’s behavior does not appear to humans).
Choice A is incorrect because it results in a comma splice. Without a conjunction
following it, a comma can’t be used in this way to join two main clauses. ChoiceB
is incorrect because it results in a run-on sentence. The two main clauses
are fused without appropriate punctuation and/or a conjunction. Choice C is
incorrect. Placing the comma between the first main clause “To humans…prey”
and the adverb “rather” illogically indicates that the information in the first main
clause is contrary to what came before, which doesn’t make sense in this context.
QUESTION 27
Choice C is the best answer. The convention being tested is the use of plural
and possessive nouns. The singular possessive noun “playa’s” and the plural
possessive noun “rocks’” correctly indicate that the sediment is that of one playa
(the Racetrack Playa) and that there are multiple rocks that have mysteriously
migrated across the sediment.
Choice A is incorrect because the context requires the singular possessive noun
“playa’s” and the plural possessive noun “rocks’,” not the plural noun “playas” and
the singular possessive noun “rock’s.” Choice B is incorrect because the context
requires the plural possessive noun “rocks’,” not the plural noun “rocks.” Choice D
is incorrect because the context requires the singular possessive noun “playa’s,”
not the plural possessive noun “playas’.”
QUESTION 28
Choice C is the best answer. The convention being tested is punctuation between
a subject and a verb. When, as in this case, a subject (“her 2019 novel Gingerbread”)
is immediately followed by a verb (“offers”), no punctuation is needed.
Choice A is incorrect because no punctuation is needed between the subject and
the verb. Choice B is incorrect because no punctuation is needed between the
subject and the verb. Choice D is incorrect because no punctuation is needed
between the subject and the verb.
SAT ANSWER EXPLANATIONS
n
READING AND WRITING: MODULE 1
16 SAT PRACTICE TEST #2 ANSWER EXPLANATIONS
QUESTION 29
Choice B is the best answer. The sentence explains an advantage of microprobes,
noting that because microprobes weigh as little as 50 milligrams, they can explore
areas inaccessible to rovers.
Choice A is incorrect. The sentence indicates that rovers can land successfully
on Mars despite their weight; it doesn’t explain an advantage of microprobes.
Choice C is incorrect. While the sentence mentions that microprobes have
been proposed as an alternative to rovers, it doesn’t explain an advantage
of microprobes. Choice D is incorrect. The sentence emphasizes a similarity
between microprobes and rovers; it doesn’t explain an advantage of microprobes.
QUESTION 30
Choice C is the best answer. The sentence effectively introduces Paradise to
an audience unfamiliar with the novel and its author, describing Paradise as a
historical novel about colonial East Africa and its author as the winner of the 2021
Nobel Prize in Literature.
Choice A is incorrect. While the sentence introduces Abdulrazak Gurnah to an
audience unfamiliar with the author, it doesn’t effectively introduce Paradise.
Choice B is incorrect. While the sentence provides background information about
Paradise, it doesn’t effectively introduce the novel to an audience unfamiliar
with its author. Choice D is incorrect. While the sentence provides background
information about Paradise, it doesn’t effectively introduce the novel to an
audience unfamiliar with its author.
QUESTION 31
Choice C is the best answer. The sentence emphasizes the relative sizes of the
capital cities’ populations, noting that even though Hanoi has more people overall,
Ulaanbaatar accounts for a larger percentage of the people in its country.
Choice A is incorrect. While the sentence indicates the population size of each
capital, it fails to emphasize their sizes relative to each other or to their countries’
overall population sizes. Choice B is incorrect. While the sentence indicates the
population size of each capital, it fails to emphasize their sizes relative to each
other or to their countries’ overall population sizes. Choice D is incorrect. The
sentence emphasizes the population sizes of the two countries; it fails to mention
the capitals.
SAT ANSWER EXPLANATIONS
n
READING AND WRITING: MODULE 1
17 SAT PRACTICE TEST #2 ANSWER EXPLANATIONS
QUESTION 32
Choice D is the best answer. The sentence explains how the House of Wisdom
preserved the world’s knowledge, noting that the library collected, translated, and
printed writings from different countries.
Choice A is incorrect. While the sentence indicates that the House of Wisdom was
known for bringing together knowledge from around the world, it doesn’t explain
how the library preserved this knowledge. Choice B is incorrect. The sentence
makes a generalization about the scholars who were employed by the House
of Wisdom; it doesn’t explain how the library preserved the world’s knowledge.
Choice C is incorrect. The sentence identifies two authors whose writings were
preserved at the House of Wisdom; it doesn’t explain how the library preserved
the world’s knowledge.
QUESTION 33
Choice D is the best answer. The sentence uses information from the notes
to make a generalization about the kind of study Glickman, Brown, and Song
conducted. Specifically, the sentence indicates that the study was of a kind that
used statistical methods to address questions of authorship within the field of
music.
Choice A is incorrect because the sentence summarizes the methodology and
findings of a particular analysis of a single song; it doesn’t make a generalization
about the kind of study conducted. Choice B is incorrect because the sentence
mentions the data and conclusion of a particular analysis of a single song; it
doesn’t make a generalization about the kind of study conducted. Choice C
is incorrect because the sentence focuses on a specific conclusion from a
particular analysis of a single song; it doesn’t make a generalization about the kind
of study conducted.
18 SAT PRACTICE TEST #2 ANSWER EXPLANATIONS
SAT ANSWER EXPLANATIONS
n
READING AND WRITING: MODULE 2
Reading and Writing
Module 2
(33 questions)
QUESTION 1
Choice C is the best answer because it logically and precisely completes the
text’s discussion of The Mule Bone, a play that Zora Neale Hurston and Langston
Hughes wrote together. In this context, “collaboration” means working together
with someone to write a literary work. The text indicates that most writers prefer
to work alone and that working together destroyed the friendship between
Hurston and Hughes. This establishes that The Mule Bone is a relatively rare
example of collaboration in literature.
Choice A is incorrect because in this context, “characterization” would mean
a literary work’s portrayal of characters’ psychological experiences and
motivations, but the text doesn’t discuss characterization in The Mule Bone
specifically or in collaborative works more generally. Choice B is incorrect
because in this context, “interpretation” would mean the explanation of a literary
work’s meaning or significance, but the text doesn’t discuss how readers or critics
have interpreted The Mule Bone; instead, the text discusses how the play was
written collaboratively and how the writing process affected the two authors.
Choice D is incorrect because in this context, “commercialization” would mean
writing a literary work in such a way as to ensure its commercial appeal, but the
text never discusses commercial appeal as a factor in the writing of The Mule
Bone specifically or the writing of collaborative works more generally.
19 SAT PRACTICE TEST #2 ANSWER EXPLANATIONS
SAT ANSWER EXPLANATIONS
n
READING AND WRITING: MODULE 2
QUESTION 2
Choice B is the best answer because it most logically completes the text’s
discussion about recycling plastics. In this context, “inadequate” means not
satisfactory. The text indicates that the mechanical plastic-recycling process
affects the environment and causes “the loss of material quality.” The text
contrasts that with Chazovachii’s chemical plastic-recycling process, which is
cleaner and produces a desirable product. The text’s emphasis on the negative
aspects of mechanical recycling suggests that it is inadequate in terms of
environmental impact and the quality of the material the process yields.
Choice A is incorrect because in this context “resilient” would mean able to
withstand difficulty and the text does not characterize the plastic-recycling
process as having this quality or describe any difficulties that these processes
might need to overcome. Choice C is incorrect because in this context “dynamic”
would mean constantly changing. Although the text suggests that there have been
changes in the field of recycling, as is the case with the advent of Chazovachii’s
chemical recycling process, there is nothing to suggest that the mechanical
process itself has changed or is prone to change. Choice D is incorrect because
in this context “satisfactory” would mean acceptable but not perfect. The text
mentions only shortcomings of the mechanical process (environmental effects
and lower material quality), so the text more strongly supports a negative view of
this process and provides no evidence that it would be considered satisfactory.
QUESTION 3
Choice D is the best answer because it most logically completes the text’s
discussion of the economist’s claim about sales of personal electronic devices.
In this context, “invalidate” most nearly means nullify or make invalid. The text
indicates that interruptions in the supply of microchips for personal electronics
“have challenged” the economist’s claim that sales of personal electronics will
show strong growth in the coming months. The text goes on to clarify the effect
of the delays on the economist’s projection, stating that the delays are very likely
to extend the time frame over which the projected growth in sales will occur.
This context suggests that the delays are unlikely to invalidate the economist’s
projection entirely—the delays will probably alter the time frame of the projection,
not nullify it or make it invalid.
Choice A is incorrect because saying that the delays are unlikely to “dispute,” or
argue against, the economist’s projection wouldn’t make sense. Since the delays
are an inanimate circumstance, they couldn’t argue against a prediction about the
sales of personal electronics. Choice B is incorrect because saying that the delays
are unlikely to “withdraw,” or remove from consideration, the economist’s projection
wouldn’t make sense. Although the economist could withdraw her projection
because of the delays, the delays themselves couldn’t withdraw her projection
since they’re an inanimate circumstance and thus can’t choose to remove
something from consideration. Choice C is incorrect because there’s nothing in
the text to suggest that the delays will “underscore,” or emphasize, the economist’s
projection. Instead, the text suggests that the delays are likely to extend the time
frame of the economist’s projection but not to undermine the projection entirely.
20 SAT PRACTICE TEST #2 ANSWER EXPLANATIONS
SAT ANSWER EXPLANATIONS
n
READING AND WRITING: MODULE 2
QUESTION 4
Choice B is the best answer because it most logically completes the text’s
discussion of the art installation Anthem. In this context, “inventive” means
characterized by invention and creativity. The text explains that critics’ responses
to the installation involved praise for Tsang’s creative transformation of a space
into a dynamic exhibit with huge images and lots of sound. This context conveys
that the critics found the piece particularly creative.
Choice A is incorrect because the text indicates that critics praised the
installation for being dynamic and including huge images and lots of sound, and
it wouldn’t make sense to describe such an exhibit as “restrained,” or limited and
not extravagant or showy. Choice C is incorrect because it wouldn’t make sense
to say that critics found the installation “inexplicable,” or incapable of being
explained or interpreted, since the critics were able to explain their praise for
the installation’s transformation of a space with huge images and lots of sound.
Choice D is incorrect because the text focuses on the idea that critics praised
Tsang for creatively transforming a space into a dynamic exhibit, not that they
found the installation “mystifying,” or bewildering and hard to understand. Nothing
in the text suggests that the critics couldn’t understand the piece.
QUESTION 5
Choice C is the best answer because it most logically completes the text’s
discussion of the kinds of mammals alive during the Mesozoic era. As used in
this context, “diverse” means to have a significant amount of variety. The text
indicates that some scientists have suggested that Mesozoic mammals can’t
be characterized in a certain way, then contrasts the view put forward by those
scientists with Luo’s research, which shows that Mesozoic mammals “weren’t all
ground-dwelling insectivores” and instead were “various.” This context suggests
that some scientists have viewed Mesozoic mammals as being all alike, or not a
very diverse group.
Choice A is incorrect because it wouldn’t make sense to say that some scientists
have suggested that Mesozoic mammals weren’t very “predatory,” or that they
didn’t prey on other animals, since the text establishes a contrast between what
some scientists have suggested and Luo’s research showing that Mesozoic
mammals “weren’t all ground-dwelling insectivores.” This context suggests that
some scientists have regarded Mesozoic mammals as all being insectivores,
or animals that prey on insects, not that some scientists have suggested that
Mesozoic mammals didn’t prey on other animals. Choice B is incorrect because it
wouldn’t make sense to say that some scientists have suggested that Mesozoic
mammals weren’t very “obscure,” or concealed or not well known, since the
text establishes a contrast between what some scientists have suggested and
Luo’s research showing that Mesozoic mammals were a varied group. There’s no
contrast between saying that the mammals weren’t concealed or well known and
the mammals being varied. Choice D is incorrect because it wouldn’t make sense
to say that some scientists have suggested that Mesozoic mammals weren’t very
“localized,” or confined to a particular area, since the text establishes a contrast
21 SAT PRACTICE TEST #2 ANSWER EXPLANATIONS
SAT ANSWER EXPLANATIONS
n
READING AND WRITING: MODULE 2
between what some scientists have suggested and Luo’s research showing that
Mesozoic mammals were a varied group. There’s no contrast between saying
that the mammals weren’t localized and the mammals being varied. Although the
text mentions mammal fossils found in China, nothing in the discussion of Luo’s
research addresses the limits of Mesozoic mammal habitats.
QUESTION 6
Choice C is the best answer because it most accurately describes the overall
structure of the text. Throughout the text, the speaker characterizes nighttime as
if it were a person who wears clothing (“a garment” that is “velvet soft” and “violet
blue”) and a veil “over her face” and who moves her hands “slowly with their gem-
starred light” through her dark hair. Thus, the text is structured as an extended
comparison of night to a human being.
Choice A is incorrect because the text never mentions any particular location;
instead, it focuses on presenting a single description of night as a person with
certain clothing and features. Choice B is incorrect because the text doesn’t
make any reference to the sun or sunrise; instead, it focuses on presenting a
single image of night as a person with certain clothing and features. Choice D is
incorrect. Rather than describing how nighttime changes seasonally (or in any
other way), the text presents a single image of night as a person with certain
clothing and features.
QUESTION 7
Choice A is the best answer because it best describes how the underlined portion
functions in the text as a whole. The text says that the increased production
quotas of food processing companies during World War II enabled employees to
make better bargains in exchange for their labor. The underlined portion presents
an example of this increased bargaining power: employees requested more
favorable benefits, and employers complied because they were under pressure
to fulfill the demanding terms of their contracts. Thus, the underlined portion of
the text elaborates on a claim about labor relations in a particular industry (food
processing) made earlier in the text.
Choice B is incorrect because there is no indication in the text that the economic
factors that influenced food processing also influenced other parts of the
economy; thus, the bargaining described in the underlined portion of the text
cannot be called an example of a trend. Choice C is incorrect because the
underlined portion supports the historical narrative of labor activism in food
processing that is sketched in the text, instead of noting an exception to that
narrative. Choice D is incorrect because while the underlined portion does
discuss the demands that workers made in exchange for their labor, it does not
discuss the identities of the workers.
22 SAT PRACTICE TEST #2 ANSWER EXPLANATIONS
SAT ANSWER EXPLANATIONS
n
READING AND WRITING: MODULE 2
QUESTION 8
Choice D is the best answer because it accurately describes how the underlined
sentence functions in the text as a whole. The text establishes that John has a
strong imagination and then goes on to describe the St. John River near John’s
home in the Florida woods. The underlined sentence depicts John sending twigs
sailing down the river while he imagines them reaching “Jacksonville, the sea, the
wide world,” where he wishes he could follow. This suggests that John longs to
expand his life experiences beyond the Florida woods.
Choice A is incorrect because the second and third sentences of the text provide
an extended description of the riverbank where John likes to go, whereas the
underlined sentence describes what John does at that location. Choice B is
incorrect because the first sentence of the text suggests that John’s behavior
“was puzzling” to others around him, whereas the underlined sentence concerns
the content of John’s imaginings. Choice C is incorrect because the underlined
sentence elaborates on John’s imagination but doesn’t mention any other
children to whom John could be compared.
QUESTION 9
Choice B is the best answer because it presents a statement about Dorian that
is directly supported by the text. The narrator of the text says that when Dorian
sees his portrait, “his cheeks flushed for a moment with pleasure” and “a look
of joy came into his eyes.” The narrator goes on to say that Dorian looked at
the portrait “in wonder” and presents him as being so entranced by the portrait
that he doesn’t notice what Hallward is saying to him. These details support the
description of Dorian as being delighted by what he sees in the portrait.
Choice A is incorrect because Dorian isn’t depicted as interested in Hallward’s
opinion of the portrait; rather, he is so enraptured by the painting that he’s hardly
even aware of Hallward. Choice C is incorrect because the portrait of Dorian is
the only painting mentioned in the text. Although Dorian is depicted as being
delighted with this particular portrait, there’s no evidence in the text that he likes
portraits better than other kinds of paintings. Choice D is incorrect because
nothing in the text suggests that Dorian is uncertain about Hallward’s talent.
Instead, the text is focused on Dorian’s delight with the portrait.
QUESTION 10
Choice A is the best answer because it uses data from the graph to effectively
support Charles and Stephens’s claim about how level of information affects
voters. The graph shows the probability of voting for both high- and low-
information voters in seven categories of political orientation. Charles and
Stephens claim that “the more informed voters are about politics…the more likely
they are to vote.” This statement correctly asserts that the graph shows a higher
probability of voting for high-information voters than for low-information voters
at each of the seven political orientations. Thus, this statement accurately cites
data from the graph that support Charles and Stephens’s claim about how level of
information affects voters.
23 SAT PRACTICE TEST #2 ANSWER EXPLANATIONS
SAT ANSWER EXPLANATIONS
n
READING AND WRITING: MODULE 2
Choice B is incorrect. Although this statement is correct that the only probability
in the graph below 50% is for low-information voters categorized as independent
(orientation 4), the claim in question is about the relative likelihood that low- and
high-information voters will vote, and without some reference to high-information
voters, this statement cannot help support such a comparison. Choice C is
incorrect. Although this statement is correct that the highest probabilities of
voting for low-information voters are at the ends of the orientation scale (1 and 7),
the claim in question is about the relative likelihood that low- and high-information
voters will vote, and without some reference to high-information voters, this
statement cannot help support such a comparison. Choice D is incorrect because
the graph does not give any information about how many people are represented
in any of the categories, so this statement is not based on data from the graph.
Furthermore, even if we did have this information, the claim is about how level
of information affects voters’ probability of voting, not whether they’re likely to
strongly identify with a particular political party.
QUESTION 11
Choice B is the best answer because it describes data from the graph that
weaken the student’s conclusion about the reduction in the spider population in
the enclosure with lizards. The graph shows that the enclosure with lizards and the
enclosure without lizards each began with about 85 spiders, and that the number
of spiders in each enclosure fell over the 30 days of the study. The student’s claim
is that the reduction in spiders in the enclosure with lizards is “entirely attributable
to the presence of the lizards,” meaning that the spider population wouldn’t have
declined except for the presence of the lizards. This claim is weakened, however,
by the fact that the enclosure without lizards also saw a substantial reduction in
the number of spiders. Since the number of spiders fell in the enclosure without
lizards as well as in the enclosure with lizards, there must be some other factor
than just the presence of the lizards that contributed to the reduction in the spider
population.
Choice A is incorrect because the fact that the two enclosures started with the
same number of spiders is irrelevant to the claim that the reduction in spider
population by day 30 in the enclosure with lizards can be entirely attributed to
the lizards. Choice C is incorrect because the fact that the spider population in
the enclosure with lizards fell more between days 1 and 10 than in other periods
has nothing to do with the student’s claim that the reduction in spiders in that
enclosure by day 30 can be entirely attributed to the lizards. Choice D is incorrect.
Although it’s true that on day 30 the spider population was lower in the enclosure
with lizards than in the enclosure without lizards, this fact doesn’t weaken the
student’s claim that the reduction in the spider population in the enclosure with
lizards can be entirely attributed to the lizards. Indeed, the lower spider population
in the enclosure with lizards suggests that the lizards are contributing to the
reduction in the spider population, though the fact that the spider population
also fell substantially in the other enclosure means that the lizards aren’t the only
cause of the reduction.
24 SAT PRACTICE TEST #2 ANSWER EXPLANATIONS
SAT ANSWER EXPLANATIONS
n
READING AND WRITING: MODULE 2
QUESTION 12
Choice A is the best answer because it presents a finding that, if true, would
most strongly support the team’s conclusion that cattle were likely raised
closer to human settlements than sheep and goats were. The text explains that
Vaiglova, Liu, and their colleagues analyzed the chemical composition of sheep,
goat, and cattle bones from the Bronze Age in China in order to investigate the
animals’ domestication, or their adaptation from a wild state to a state in which
they existed in close connection with humans. According to the text, the team’s
analysis showed that sheep and goats of the era fed largely on wild plants,
whereas cattle fed on millet—importantly, a crop cultivated by humans. If analysis
of the animal bones shows that the cattle’s diet also consisted of wheat, another
crop cultivated by humans in China during the Bronze Age, the finding would
support the team’s conclusion by offering additional evidence that cattle during
this era fed on human-grown crops—and, by extension, that humans raised cattle
relatively close to the settlements where they grew these crops, leaving goats and
sheep to roam farther away in areas with wild vegetation, uncultivated by humans.
Choice B is incorrect because if it were true that sheep’s and goats’ diets
consisted of small portions of millet, which the text states was a crop cultivated
by humans, the finding would suggest that sheep and goats were raised relatively
close to human settlements, weakening the team’s conclusion that cattle were
likely raised closer to those settlements than sheep and goats were. ChoiceC
is incorrect because the finding that cattle generally require more food and
nutrients than do sheep and goats wouldn’t support the team’s conclusion that
cattle were likely raised closer to human settlements than sheep and goats were.
Nothing in the text suggests that cattle were incapable of obtaining sufficient
food and nutrients without access to human-grown crops. Hence, even if cattle’s
diets are found to have different requirements than the diets of sheep and goats,
the cattle could have met those requirements from food located far from human
settlements. Choice D is incorrect because if it were true that the diets of sheep,
goats, and cattle varied based on what the farmers in each Bronze Age settlement
could grow, the finding would weaken the team’s conclusion that cattle were
likely raised closer to human settlements than sheep and goats were, suggesting
instead that all three types of animals were raised close enough to human
settlements to feed on those settlements’ crops.
25 SAT PRACTICE TEST #2 ANSWER EXPLANATIONS
SAT ANSWER EXPLANATIONS
n
READING AND WRITING: MODULE 2
QUESTION 13
Choice C is the best answer because it presents the finding that, if true, would
best support Suarez, Pérez-Huerta, and Harrell’s claim about mosasaurs. The
text states that Suarez, Pérez-Huerta, and Harrell’s research on mosasaur tooth
enamel led them to conclude that mosasaurs were endothermic, which means
that they could live in waters at many different temperatures and still maintain
a stable body temperature. The researchers claim that endothermy enabled
mosasaurs to live in relatively cold waters near the poles. If several mosasaur
fossils have been found in areas that were near the poles during the period
when mosasaurs were alive and fossils of nonendothermic marine reptiles are
rare in such locations, that would support the researchers’ claim: it would show
that mosasaurs inhabited polar waters but nonendothermic marine mammals
tended not to, suggesting that endothermy may have been the characteristic that
enabled mosasaurs to include polar waters in their range.
Choice A is incorrect because finding that it’s easier to determine mosasaur body
temperatures from tooth enamel data than it is to determine nonendothermic
reptile body temperatures wouldn’t support the researchers’ claim. Whether one
research process is more difficult than another indicates nothing about the results
of those processes and therefore is irrelevant to the issue of where mosasaurs
lived and what enabled them to live in those locations. Choice B is incorrect
because finding roughly equal numbers of mosasaur and nonendothermic marine
reptile fossils in areas that were near the poles in the Late Cretaceous would
suggest that endothermy didn’t give mosasaurs any particular advantage when
it came to expanding their range to include relatively cold polar waters, thereby
weakening the researchers’ claim rather than supporting it. Choice D is incorrect
because finding that the temperature of seawater in the Late Cretaceous was
warmer than seawater today wouldn’t weaken the researchers’ claim. Seawater in
the Late Cretaceous could have been warmer than seawater today but still cold
enough for endothermy to be advantageous to mosasaurs, so this finding wouldn’t
provide enough information to either support or weaken the researchers’ claim.
26 SAT PRACTICE TEST #2 ANSWER EXPLANATIONS
SAT ANSWER EXPLANATIONS
n
READING AND WRITING: MODULE 2
QUESTION 14
Choice D is the best answer because it presents a finding that, if true, would most
directly support the researchers’ hypothesis about the connection between the
dusky shark population decline and the eastern oyster population decline. The
text indicates that although dusky sharks don’t usually eat eastern oysters, they
do consume cownose rays, which are the main predators of eastern oysters.
An increase in the abundance of cownose rays in the region in response to a
decline in the abundance of dusky sharks would directly support the researchers’
hypothesis: a higher number of cownose rays would consume more eastern
oysters, driving down the oyster population.
Choice A is incorrect because a finding that there’s an association between
a decline in the regional abundance of some of dusky sharks’ prey and the
regional abundance of dusky sharks wouldn’t directly support the researchers’
hypothesis that a decline in dusky sharks has led to a decline in eastern oysters
in the region. Although such a finding might help explain why shark abundance
has declined, it would reveal nothing about whether the shark decline is related
to the oyster decline. Choice B is incorrect because a finding that eastern oyster
abundance tends to be greater when dusky sharks and cownose rays are present
than when only dusky sharks are present wouldn’t support the researchers’
hypothesis that a decline in dusky sharks has led to a decline in eastern oysters
in the region. The text indicates that the sharks prey on the rays, which are the
main predators of the oysters; if oyster abundance is found to be greater when
rays are present than when rays are absent, that would suggest that rays aren’t
keeping oyster abundance down, and thus that a decline in rays’ predators, which
would be expected to lead to an increase in the abundance of rays, wouldn’t bring
about a decline in oyster abundance as the researchers hypothesize. Choice C
is incorrect because a finding that consumption of eastern oysters by cownose
rays increased substantially before dusky sharks declined in regional abundance
wouldn’t support the researchers’ hypothesis that the decline in dusky sharks has
led to a decline in eastern oysters in the region. Such a finding would suggest that
some factor other than shark abundance led to an increase in rays’ consumption
of oysters and thus to a decrease in oyster abundance, thereby weakening the
researchers’ hypothesis.
27 SAT PRACTICE TEST #2 ANSWER EXPLANATIONS
SAT ANSWER EXPLANATIONS
n
READING AND WRITING: MODULE 2
QUESTION 15
Choice C is the best answer because it presents a finding that, if true, would
weaken the claim made by people who favor the traditional view of voter behavior.
According to the text, people who favor that view believe that voting in an
election doesn’t change a voter’s attitude toward the candidates in that election.
If Washington and Mullainathan found that two years after an election, attitudes
toward the winning candidate were significantly more polarized among subjects
who had voted than among subjects who had been too young to vote, that would
suggest that the act of voting did have an effect on the voters’ attitudes toward
the candidates, which would undermine the claim that voting doesn’t change
voters’ attitudes.
Choice A is incorrect because a finding about links between subjects’ attitudes
and general political orientation, regardless of age and ability to vote, wouldn’t
address the presence or absence of changes in attitudes among those subjects
who did actually vote. Therefore, the finding wouldn’t have any bearing on the
claim that voting in an election doesn’t change a voter’s attitude toward the
candidates in that election. Choice B is incorrect because a finding that positive
attitudes toward a winning candidate significantly increased in the two years
after the election among subjects who had been too young to vote would involve
only people who didn’t vote; therefore, the finding wouldn’t have any bearing
on the claim that when people do vote, the act of voting doesn’t change their
attitudes toward the candidates. Choice D is incorrect because the finding that
subjects in both groups were more likely to have negative attitudes than positive
attitudes toward the winning candidate two years after an election would reflect
all subjects’ attitudes at one particular time whether they voted or not, rather than
the presence or absence of a change in voters’ attitudes after voting. Therefore,
the finding would neither weaken nor strengthen the claim that voting in an
election doesn’t change a voter’s attitude toward the candidates.
28 SAT PRACTICE TEST #2 ANSWER EXPLANATIONS
SAT ANSWER EXPLANATIONS
n
READING AND WRITING: MODULE 2
QUESTION 16
Choice B is the best answer because it describes data from the graph that
support Taylor and colleagues’ conclusion that spray coating holds promise
for improving the power conversion efficiency of ETLs in perovskite solar cells.
The text explains that perovskite solar cells’ efficiency at converting light into
electricity is diminished by their electron transport layer (ETL), which is applied
through spin coating, but that Taylor’s team devised a new spray coating method
for applying the ETL that improves its power conversion efficiency. The graph
displays data on the power conversion efficiency of solar cells in tests conducted
by Taylor’s team, with bars for both the highest- and lowest-performing ETLs in
two data categories: spray coating and spin coating. According to the graph, the
lowest-performing ETL applied through spray coating had a power conversion
efficiency of between 14% and 16%, while the highest-performing ETL applied
through spin coating had a power conversion efficiency of less than 14%. These
data confirm that ETLs applied through novel spray coating are more efficient
than those applied though traditional spin coating. Thus, the data support Taylor
and colleagues’ conclusion about spray coating’s potential value.
Choice A is incorrect. Although this claim correctly describes the data in the
graph by stating that both the lowest-performing ETL applied through spin
coating and the lowest-performing ETL applied through spray coating had a
power conversion efficiency greater than 10%, this relationship in the data
doesn’t support or relate to Taylor and colleagues’ conclusion that spray coating
promises greater efficiency for solar cells than traditional spin coating does.
Choice C is incorrect. This claim does address the greater power conversion
efficiency of the highest-performing ETL applied through spray coating,
compared with the highest-performing ETL applied through spin coating.
However, it also incorrectly cites the value for the efficiency of the highest-
performing ETL applied through spray coating as approximately 13%, instead of
a value between 14% and 16%, and the value for the efficiency of the highest-
performing ETL applied through spin coating as approximately 11%, instead
of a value between 12% and 14%, as shown in the graph. Choice D is incorrect
because Taylor and colleagues’ conclusion is based on the difference in the
power conversion efficiency of ETLs applied through spray coating and that of
ETLs applied through spin coating, not on the difference between the highest-
and lowest-performing ETLs applied through just spray coating.
29 SAT PRACTICE TEST #2 ANSWER EXPLANATIONS
SAT ANSWER EXPLANATIONS
n
READING AND WRITING: MODULE 2
QUESTION 17
Choice B is the best answer because it presents a finding that, if true, would most
directly support the arts journalist’s claim about Enwezor’s work as a curator
and art historian. In the text, the arts journalist asserts that Enwezor wished not
just to focus on modern African artists but also to show “how their work fits into
the larger context of global modern art and art history,” or how their work relates
to artistic developments and work by other artists elsewhere in the world. The
description of Postwar: Art Between the Pacific and the Atlantic, 1945–1965
indicates that Enwezor and Siegel’s exhibition brought works by African artists
together with works by artists from other countries, thus supporting the arts
journalist’s claim that Enwezor sought to show works by African artists in a
context of global modern art and art history.
Choice A is incorrect because it describes a retrospective that wouldn’t support
the arts journalist’s claim that Enwezor wanted to show how works by modern
African artists fit into the larger context of global modern art and art history.
The description of El Anatsui: Triumphant Scale indicates that the retrospective
focused only on the work of a single African artist, El Anatsui. The description
doesn’t suggest that the exhibition showed how El Anatsui’s works fit into a
global artistic context. Choice C is incorrect because it describes an exhibition
that wouldn’t support the arts journalist’s claim that Enwezor wanted to show
how works by modern African artists relate to the larger context of global
modern art and art history. The description of The Short Century: Independence
and Liberation Movements in Africa, 1945–1994 indicates that the exhibition
showed how African artists were influenced by movements for independence
from European colonial powers following the Second World War. Although this
suggests that Enwezor intended the exhibition to place works by African artists
in a political context, it doesn’t indicate that the works were placed in a global
artistic context. Choice D is incorrect because it describes an exhibition that
wouldn’t support the arts journalist’s claim that Enwezor wanted to show how
works by modern African artists relate to the larger context of global modern art
and art history. The description of In/sight: African Photographers, 1940 to the
Present indicates that the exhibition was intended to reveal the broad range of
approaches taken by African photographers, not that the exhibition showed how
photography by African artists fits into a global artistic context.
QUESTION 18
Choice B is the best answer. The convention being tested is finite and nonfinite
verb forms within a sentence. A main clause requires a finite verb to perform
the action of the subject (in this case, “people in the Americas”), and this choice
supplies the finite past perfect tense verb “have used” to indicate what people in
the Americas used the gourd for.
Choice A is incorrect because the nonfinite to-infinitive “to use” doesn’t supply
the main clause with a finite verb. Choice C is incorrect because the nonfinite
participle “having used” doesn’t supply the main clause with a finite verb.
ChoiceD is incorrect because the nonfinite participle “using” doesn’t supply the
main clause with a finite verb.
30 SAT PRACTICE TEST #2 ANSWER EXPLANATIONS
SAT ANSWER EXPLANATIONS
n
READING AND WRITING: MODULE 2
QUESTION 19
Choice C is the best answer. The convention being tested is punctuation between
a subordinate clause and a main clause. This choice correctly uses a comma to
mark the boundary between the subordinate clause (“While…lifelike”) and the
main clause (“others look to the past”).
Choice A is incorrect because it results in an incomplete sentence with no main
clause. Choice B is incorrect because it fails to mark the boundary between
the subordinate clause (“While…lifelike”) and the main clause (“others…past”).
ChoiceD is incorrect because it results in an incomplete sentence with no main
clause.
QUESTION 20
Choice A is the best answer. The convention being tested is the use of verbs to
express tense. In this choice, the simple past tense verb “suggested” properly
indicates that Zufall offered her suggestion for the product’s name in the past.
This verb tense is consistent with the previous sentence’s use of a simple past
tense verb (“advised”) to describe Zufall’s advice to McVicker in the 1950s.
Choice B is incorrect because the present tense verb “suggests” doesn’t
indicate that Zufall offered her suggestion in the past. Choice C is incorrect
because the past perfect verb “had suggested” isn’t consistent with the previous
sentence’s use of the simple past tense verb “advised” to describe Zufall’s advice
to McVicker. Choice D is incorrect because the past progressive verb “was
suggesting” isn’t consistent with the previous sentence’s use of the simple past
tense verb “advised” to describe Zufall’s advice to McVicker.
QUESTION 21
Choice D is the best answer. The convention being tested is punctuation use
between two supplementary phrases following the coordinate clause (“but she
mycology”). This choice correctly uses a comma to mark the boundary between
the supplementary noun phrase (“the study of fungi”) that defines the term
“mycology” and the supplementary participial phrase (“producing...London”) that
provides additional information about the extent to which Potter dedicated herself
to mycology.
Choice A is incorrect because a semicolon can’t be used in this way to join two
supplementary phrases following a coordinate clause. Choice B is incorrect
because it results in a rhetorically unacceptable sentence fragment beginning
with “producing.” Choice C is incorrect. The lack of punctuation results in a
sentence that illogically suggests that the study of fungi is producing more than
350 paintings.
31 SAT PRACTICE TEST #2 ANSWER EXPLANATIONS
SAT ANSWER EXPLANATIONS
n
READING AND WRITING: MODULE 2
QUESTION 22
Choice A is the best answer. The convention being tested is subject-modifier
placement. This choice makes the noun phrase “many critics” the subject of the
sentence and places it immediately after the modifying phrase “in assessing…
Kurosawa.” In doing so, this choice clearly establishes that it is the critics—and
not another noun in the sentence—who assess Kurosawa’s films.
Choice B is incorrect because it results in a dangling modifier. The placement of
the noun phrase “Kurosawa’s…sources” immediately after the modifying phrase
illogically suggests that his use of Western literary sources is what assesses
Kurosawa’s films. Choice C is incorrect because it results in a dangling modifier.
The placement of the function word “there” immediately after the modifying
phrase illogically suggests that “there” is what assesses Kurosawa’s films.
ChoiceD is incorrect because it results in a dangling modifier. The placement
of the noun phrase “the focus…critics” immediately after the modifying phrase
illogically suggests that the critics’ focus is what assesses Kurosawa’s films.
QUESTION 23
Choice C is the best answer. The convention being tested is the punctuation of
items in a complex series. It’s conventional to use a semicolon to separate items
in a complex series with internal punctuation, and in this choice, the semicolon
after “2009” is conventionally used to separate the first item (“the world’s…2009”)
and the second item (“an online…2010”) in the series of things that Hinson helped
create. Further, the comma after “Basic” correctly pairs with the comma after
“app,” and the comma after “network” correctly pairs with the comma after “TV
to set off the supplemental elements (“Chickasaw Basic” and “Chickasaw TV”)
that provide the names of the app and the TV network, respectively. Altogether,
the punctuation in this choice results in a sentence that clearly indicates that
Hinson helped make a language app in 2009, an online TV network in 2010, and a
language course in 2015.
Choice A is incorrect because it fails to punctuate the complex series in a way that
makes clear that Hinson helped make a language app in 2009, an online TV network
in 2010, and a language course in 2015. Choice B is incorrect because it fails to
punctuate the complex series in a way that makes clear that Hinson helped make
a language app in 2009, an online TV network in 2010, and a language course in
2015. Choice D is incorrect because the comma after “2009” doesn’t match the
semicolon used to separate the second and third items in the complex series.
32 SAT PRACTICE TEST #2 ANSWER EXPLANATIONS
SAT ANSWER EXPLANATIONS
n
READING AND WRITING: MODULE 2
QUESTION 24
Choice C is the best answer. The convention being tested is the punctuation of
a supplementary element within a sentence. The dash after “Springs” pairs with
the dash after “earth” to separate the supplementary element “in this case, the
porous rocks of the hills around Hot Springs” from the rest of the sentence.
Choice A is incorrect because it fails to use appropriate punctuation to
separate the supplementary element from the rest of the sentence. Choice B
is incorrect because a colon can’t be paired with a dash in this way to separate
the supplementary element from the rest of the sentence. Choice D is incorrect
because a comma can’t be paired with a dash in this way to separate the
supplementary element from the rest of the sentence.
QUESTION 25
Choice A is the best answer. The convention being tested is the use of a colon
within a sentence. In this choice, the colon is used in a conventional way to
introduce the following description of how the number of jams available varied.
Choice B is incorrect because it creates a comma splice. A comma can’t be
used in this way to join two main clauses (“the numbervaried” and “some…six”).
Choice C is incorrect because it results in an illogical and confusing sentence.
Using the conjunction “while” to join the main clause (“the numbervaried”) with
the following clause’s description of the number of jams available suggests
that the variation in the number of jams is in contrast to some shoppers having
twenty-four options. Choice D is incorrect because it results in an illogical and
confusing sentence. Using “while” in this way suggests that the number of jams
available varied during the time in which some shoppers had twenty-four options
and others had six. The sentence makes clear, however, that what follows “varied”
is a description of the variation, not a separate, simultaneous occurrence.
QUESTION 26
Choice B is the best answer. The convention being tested is the punctuation of
items in a complex series (a series including internal punctuation). In this choice,
the semicolon after “Lagos” is conventionally used to separate the first item (“The
Joys…Lagos”) and the second item (“A Kind…Nigeria”) in the series. Further, the
comma after “Marriage” correctly separates the title “A Kind of Marriage” from the
supplementary phrase (“a television…Nigeria”) that describes it.
Choice A is incorrect because the comma after “Lagos” doesn’t match the
semicolon used later in the series to separate the second item (“A Kind…Nigeria”)
from the third item (“and…autobiography”). Choice C is incorrect because the
comma after “Lagos” doesn’t match the semicolon used later in the series
to separate the second item (“A Kind…Nigeria”) from the third item (“and…
autobiography”). Additionally, a colon can’t be used in this way to separate the title
A Kind of Marriage” from the supplementary phrase (“a television… Nigeria”) that
describes it. Choice D is incorrect because it fails to use appropriate punctuation
to separate the title “A Kind of Marriage” from the supplementary phrase (“a
television…Nigeria”) that describes it.
33 SAT PRACTICE TEST #2 ANSWER EXPLANATIONS
SAT ANSWER EXPLANATIONS
n
READING AND WRITING: MODULE 2
QUESTION 27
Choice A is the best answer. “Meanwhile” logically signals that the action
described in this sentence (Obinze’s move to London to pursue a career) is
simultaneous with the action described in the previous sentence (Ifemelu’s
move to the United States). The first sentence establishes that the actions take
place around the same time, referring to the characters’ “divergent experiences”
following high school.
Choice B is incorrect because “nevertheless” illogically signals that the
information in this sentence about Obinze’s move to London is true despite the
previous information about Ifemelu’s move to the United States. Instead, as the
first sentence establishes, Obinze’s move and Ifemelu’s move are related, parallel
experiences that occur around the same time. Choice C is incorrect because
“secondly” illogically signals that the information in this sentence is a second
point or reason separate from the previous information about Ifemelu’s move to
the United States. Instead, as the first sentence establishes, Obinze’s move and
Ifemelu’s move are related, parallel experiences that occur around the same time.
Choice D is incorrect because “in fact” illogically signals that the information
in this sentence emphasizes, modifies, or contradicts the previous information
about Ifemelu’s move to the United States. Instead, as the first sentence
establishes, Obinze’s move and Ifemelu’s move are related, parallel experiences
that occur around the same time.
QUESTION 28
Choice B is the best answer. “For example” logically signals that the information
in this sentence—that tadpole shrimp embryos can pause development during
extended periods of drought—exemplifies the previous sentence’s claim that
organisms have evolved surprising adaptations to survive in adverse conditions.
Choice A is incorrect because “in contrast” illogically signals that the information
in this sentence contrasts with the claim about organisms in the previous
sentence. Instead, it exemplifies this claim. Choice C is incorrect because
“meanwhile” illogically signals that the information in this sentence is separate
from (while occurring simultaneously with) the claim about organisms in the
previous sentence. Instead, it exemplifies this claim. Choice D is incorrect
because “consequently” illogically signals that the information in this sentence is
a consequence, or result, of the claim about organisms in the previous sentence.
Instead, it exemplifies this claim.
34 SAT PRACTICE TEST #2 ANSWER EXPLANATIONS
SAT ANSWER EXPLANATIONS
n
READING AND WRITING: MODULE 2
QUESTION 29
Choice D is the best answer. “In addition” logically signals that the information
in this sentence—that the Twentieth Amendment requires newly elected US
senators and representatives to be sworn in on January 3 —is separate from and
additional to the amendment’s mandate concerning presidential inaugurations.
Choice A is incorrect because “instead” illogically signals that the information
in the sentence presents an alternative to or substitute for the Twentieth
Amendment’s mandate concerning presidential inaugurations. Rather, the
sentence presents a separate requirement in addition to that one. Choice B
is incorrect because “for instance” illogically signals that the information in
the sentence exemplifies the Twentieth Amendment’s mandate concerning
presidential inaugurations. Instead, the sentence presents a separate requirement
in addition to that one. Choice C is incorrect because “specifically” illogically
signals that the sentence provides specific, precise details elaborating on the
Twentieth Amendment’s mandate concerning presidential inaugurations. Instead,
the sentence presents a separate requirement in addition to that one.
QUESTION 30
Choice D is the best answer. “Similarly” logically signals that the information in
the sentence—that Dove situates Beulah’s life in the context of the US Civil Rights
Movement—is similar to the previous information about Thomas and the Great
Migration. Both sentences support the first sentence’s claim that Dove portrays
her characters in the context of broader historical narratives.
Choice A is incorrect because “specifically” illogically signals that the
information about Beulah in this sentence provides specific details elaborating
on the previous information about Thomas. Instead, it’s similar to the previous
information about Thomas. Choice B is incorrect because “thus” illogically signals
that the information about Beulah in this sentence is a result or consequence
of the previous information about Thomas. Instead, it’s similar to the previous
information about Thomas. Choice C is incorrect because “regardless” illogically
signals that the information about Beulah in this sentence is true despite
the previous information about Thomas. Instead, it’s similar to the previous
information about Thomas.
QUESTION 31
Choice A is the best answer. The sentence emphasizes the distance covered
by the Philadelphia and Lancaster Turnpike, noting that the turnpike, which
connected the two Pennsylvania cities in its name, was sixty-two miles long.
Choice B is incorrect. The sentence emphasizes the significance of the turnpike;
it doesn’t emphasize the distance that the turnpike covered. Choice C is incorrect.
While the sentence mentions that the turnpike connected two Pennsylvania cities,
it doesn’t emphasize the specific distance covered by the turnpike. ChoiceD
is incorrect. The sentence emphasizes when the turnpike was built; it doesn’t
emphasize the distance that the turnpike covered.
35 SAT PRACTICE TEST #2 ANSWER EXPLANATIONS
SAT ANSWER EXPLANATIONS
n
READING AND WRITING: MODULE 2
QUESTION 32
Choice C is the best answer. The sentence emphasizes the aim, or goal, of the
research study, noting what Terada set out to do: determine whether some of the
Moon’s oxygen was coming from Earth.
Choice A is incorrect. The sentence focuses on how the Kaguya satellite collected
data; it doesn’t emphasize the aim of the research study. Choice B is incorrect.
While the sentence mentions what Terada was curious about before conducting
the research study, it doesn’t emphasize his study’s aim. Choice D is incorrect.
The sentence presents the research study’s conclusion; it doesn’t emphasize the
study’s aim.
QUESTION 33
Choice B is the best answer. The sentence presents both the study and its
methodology (that is, the researcher’s approach to the problem), explaining that
Yuan used computer simulations to study the effect of the mother duck’s wake on
the ducklings’ energy expenditure.
Choice A is incorrect. The sentence describes the findings of Yuan’s study; it
doesn’t present the study and its methodology. Choice C is incorrect. While the
sentence provides general information about Yuan’s study, it doesn’t present the
study’s methodology. Choice D is incorrect. The sentence describes the findings
of Yuan’s study; it doesn’t present the study and its methodology.
SAT ANSWER EXPLANATIONS
n
MATH: MODULE 1
36 SAT PRACTICE TEST #2 ANSWER EXPLANATIONS
Math
Module 1
(27 questions)
QUESTION 1
Choice C is correct. For the given line graph, the percent of cars for sale at a used
car lot on a given day is represented on the vertical axis. The percent of cars for
sale is the smallest when the height of the line graph is the lowest. The lowest
height of the line graph occurs for cars with a model year of
2014
.
Choice A is incorrect and may result from conceptual errors. Choice B is incorrect
and may result from conceptual errors. Choice D is incorrect and may result from
conceptual errors.
QUESTION 2
Choice C is correct. It’s given that
29
out of every
100
beads that the machine
produces have a defect. It follows that if the machine produces
k
beads, then the
number of beads that have a defect is
29
100
k
, for some constant
k
. If a bead
produced by the machine will be selected at random, the probability of selecting
a bead that has a defect is given by the number of beads with a defect,
29
100
k
,
divided by the number of beads produced by the machine,
k
. Therefore, the
probability of selecting a bead that has a defect is
29
100
k
k
, or
29
100
.
Choice A is incorrect and may result from conceptual or computational errors.
Choice B is incorrect and may result from conceptual or computational errors.
Choice D is incorrect and may result from conceptual or computational errors.
QUESTION 3
Choice D is correct. It’s given that line
m
is parallel to line
n
, and line
t
intersects
both lines. It follows that line
t
is a transversal. When two lines are parallel and
intersected by a transversal, exterior angles on the same side of the transversal
SAT ANSWER EXPLANATIONS
n
MATH: MODULE 1
37 SAT PRACTICE TEST #2 ANSWER EXPLANATIONS
are supplementary. Thus,
x +33 =180
. Subtracting
33
from both sides of this
equation yields
x =147
. Therefore, the value of
x
is
147
.
Choice A is incorrect and may result from conceptual or calculation errors.
ChoiceB is incorrect and may result from conceptual or calculation errors.
ChoiceC is incorrect and may result from conceptual or calculation errors.
QUESTION 4
Choice D is correct. The y-intercept of a graph in the xy-plane is the point at
which the graph crosses the y-axis. The graph shown crosses the y-axis at the
point
(
0, 8
)
. Therefore, the y-intercept of the graph shown is
(
0, 8
)
.
Choice A is incorrect and may result from conceptual or calculation errors.
ChoiceB is incorrect and may result from conceptual or calculation errors.
ChoiceC is incorrect and may result from conceptual or calculation errors.
QUESTION 5
Choice C is correct. It’s given that
f
(
x
)
is the total cost, in dollars, to lease a
car from this dealership with a monthly payment of
x
dollars. Therefore, the
total cost, in dollars, to lease the car when the monthly payment is
$400
is
represented by the value of
f
(
x
)
when
x = 400
. Substituting
400
for
x
in the
equation
( )
fx 36x=+1,000
yields
f
(
400
)
=36
(
400
)
+1,000
, or
f
(
400
)
=15,400
.
Thus, when the monthly payment is
$400
, the total cost to lease a car is
$15,400
.
Choice A is incorrect and may result from conceptual or calculation errors.
ChoiceB is incorrect and may result from conceptual or calculation errors.
ChoiceD is incorrect and may result from conceptual or calculation errors.
QUESTION 6
The correct answer is
180
. The perimeter of a polygon is equal to the sum of the
lengths of the sides of the polygon. It’s given that each side of the square has
a length of
45
. Since a square is a polygon with
4
sides, the perimeter of this
square is
45+45+45+45
, or
180
.
QUESTION 7
The correct answer is
5
. Multiplying both sides of the given equation by
x +6
results in
55=x
(
x+6
)
. Applying the distributive property of multiplication to
the right-hand side of this equation results in
2
55 6xx=+
. Subtracting
55
from both sides of this equation results in
2
0 6 55xx=+-
. The right-hand side
of this equation can be rewritten by factoring. The two values that multiply to
-55
and add to
6
are
11
and
-5
. It follows that the equation
2
0 6 55xx=+-
can be rewritten as
0
( )( )
11 5xx=+ -
. Setting each factor equal to
0
yields
two equations:
x +11=0
and
50x -=
. Subtracting
11
from both sides of the
equation
x +11=0
results in
x =-11
. Adding
5
to both sides of the equation
50x -=
results in
x = 5
. Therefore, the positive solution to the given equation
is
5
.
SAT ANSWER EXPLANATIONS
n
MATH: MODULE 1
38 SAT PRACTICE TEST #2 ANSWER EXPLANATIONS
QUESTION 8
Choice A is correct. If the object travels
108
centimeters at a speed of
12
centimeters per second, the time of travel can be determined by dividing
the total distance by the speed. This results in
108 centimeters
12 centimeters/second
, which is
9
seconds.
Choice B is incorrect and may result from conceptual or calculation errors.
ChoiceC is incorrect and may result from conceptual or calculation errors.
ChoiceD is incorrect and may result from conceptual or calculation errors.
QUESTION 9
Choice B is correct. The mean of a data set is the sum of the values in the data set
divided by the number of values in the data set. It follows that the mean of data
set X is
5991
4
+++
3
, or
9
, and the mean of data set Y is
5 9 9 13 27
5
+++ +
, or
12.6
. Since
9
is less than
12.6
, the mean of data set X is less than the mean of data set Y.
Alternate approach: Data set Y consists of the
4
values in data set X and one
additional value,
27
. Since the additional value,
27
, is larger than any value in data
set X, the mean of data set X is less than the mean of data set Y.
Choice A is incorrect and may result from conceptual or calculation errors.
ChoiceC is incorrect and may result from conceptual or calculation errors.
ChoiceD is incorrect and may result from conceptual or calculation errors.
QUESTION 10
Choice A is correct. It’s given that the rocket contained
467,000 kilograms (kg)
of propellant before launch and had
362,105 kg
remaining exactly
21
seconds
after launch. Finding the difference between the amount, in
kg
, of propellant
before launch and the remaining amount, in
kg
, of propellant after launch
gives the amount, in
kg
, of propellant burned during the
21
seconds:
467,000 362,105 104,895-=
. Dividing the amount of propellant burned by
the number of seconds yields
104,895
21
4,995=
. Thus, an average of
4,995 kg
of
propellant burned each second after launch.
Choice B is incorrect and may result from conceptual or calculation errors.
ChoiceC is incorrect and may result from conceptual or calculation errors.
ChoiceD is incorrect and may result from finding the amount of propellant
burned, rather than the amount of pr
opellant burned each second.
QUESTION 11
Choice B is correct. Multiplying both sides of the given equation by
4
yields
(
4
)(
4x 2
)
4
( )(
12
)
+=
, or
16x +8 =48
. Therefore, the value of
16x +8
is
48
.
Choice A is incorrect and may result from conceptual or calculation errors.
ChoiceC is incorrect and may result from conceptual or calculation errors.
ChoiceD is incorrect and may result from conceptual or calculation errors.
SAT ANSWER EXPLANATIONS
n
MATH: MODULE 1
39 SAT PRACTICE TEST #2 ANSWER EXPLANATIONS
QUESTION 12
Choice D is correct. It’s given that the equation
2
4.9 7 9h tt=- + +
represents
this situation, where
h
is the height, in meters, of the object
t
seconds after it is
kicked. It follows that the height, in meters, from which the object was kicked is
the value of
h
when
t = 0
. Substituting
0
for
t
in the equation
2
4.9 7 9h tt=- + +
yields
( ) ( )
2
4.9 0 7 0 9h =- + +
, or
h = 9
. Therefore, the object was kicked from a
height of
9
meters.
Choice A is incorrect and may result from conceptual or calculation errors.
ChoiceB is incorrect and may result from conceptual or calculation errors.
ChoiceC is incorrect and may result from conceptual or calculation errors.
QUESTION 13
The correct answer is
25
4
. The given equation can be rewritten in the
form
( ) ( )
2
f x ax h k= -+
, where
a
,
h
, and
k
are constants. When
a > 0
,
h
is the value of
x
for which
f
(
x
)
reaches its minimum. The given equation
can be rewritten as
( )
( )
2
50
4
4 126fx x x=-+
, which is equivalent to
( )
( ) ( )
22
2
50
50 50
4
88
4 126fx x x
æö
÷
ç
= -+ - +
÷
ç
÷
÷
ç
èø
. This equation can be rewritten as
( )
( )
( )
2
2
50
50
8
8
4 126fx x
æö
÷
ç
÷
= -- +
ç
÷
ç
÷
ç
èø
, or
( )
( )
( )
2
2
50
50
8
8
4 4 126fx x=-- +
, which is
equivalent to
( )
( )
2
25 121
44
4fx x=--
. Therefore,
25
4
h =
, so the value of
x
for which
f
(
x
)
reaches its minimum is
25
4
. Note that 25/4 and 6.25 are examples of ways to
enter a correct answer.
QUESTION 14
The correct answer is
182
. Let
s
represent the number of small candles the
owner can purchase, and let
represent the number of large candles the owner
can purchase. It’s given that the owner pays
$4.90
per candle to purchase small
candles and
$11.60
per candle to purchase large candles. Therefore, the owner
pays
4.90s
dollars for
s
small candles and
11.60
dollars for
large candles,
which means the owner pays a total of
4.90 +11.60s
dollars to purchase
candles. It’s given that the owner budgets
$2,200
to purchase candles.
Therefore,
4.90 11.60 2,200s
. It’s also given that the owner must purchase a
minimum of
200
candles. Therefore,
200s
. The inequalities
4.90 11.60 2,200s
and
200s
can be combined into one compound
inequality by rewriting the second inequality so that its left-hand side is equivalent
to the left-hand side of the first inequality. Subtracting
from both sides of the
inequality
200s
yields
200s ³-
. Multiplying both sides of this inequality
by
4.90
yields
4.90 4.90
(
200
)
s ³-
, or
4.90 980 4.90
s ³-
. Adding
11.60
to
both sides of this inequality yields
4.90 11.60
980 4.90

11.60s + ³- +
, or
4.90 11.60980 6.70s + ³+
. This inequality can be combined with the inequality
4.90 11.60 2,200s
, which yields the compound inequality
SAT ANSWER EXPLANATIONS
n
MATH: MODULE 1
40 SAT PRACTICE TEST #2 ANSWER EXPLANATIONS
980 6.70

4.90 11.60 2,200s+£+ £
. It follows that
980 6.70
2,200
.
Subtracting
980
from both sides of this inequality yields
6.70 2,200£
. Dividing
both sides of this inequality by
6.70
yields approximately
182.09£
. Since the
number of large candles the owner purchases must be a whole number, the
maximum number of large candles the owner can purchase is the largest whole
number less than
182.09
, which is
182
.
QUESTION 15
Choice D is correct. Since
f
is a linear function, it can be defined by an equation
of the form
f
(
x
)
=ax +b
, where
a
and
b
are constants. It’s given that
f
(
0
)
= 8
.
Substituting
0
for
x
and
8
for
f
(
x
)
in the equation
f
(
x
)
=ax +b
yields
8=a
(
0
)
+b
, or
8 = b
. Substituting
8
for
b
in the equation
f
(
x
)
=ax +b
yields
f
(
x
)
=ax +8
. It’s given that
f
(
1
)
=12
. Substituting
1
for
x
and
12
for
(
fx
)
in the
equation
f
(
x
)
=ax +8
yields
12=a
(
1
)
+8
, or
12 =a +8
. Subtracting
8
from both
sides of this equation yields
a = 4
. Substituting
4
for
a
in the equation
f
(
x
)
=ax +8
yields
f
(
x
)
=4x+8
. Therefore, an equation that defines
f
is
f
(
x
)
=4x+8
.
Choice A is incorrect and may result from conceptual or calculation errors.
ChoiceB is incorrect and may result from conceptual or calculation errors.
ChoiceC is incorrect and may result from conceptual or calculation errors.
QUESTION 16
Choice A is correct. The function
f
gives the area of the rectangle, in
2
ft
, if its
width is
w ft
. Since the value of
f
(
14
)
is the value of
f
(
w
)
if
w =14
, it follows
that
f
(
14
)
=1,176
means that
f
(
w
)
is
1,176
if
w =14
. In the given context, this
means that if the width of the rectangle is
14 ft
, then the area of the rectangle is
1,176 ft
2
.
Choice B is incorrect and may result from conceptual errors. Choice C is incorrect
and may result from conceptual errors. Choice D is incorrect and may result from
interpreting
f
(
w
)
as the width, in
ft
, of the rectangle if its area is
w
2
ft
, rather than
as the area, in
2
ft
, of the rectangle if its width is
w ft
.
QUESTION 17
Choice B is correct. Since
PR
and
QS
are diameters of the circle shown,
OS
,
OR
,
OP
, and
OQ
are radii of the circle and are therefore congruent. Since
SOP
and
ROQ
are vertical angles, they are congruent. Therefore, arc
PS
and arc
QR
are formed by congruent radii and have the same angle measure, so
they are congruent arcs. Similarly,
SOR
and
POQ
are vertical angles, so they
are congruent. Therefore, arc
SR
and arc
PQ
are formed by congruent radii and
have the same angle measure, so they are congruent arcs. Let
x
represent the
length of arc
SR
. Since arc
SR
and arc
PQ
are congruent arcs, the length of arc
PQ
can also be represented by
x
. It’s given that the length of arc
PS
is twice the
length of arc
PQ
. Therefore, the length of arc
PS
can be represented by the
expression
2x
. Since arc
PS
and arc
QR
are congruent arcs, the length of arc
QR
can also be represented by
2x
. This gives the expression
x+x+2x+
2x
.
SAT ANSWER EXPLANATIONS
n
MATH: MODULE 1
41 SAT PRACTICE TEST #2 ANSWER EXPLANATIONS
Since it’s given that the circumference is
π
144
, the expression
x+x+2x+2x
is
equal to
π
144
. Thus
2 2 144xxxx++ + =
π
, or
6 144x =
π
. Dividing both sides of
this equation by
6
yields
π
24x =
. Therefore, the length of arc
QR
is
( )
π
2 24
, or
π
48
.
Choice A is incorrect. This is the length of arc
PQ
, not arc
QR
. ChoiceC is
incorrect and may result from conceptual or calculation errors. ChoiceD is
incorrect and may result from conceptual or calculation errors.
QUESTION 18
Choice C is correct. Let
x
represent the number of children in a whale-watching
tour group. Let
y
represent the number of adults in this group. Because it’s given
that
21
people are in a group and the group consists of adults and children, it
must be true that
x+y=21
. Since the company’s revenue is
60
dollars per
child, the total revenue from
x
children in this group was
60x
dollars. Since
the company’s revenue is
80
dollars per adult, the total revenue from
y
adults
in this group was
80y
dollars. Because it’s given that the total revenue for this
group was
1,440
dollars, it must be true that
60x+80y=1,440
. The equations
x+y=21
and
60x+80y=1,440
form a linear system of equations that can
be solved to find the value of
x
, which represents the number of children in
the group, using the elimination method. Multiplying both sides of the equation
x+y=21
by
80
yields
80x+80y=1,680
. Subtracting
60x+80y=1,440
from
80x+80y=1,680
yields
( ) ( )
80 80 60 80 1,680 1,440xy xy+-+= -
, which is
equivalent to
80 60 80 80 240xxyy-+-=
, or
20x = 240
. Dividing both sides
of this equation by
20
yields
x =12
. Therefore,
12
people in the group were
children.
Choice A is incorrect and may result from conceptual or calculation errors.
ChoiceB is incorrect. This is the number of adults in the group, not the number
of children in the group. ChoiceD is incorrect and may result from conceptual or
calculation errors.
QUESTION 19
Choice A is correct. The x-intercept of a graph in the xy-plane is the point on
the graph where
y = 0
. It’s given that function
h
is defined by
h
(
x
)
=4x+28
.
Therefore, the equation representing the graph of
y= h
(
x
)
is
y=4x+28
.
Substituting
0
for
y
in the equation
y=4x+28
yields
0 =4x +28
. Subtracting
28
from both sides of this equation yields
28 4x-=
. Dividing both sides of this
equation by
4
yields
7 x-=
. Therefore, the x-intercept of the graph of
y= h
(
x
)
in the xy-plane is
( )
7, 0-
. It’s given that the x-intercept of the graph of
y= h
(
x
)
is
(
a, 0
)
. Therefore,
7a =-
. The y-intercept of a graph in the xy-plane is the point on
the graph where
x = 0
. Substituting
0
for
x
in the equation
y=4x+28
yields
y =4
(
0
)
+28
, or
y = 28
. Therefore, the y-intercept of the graph of
y= h
(
x
)
in the
xy-plane is
(
0, 28
)
. It’s given that the y-intercept of the graph of
y= h
(
x
)
is
(
0, b
)
.
Therefore,
b = 28
. If
7a =-
and
b = 28
, then the value of
a+ b
is
7 28-+
, or
21
.
SAT ANSWER EXPLANATIONS
n
MATH: MODULE 1
42 SAT PRACTICE TEST #2 ANSWER EXPLANATIONS
Choice B is incorrect. This is the value of
b
, not
a
+b
. Choice C is incorrect and
may result from conceptual or calculation errors. Choice D is incorrect. This is the
value of
ab-+
, not
a+ b
.
QUESTION 20
The correct answer is
8
. Since each term of the given expression,
32
2 42 208xx x++
, has a factor of
2x
, the expression can be rewritten as
( )
( ) ( )
2
2 2 21 2 104xx x x x++
, or
( )
2
2 21 104xx x++
. Since the values
8
and
13
have a sum of
21
and a product of
104
, the expression
2
21 104xx++
can be
factored as
(
x+8
)(
x+13
)
. Therefore, the given expression can be factored as
2x
(
x+8
)(
x+13
)
. It follows that the factors of the given expression are
2
,
x
,
x +8
, and
x +13
. Of these factors, only
x +8
and
x +13
are of the form
x+ b
,
where
b
is a positive constant. Therefore, the possible values of
b
are
8
and
13
.
Thus, the smallest possible value of
b
is
8
.
QUESTION 21
The correct answer is
29
2
. According to the first equation in the given system, the
value of
y
is
1.5-
. Substituting
1.5-
for
y
in the second equation in the given
system yields
2
1.5 8x xa- =++
. Adding
1.5
to both sides of this equation yields
2
081.5x xa= + ++
. If the given system has exactly one distinct real solution, it
follows that
2
081.5x xa= + ++
has exactly one distinct real solution. A quadratic
equation in the form
2
0 px qx r= ++
, where
p
,
q
, and
r
are constants, has
exactly one distinct real solution if and only if the discriminant,
2
4q pr-
, is equal
to
0
. The equation
2
081.5x xa= + ++
is in this form, where
p =1
,
q = 8
, and
r
=a
+
1.5
. Therefore, the discriminant of the equation
2
081.5x xa= + ++
is
( ) ( )( )
2
8 4 1 1.5a-+
, or
58 4a-
. Setting the discriminant equal to
0
to solve for
a
yields
58 4 0a-=
. Adding
4a
to both sides of this equation yields
58 = 4a
.
Dividing both sides of this equation by
4
yields
58
4
a=
, or
29
2
a=
. Therefore, if the
given system of equations has exactly one distinct real solution, the value of
a
is
29
2
. Note that 29/2 and 14.5 are examples of ways to enter a correct answer.
QUESTION 22
Choice B is correct. It’s given that
( ) ( )( )( )
654fxxxx=+ + -
and
( )
3y fx=-
.
Substituting
( )( )( )
654xxx++-
for
f
(
x
)
in the equation
( )
3y fx=-
yields
( )( )( )
6 5 43yxxx= + + --
. Substituting
6-
for
x
in this equation yields
( )( )( )
66 65 64 3y =-+ -+ -- -
, or
3y =-
. Substituting
5-
for
x
in the equation
( )( )( )
6 5 43yxxx= + + --
yields
( )( )( )
56 55 54 3y =-+ -+ -- -
, or
3y =-
.
Substituting
4
for
x
in the equation
( )( )( )
6 5 43yxxx= + + --
yields
( )( )( )
4 64 54 4 3y =+ + --
, or
3y =-
. Therefore, when
6x =-
then
3y =-
,
when
5x =-
then
3y =-
, and when
x = 4
then
3y =-
. Thus, the table of values
in choice B represents
( )
3y fx=-
.
SAT ANSWER EXPLANATIONS
n
MATH: MODULE 1
43 SAT PRACTICE TEST #2 ANSWER EXPLANATIONS
Choice A is incorrect. This table represents
3yx=-
rather than
( )
3y fx=-
.
Choice C is incorrect. This table represents
y=x3
+
rather than
( )
3y fx=-
.
Choice D is incorrect. This table represents
y=f
(
x
)
+3
rather than
( )
3y fx=-
.
QUESTION 23
Choice C is correct. Since the value of
q
(
x
)
decreases by a fixed percentage,
45%
, for every increase in the value of
x
by
1
, the function
q
is a decreasing
exponential function. A decreasing exponential function can be written in the form
( )
( )
100
1
x
p
qx a=-
, where
a
is the value of
q
(
0
)
and the value of
q
(
x
)
decreases
by
%p
for every increase in the value of
x
by
1
. If
q
(
0
)
=14
, then
a =14
. Since
the value of
q
(
x
)
decreases by
45%
for every increase in the value of
x
by
1
,
p = 45
. Substituting
14
for
a
and
45
for
p
in the equation
( )
( )
100
1
x
p
qx a=-
yields
( )
( )
45
100
14 1
x
qx=-
, which is equivalent to
( ) ( )
14 1 0.45
x
qx=-
, or
( ) ( )
14 0.55
x
qx=
.
Choice A is incorrect and may result from conceptual or calculation errors.
ChoiceB is incorrect and may result from conceptual or calculation errors.
Choice D is incorrect. For this function, the value of
q
(
x
)
increases, rather than
decreases, by
45%
for every increase in the value of
x
by
1
.
QUESTION 24
Choice A is correct. An equation for the graph shown can be written in slope-
intercept form
y =mx +b
, where
m
is the slope of the graph and its y-intercept
is
(
0, b
)
. Since the y-intercept of the graph shown is
(
0, 2
)
, the value of
b
is
2
.
Since the graph also passes through the point
(
4, 1
)
, the slope can be calculated
as
12
40
-
-
, or
1
4
-
. Therefore, the value of
m
is
1
4
-
. Substituting
1
4
-
for
m
and
2
for
b
in the equation
y =mx +b
yields
1
4
2yx=- +
. It’s given that an equation
for the graph shown is
y=f
(
x
)
+14
. Substituting
f
(
x
)
+14
for
y
in the equation
1
4
2yx=- +
 yields
1
4
14 2fx x+ =- +
( )
. Subtracting
14
from both sides of this
equation yields
( )
1
4
12fx x=- -
.
Choice B is incorrect and may result from conceptual or calculation errors.
ChoiceC is incorrect and may result from conceptual or calculation errors.
ChoiceD is incorrect and may result from conceptual or calculation errors.
QUESTION 25
Choice B is correct. It’s given that right triangle
RST
is similar to triangle
UVW
,
where
S
corresponds to
V
and
T
corresponds to
W
. It’s given that the side
lengths of the right triangle
RST
are
RS = 20
,
ST = 48
, and
TR =52
.
Corresponding angles in similar triangles are equal. It follows that the measure of
angle
T
is equal to the measure of angle
W
.The hypotenuse of a right triangle is
SAT ANSWER EXPLANATIONS
n
MATH: MODULE 1
44 SAT PRACTICE TEST #2 ANSWER EXPLANATIONS
the longest side. It follows that the hypotenuse of triangle
RST
is side
TR
. The
hypotenuse of a right triangle is the side opposite the right angle. Therefore, angle
S
is a right angle. The adjacent side of an acute angle in a right triangle is the side
closest to the angle that is not the hypotenuse. It follows that the adjacent side of
angle
T
is side
ST
. The opposite side of an acute angle in a right triangle is the
side across from the acute angle. It follows that the opposite side of angle
T
is
side
RS
. The tangent of an acute angle in a right triangle is the ratio of the length
of the opposite side to the length of the adjacent side. Therefore,
tan
RS
ST
T =
.
Substituting
20
for
RS
and
48
for
ST
in this equation yields
20
48
tan T =
, or
5
12
tan T =
. The tangents of two acute angles with equal measures are equal.
Since the measure of angle
T
is equal to the measure of angle
W
, it follows that
tan T= tan W
. Substituting
5
12
for
tan T
in this equation yields
5
12
tan W=
.
Therefore, the value of
tan W
is
5
12
.
Choice A is incorrect. This is the value of
sin W
. Choice C is incorrect. This is the
value of
cos W
. Choice D is incorrect. This is the value of
1
tan W
.
QUESTION 26
Choice A is correct. It’s given that
w
represents the total wall area, in square feet.
Since the walls of the room will be painted twice, the amount of paint, in gallons,
needs to cover
2w
square feet. It’s also given that one gallon of paint will cover
220
square feet. Dividing the total area, in square feet, of the surface to be
painted by the number of square feet covered by one gallon of paint gives the
number of gallons of paint that will be needed. Dividing
2w
by
220
yields
2
220
w
,
or
w
110
. Therefore, the equation that represents the total amount of paint
P
, in
gallons, needed to paint the walls of the room twice is
110
w
P =
.
Choice B is incorrect and may result from conceptual or calculation errors.
ChoiceC is incorrect and may result fr
om finding the amount of paint needed to
paint the walls once rather than twice. Choice D is incorrect and may result from
conceptual or calculation errors.
QUESTION 27
The correct answer is
9.87
. It’s given that the number
a
is
110%
greater than the
number
b
. It follows that
( )
110
100
1ab=+
, or
a= 2.1b
. It’s also given that the number
b
is
90%
less than
47
. It follows that
( )
( )
90
100
1 47b =-
, or
b = 0.1
(
47
)
, which
yields
b = 4.7
. Substituting
4.7
for
b
in the equation
a= 2.1b
yields
a = 2.1
(
4.7
)
,
which is equivalent to
a = 9.87
. Therefore, the value of
a
is
9.87
.
45 SAT PRACTICE TEST #2 ANSWER EXPLANATIONS
SAT ANSWER EXPLANATIONS
n
MATH: MODULE 2
Math
Module 2
(27 questions)
QUESTION 1
Choice A is correct. It’s given that
20%
of the students surveyed responded that
they intend to enroll in the study program. Therefore, the proportion of students
in Spanish club who intend to enroll in the study program, based on the survey,
is
0.20
. Since there are
55
total students in Spanish club, the best estimate for
the total number of these students who intend to enroll in the study program is
55
(
0.20
)
, or
11
.
Choice B is incorrect. This is the best estimate for the percentage, rather than
the total number, of students in Spanish club who intend to enroll in the study
program. Choice C is incorrect. This is the best estimate for the total number of
Spanish club students who do not intend to enroll in the study program. Choice D
is incorrect. This is the total number of students in Spanish club.
QUESTION 2
Choice A is correct. Since Jay walks at a speed of
3
miles per hour for
w
hours,
Jay walks a total of
3w
miles. Since Jay runs at a speed of
5
miles per hour for
r
hours, Jay runs a total of
5r
miles. Therefore, the total number of miles Jay
travels can be represented by
3w+5r
. Since the combined total number of miles
is
14
, the equation
3w+5r=14
represents this situation.
Choice B is incorrect and may result from conceptual errors. ChoiceC is incorrect
and may result from conceptual errors. ChoiceD is incorrect and may result from
conceptual errors.
QUESTION 3
Choice A is correct. The line of best fit shown intersects the y-axis at a positive
y-value and has a positive slope. The graph of an equation of the form
y =a +bx
,
where
a
and
b
are constants, intersects the y-axis at a y-value of
a
and has a
46 SAT PRACTICE TEST #2 ANSWER EXPLANATIONS
SAT ANSWER EXPLANATIONS
n
MATH: MODULE 2
slope of
b
. Of the given choices, only choice A represents a line that intersects
the y-axis at a positive y-value,
2.8
, and has a positive slope,
1.7
.
Choice B is incorrect. This equation represents a line that has a negative slope,
not a positive slope. Choice C is incorrect. This equation represents a line that
intersects the y-axis at a negative y-value, not a positive y-value. Choice D is
incorrect. This equation represents a line that intersects the y-axis at a negative
y-value, not a positive y-value, and has a negative slope, not a positive slope.
QUESTION 4
Choice D is correct. Because the graph of
y=f
(
x
)
is shown, the value of
f
(
0
)
is the value of
y
on the graph that corresponds with
x = 0
. When
x = 0
, the
corresponding value of
y
is
3
. Therefore, the value of
f
(
0
)
is
3
.
Choice A is incorrect and may result from conceptual errors. ChoiceB is incorrect
and may result from conceptual errors. ChoiceC is incorrect and may result from
conceptual errors.
QUESTION 5
Choice B is correct. Applying the commutative property of multiplication, the
expression
(
m
4
q
4
z
-1
)(
mq
53
z
)
can be rewritten as
(
mm
4
)(
qq
4 5
)(
z
-1
z
)
3
. For
positive values of
x
,
(
xx
a
)(
b
)
= x
a+b
. Therefore, the expression
-
(
mm
4
)(
qq
4 5
)(
z
13
z
)
can be rewritten as
(
mq
4+1
)(
4+5
)(
z
-+13
)
, or
mq
59
z
2
.
Choice A is incorrect and may result from multiplying, not adding, the exponents.
Choice C is incorrect and may result from conceptual or calculation errors.
ChoiceD is incorrect and may result from conceptual or calculation errors.
QUESTION 6
The correct answer is
79
. The median of a data set with an odd number of values
is the middle value of the set when the values are ordered from least to greatest.
Because the given data set consists of nine values that are ordered from least to
greatest, the median is the fifth value in the data set. Therefore, the median of the
data shown is
79
.
QUESTION 7
The correct answer is
55
. Subtracting
40
from both sides of the given equation
yields
x = 55
. Therefore, the value of
x
is
55
.
QUESTION 8
Choice C is correct. Adding the second equation of the given system to the first
equation yields
5x+-
(
4xy+
)
=15+-
(
2
)
, which is equivalent to
x+y=13
. So
the value of
x+ y
is
13
.
47 SAT PRACTICE TEST #2 ANSWER EXPLANATIONS
SAT ANSWER EXPLANATIONS
n
MATH: MODULE 2
Choice A is incorrect and may result from conceptual or calculation errors.
ChoiceB is incorrect. This is the value of
-+
(
x
)
y
. ChoiceD is incorrect and may
result from conceptual or calculation errors.
QUESTION 9
Choice A is correct. It’s given that the function
g
models the number of gallons
that remain from a full gas tank in a car after driving
m
miles. In the given
function
g
(
m
)
=-0.05m+12.1
, the coefficient of
m
is
-0.05
. This means that
for every increase in the value of
m
by
1
, the value of
g
(
m
)
decreases by
0.05
. It
follows that for each mile driven, there is a decrease of
0.05
gallons of gasoline.
Therefore,
0.05
gallons of gasoline are used to drive each mile.
Choice B is incorrect and represents the number of gallons of gasoline in a full
gas tank. Choice C is incorrect and may result from conceptual errors. Choice D is
incorrect and may result from conceptual errors.
QUESTION 10
Choice C is correct. Multiplying each side of the given equation by
y
yields the
equivalent equation
y
=
7b
11x
11
. Dividing each side of this equation by yields
y
=
77b
x
, or
x =
y
77b
.
Choice A is incorrect. This equation is not equivalent to the given equation.
ChoiceB is incorrect. This equation is not equivalent to the given equation.
ChoiceD is incorrect. This equation is not equivalent to the given equation.
QUESTION 11
Choice B is correct. Since the point
(
x, y
)
is an intersection point of the graphs of
the given equations in the xy-plane, the pair
(
x, y
)
should satisfy both equations,
and thus is a solution of the given system. According to the first equation,
y = 76
.
Substituting
76
in place of
y
in the second equation yields
2
x -=5 76
. Adding
5
to both sides of this equation yields
x
2
= 81
. Taking the square root of both
sides of this equation yields two solutions:
x = 9
and
x =-9
. Of these two
solutions, only
-9
is given as a choice.
Choice A is incorrect and may result from conceptual or calculation errors.
ChoiceC is incorrect and may result from conceptual or calculation errors.
ChoiceD is incorrect. This is the value of coordinate
y
, rather than
x
, of the
intersection point
(
x, y
)
.
QUESTION 12
Choice A is correct. It’s given that the point
(
x, 53
)
is a solution to the given
system of inequalities in the xy-plane. This means that the coordinates of the
point, when substituted for the variables
x
and
y
, make both of the inequalities
in the system true. Substituting
53
for
y
in the inequality
y >14
yields
53>14
,
which is true. Substituting
53
for
y
in the inequality
4x+y<18
yields
4x +53<18
. Subtracting
53
from both sides of this inequality yields
4x <-35
.
48 SAT PRACTICE TEST #2 ANSWER EXPLANATIONS
SAT ANSWER EXPLANATIONS
n
MATH: MODULE 2
Dividing both sides of this inequality by
4
yields
x <-8.75
. Therefore,
x
must be
a value less than
-8.75
. Of the given choices, only
-9
is less than
-8.75
.
Choice B is incorrect. Substituting
-5
for
x
and
53
for
y
in the inequality
4x+y<18
yields
4
(
-+5
)
53<18
, or
33<18
, which is not true. Choice C is
incorrect. Substituting
5
for
x
and
53
for
y
in the inequality
4x+y<18
yields
4
(
5
)
+<53 18
, or
73<18
, which is not true. Choice D is incorrect. Substituting
9
for
x
and
53
for
y
in the inequality
4x+y<18
yields
4
(
9
)
+<53 18
, or
89<18
, which is not true.
QUESTION 13
The correct answer is
240
. It’s given that
80%
of the
300
seeds sprouted.
Therefore, the number of seeds that sprouted can be calculated by multiplying
the number of seeds that were planted by
80
100
, which gives
(
80
300
100
)
, or
240
.
QUESTION 14
The correct answer is
2
. Substituting
8
for
f
(
x
)
in the given equationyields
8= 4x
. Dividing the left- and right-hand sides of this equation by
4
yields
x = 2
.
Therefore, the value of
x
is
2
when
f
(
x
)
= 8
.
QUESTION 15
Choice B is correct. The given expression has a common factor of
2
in the
denominator, so the expression can be rewritten as
8xx
(
--73
) (
x-
)
27
(
x-
)
7
. The three
terms in this expression have a common factor of
(
x -7
)
. Since it’s given that
x >7
,
x
can’t be equal to
7
, which means
(
x -7
)
can’t be equal to
0
. Therefore,
each term in the expression,
8xx
(
--73
) (
x-
)
27
(
x-
)
7
, can be divided by
(
x -7
)
, which
gives
83x-
2
.
Choice A is incorrect and may result from conceptual or calculation errors.
ChoiceC is incorrectand may result from conceptual or calculation errors.
ChoiceD is incorrectand may result from conceptual or calculation errors.
QUESTION 16
Choice C is correct. It’s given that line
r
is perpendicular to line
p
in the
xy-plane. This means that the slope of line
r
is the negative reciprocal of the
slope of line
p
. If the equation for line
p
is rewritten in slope-intercept form
y =mx +b
, where
m
and
b
are constants, then
m
is the slope of the line and
(
0, b
)
is its y-intercept. Subtracting
18x
from both sides of the equation
2y+18x=9
yields
2y=-18 +9x
. Dividing both sides of this equation by
2
yields
y=-9 +
9
x
2
. It follows that the slope of line
p
is
-9
. The negative
reciprocal of a number is
-1
divided by the number. Therefore, the negative
reciprocal of
-9
is
-1
-9
, or
1
9
. Thus, the slope of line
r
is
1
9
.
49 SAT PRACTICE TEST #2 ANSWER EXPLANATIONS
SAT ANSWER EXPLANATIONS
n
MATH: MODULE 2
Choice A is incorrect. This is the slope of line
p
, not line
r
. Choice B is incorrect.
This is the reciprocal, not the negative reciprocal, of the slope of line
p
. Choice D
is incorrect. This is the negative, not the negative reciprocal, of the slope of line
p
.
QUESTION 17
Choice D is correct. The y-intercept of a graph in the ty-plane is the point where
t = 0
. For the given function
f
, they-intercept of the graph of
y= f
(
t
)
in the
ty-plane can be found by substituting
0
for
t
in the equation
y 8,000
(
0.65
)
t
=
,
which gives
y 8,000
(
0.65
)
0
=
. This is equivalent to
y = 8,000
(
1
)
, or
y = 8,000
.
Therefore, the y-intercept of the graph of
y= f
(
t
)
is
(
0, 8,000
)
. It’s given that the
function
f
models the number of coupons a company sent to their customers at
the end of each year. Therefore,
f
(
t
)
represents the estimated number of
coupons the company sent to their customers at the end of each year. It’s also
given that
t
represents the number of years since the end of
1998
. Therefore,
t = 0
represents
0
years since the end of
1998
, or the end of
1998
. Thus, the
best interpretation of the y-intercept of the graph of
y= f
(
t
)
isthat the estimated
number of coupons the company sent to their customers at the end of
1998
was
8,000
.
Choice A is incorrect and may result from conceptual or calculation errors.
ChoiceB is incorrect and may result from conceptual or calculation errors.
ChoiceC is incorrect and may result from conceptual or calculation errors.
QUESTION 18
Choice C is correct. It’s given that triangle
XYZ
is similar to triangle
RST
, such
that
X
,
Y
, and
Z
correspond to
R
,
S
, and
T
, respectively. Since corresponding
angles of similar triangles are congruent, it follows that the measure of
Z
is
congruent to the measure of
T
. It’s given that the measure of
Z
o
o
is
20
°
.
Therefore, the measure of
T
is
20
°
.
Choice A is incorrect and may result from a conceptual error. ChoiceB is
incorrect. This is half the measure of
Z
. ChoiceD is incorrect. This is twice the
measure of
Z
.
QUESTION 19
Choice B is correct. A system of two linear equations in two variables,
x
and
y
, has no solution if the lines represented by the equations in the xy-plane
are parallel and distinct. Lines represented by equations in standard form,
Ax +By =C
and
Dx +Ey =F
, are parallel if the coefficients for
x
and
y
in one
equation are proportional to the corresponding coefficients in the other
equation, meaning
D
=
E
AB
; and the lines are distinct if the constants are not
proportional, meaning
F
C
is not equal to
D
A
or
E
B
. The given equation,
y=6x+18
,
can be written in standard form by subtracting
6x
from both sides of the
equation to yield
-6xy+=18
. Therefore, the given equation can be written in the
form
Ax +By =C
, where
A =-6
,
B =1
, and
C =18
. The equation in choice B,
50 SAT PRACTICE TEST #2 ANSWER EXPLANATIONS
SAT ANSWER EXPLANATIONS
n
MATH: MODULE 2
-6xy+=22
, is written in the form
Dx +Ey =F
, where
D =-6
,
E =1
, and
F = 22
.
Therefore,
D
=
-6
A -6
, which can be rewritten as
D
=
A
1
;
E
=
1
B 1
, which can be rewritten
as
E
=
B
1
; and
F
=
22
C 18
, which can be rewritten as
F
=
11
C 9
. Since
D
=
A
1
,
E
=
B
1
, and
F
C
is not equal to
1
, it follows that the given equation and the equation
-6xy+=22
are parallel and distinct. Therefore, a system of two linear equations consisting of
the given equation and the equation
-6xy+=22
has no solution. Thus,
theequation in choice B could be the second equation in the system.
Choice A is incorrect. The equation
-6xy+=18
and the given equation
represent the same line in the xy-plane. Therefore, a system of these linear
equations would have infinitely many solutions, rather than no solution.ChoiceC
is incorrect. The equation
-12xy+=36
and the given equation represent
lines in the xy-plane that are distinct and not parallel. Therefore, a system of
these linear equations would have exactly one solution, rather than no solution.
ChoiceD is incorrect. The equation
-12xy+=18
and the given equation
represent lines in the xy-plane that are distinct and not parallel. Therefore, a
system of these linear equations would haveexactly one solution, rather than no
solution.
QUESTION 20
The correct answer is
986
. The area,
A
, of a rectangle is given by
A= w
, where
is the length of the rectangle and
w
is its width. It’s given that the length of the
rectangle is
34
centimeters (cm) and the width is
29
cm. Substituting
34
for
and
29
for
w
in the equation
A= w
yields
A =
(
34
)(
29
)
, or
A = 986
. Therefore,
the area, in square centimeters, of this rectangle is
986
.
QUESTION 21
The correct answer is
35
. The first equation in the given system of equations
defines
y
as
4x +1
. Substituting
4x +1
for
y
in the second equation in the given
system of equations yields
4
(
4x+=1
)
15 8x-
. Applying the distributive property
on the left-hand side of this equation yields
16x+4 =15 8x-
. Subtracting
15x
from each side of this equation yields
x +4=-8
. Subtracting
4
from each side
of this equation yields
x =-12
. Substituting
-12
for
x
in the first equation of
the given system of equations yields
y =-4
(
12
)
+1
,or
y =-47
. Substituting
-12
for
x
and
-47
for
y
into the expression
x- y
yields
-12--
(
47
)
, or
35
.
QUESTION 22
Choice D is correct. The number of solutions of a quadratic equation of the form
ax
2
+bx +=c 0
, where
a
,
b
, and
c
are constants, can be determined by the
value of the discriminant,
b
2
-4ac
. If the value of the discriminant is positive, then
the quadratic equation has exactly two distinct real solutions. If the value of the
discriminant is equal to zero, then the quadratic equation has exactly one real
solution. If the value of the discriminant is negative, then the quadratic equation
51 SAT PRACTICE TEST #2 ANSWER EXPLANATIONS
SAT ANSWER EXPLANATIONS
MATH: MODULE 2
has zero real solutions. In the given equation,
5x
2
+10x +16 = 0
,
a = 5
,
b =10
,
and
c =16
. Substituting these values for
a
,
b
, and
c
in
b
2
-4ac
yields
(
10
)
2
-4 5
( )(
16
)
, or
-220
. Since the value of its discriminant is negative, the given
equation has zero real solutions. Therefore, the number of distinct real solutions
the given equation has is zero.
Choice A is incorrect and may result from conceptual or calculation errors.
Choice B is incorrect and may result from conceptual or calculation errors.
Choice C is incorrect and may result from conceptual or calculation errors.
QUESTION 23
Choice B is correct. Since
1
mile is equal to
1,760
yards,
1
square mileis equal
to 1,760
2
, or
3,097,600
, square yards. It’s given that the park has an area of
11,863,808
square yards. Therefore, the park has an area of
1 square mile
(
11,863,808 square yards
)
( )
3,097,600 square yards
, or
11,863,808
3,097,600
square miles. Thus,
the area, in square miles, of the park is
3.83
.
Choice A is incorrect and may result from conceptual or calculation errors.
ChoiceC is incorrect. This is the square root of the area of the park in square
yards, not the area of the park in square miles. ChoiceD is incorrect and may
result from converting
11,863,808
yards to miles, rather than converting
11,863,808
square yards to square miles.
QUESTION 24
Choice C is correct. The graph of the equation
2
(
xh-
)
2
+
(
y -k
)
2
= r
in the
xy-plane is a circle with center
(
h, k
)
and a radius of length
r
. The radius of a
circle is the distance from the center of the circle to any point on the circle. If a
circle in the xy-plane intersects the y-axis at exactly one point, then the
perpendicular distance from the center of the circle to this point on the y-axis
must be equal to the length of the circle’s radius. It follows that the x-coordinate
of the circle’s center must be equivalent to the length of the circle’s radius. In
other words, if the graph of
(
xh
)
2
+
(
y -k
)
2
= r
2
-
is a circle that intersects the
y-axis at exactly one point, then
r = h
must be true. The equation in choice C is
2 2
(
x -4
)
+
(
y -9
)
=16
, or
2
(
x -4
)
+
(
y -9
)
2
= 4
2
. This equation is in the form
-
2
(
2
(
xh
)
+ y -k
)
= r
2
, where
h = 4
,
k = 9
, and
r = 4
, and represents a circle in the
xy-plane with center
(
4,9
)
and radius of length
4
. Substituting
4
for
r
and
4
for
h
in the equation
r = h
yields
4 = 4
, or
4= 4
, which is true. Therefore, the
equation in choice C represents a circle in the xy-plane that intersects the y-axis
at exactly one point.
Choice A is incorrect. This is the equation of a circle that does not intersect
the y-axis at any point. Choice B is incorrect. This is an equation of a circle that
intersects the x-axis, not the y-axis, at exactly one point. Choice D is incorrect.
This is the equation of a circle with the center located on the y-axis and thus
intersects the y-axis at exactly two points, not exactly one point.
52 SAT PRACTICE TEST #2 ANSWER EXPLANATIONS
SAT ANSWER EXPLANATIONS
MATH: MODULE 2
QUESTION 25
Choice C is correct. Since angles
B
and
E
each have the same measure and
angles
C
and
F
each have the same measure, triangles
ABC
and
DEF
are
similar, where side
BC
corresponds to side
EF
. To determine whether two
similar triangles are congruent, it is sufficient to determine whether one pair of
corresponding sides are congruent. Therefore, to determine whether triangles
ABC
and
DEF
are congruent, it is sufficient to determine whether sides
BC
and
EF
have equal length. Thus, knowing the lengths of
BC
and
EF
is sufficient to
determine whether triangle
ABC
is congruent to triangle
DEF
.
Choice A is incorrect and may result from conceptual errors. Choice B is
incorrect and may result from conceptual errors. Choice D is incorrect. The given
information is sufficient to determine that triangles
ABC
and
DEF
are similar, but
not whether they are congruent.
QUESTION 26
Choice B is correct. The histograms shown have the same shape, but data setA
contains values between
20
and
60
and data set B contains values between
10
and
50
. Thus, the mean of data set A is greater than the mean of data set B.
Therefore, the smallest possible difference between the mean of data set A
and the mean of data set B is the difference between the smallest possible
mean of data set A and the greatest possible mean of data set B. In data set A,
since there are
3
integers in the interval greater than or equal to
20
but less
than
30
,
4
integers greater than or equal to
30
but less than
40
,
7
integers
greater than or equal to
40
but less than
50
, and
9
integers greater than or
equal to
50
but less than
60
, the smallest possible mean for data set A is
(
3 20
)
+
(
4 30
)
+
(
7 40
)
+
(
9 50
)
23
. In data set B, since there are
3
integers greater than
or equal to
10
but less than
20
,
4
integers greater than or equal to
20
but less
than
30
,
7
integers greater than or equal to
30
but less than
40
, and
9
integers
greater than or equal to
40
but less than
50
, the largest possible mean for
data set B is
3 19
)
+
(
4 29
)
+
(
7 39
)
+
(
9 49
)(
23
. Therefore, the smallest possible
difference between the mean of data set A and the mean of data set B is
(
3 20
)
+
(
4 30
)
+
(
7 40
)
+
(
9 50
) (
3 19
)
+
(
4 29
)
+
(
7 39
)
+
(
9 49
)
-
23 23
, which is equivalent to
(
3 20
) (
3 + 4
) (
4 + 7
) (
7 39 + 9 50
) (
9- 19
) (
30 - 29
) (
40 -
) (
- 49
)
23
. This expression can be
rewritten as
(
29 + 39 +
(
49 3 20
(
-19
)
+4 30-
)
7 40
(
-
)
9 50-
)
23
, or
23
23
, which is equal to
1
.
Therefore, the smallest possible difference between the mean of data set A and
the mean of data set B is
1
.
Choice A is incorrect. This is the smallest possible difference between the ranges,
not the means, of the data sets. Choic
e C is incorrect. This is the difference
between the greatest possible mean, not the smallest possible mean, of data
set A and the greatest possible mean of data set B. Choice D is incorrect. This is
the smallest possible difference between the sum of the values in data set A and
53 SAT PRACTICE TEST #2 ANSWER EXPLANATIONS
SAT ANSWER EXPLANATIONS
MATH: MODULE 2
the sum of the values in data set B, not the smallest possible difference between
the means.
QUESTION 27
The correct answer is
113
. It’s given that the legs of a right triangle have lengths
24
centimeters and
21
centimeters. In a right triangle, the square of the length of
the hypotenuse is equal to the sum of the squares of the lengths of the two legs.
It follows that if
h
represents the length, in centimeters, of the hypotenuse of
the right triangle,
2 2 2
h = 24 +21 . This equation is equivalent to
2
h =1,017. Taking
the square root of each side of this equation yields
h = 1,017
. This equation
can be rewritten as
h = 9 113
, or
h = 9 113
. This equation is equivalent to
h = 3
113
. It’s given that the length of the triangle’s hypotenuse, in centimeters,
can be written in the form
3 d
. It follows that the value of
d
is
113
.
1
Scoring Your Paper
SAT Practice Test #2
IMPORTANT: This guide is for students who completed a paper version of the digital SAT practice test.
The scoring method used for this nondigital version isn’t exactly the same as the digital version.
1 Total Score
400–1600 Scale
Reading
and Writing
Modules 1 & 2 Modules 1 & 2
Math
Total Score
2 Section
Scores
200–800 Scale
Congratulations on completing an SAT
®
practice test.
To score your test, follow the instructions in this guide.
Scores Overview
Each assessment in the SAT Suite (SAT, PSAT/NMSQT®, PSAT
10, and PSAT
8/9) reports test
scores on a common scale.
For more details about scores, visit sat.org/scores.
The College Board Assessment Design & Development team developed the practice tests
using the same processes and review standards they used when developing the actual SAT.
2
How to Calculate Your Practice Test Scores
The worksheets on pages 4 and 5 help you calculate your test scores.
GET SET UP
1
In addition to your practice test, you’ll need
the conversion tables and answer key at the
end of this guide.
Have your students make time to take
the practice test.
It is one of the best ways to get ready
for the SAT.
The SAT
®
Practice Test for
Classroom Use
5
SAT Practice Test Worksheet:
Section and Total Scores

RAW SCORE
Reading and Writing
Section Score
Math
Section Score
(# OF CORRECT ANSWERS) LOWE R UPPER LOWER UPPER
0 200 200 200 200
1 200 200 200 200
2 200 200 200 200
3 200 200 200 200
4 200 200 200 200
5 200 200 200 200
6 200 200 200 200
7 200 210 200 220
8 210 230 200 230
9 220 240 220 250
10 230 250 250 280
11 250 270 270 300
12 260 280 290 320
13 280 300 300 330
14 290 310 310 340
15 300 320 320 350
16 330 350 330 360
17 350 370 340 370
18 360 380 350 380
19 370 390 360 390
20 380 400 370 400
21 390 410 380 410
22 400 420 400 430
23 410 430 410 440
24 420 440 420 450
25 430 450 430 460
26 430 450 440 470
27 440 460 450 480
28 450 470 460 490
29 460 480 470 500
30 460 480 480 510
31 470 490 490 520
32 480 500 510 540
33 490 510 520 550
RAW SCORE
Reading and Writing
Section Score
Math
Section Score
(# OF CORRECT ANSWERS) LOWE R UPPER LOWER UPPER
34 490 510 530 560
35 500 520 540 570
36 510 530 550 580
37 520 540 560 590
38 520 540 570 600
39 530 550 590 620
40 540 560 600 630
41 550 570 610 640
42 550 570 620 650
43 560 580 630 660
44 570 590 640 670
45 570 590 660 690
46 580 600 670 700
47 590 610 690 720
48 590 610 700 730
49 600 620 720 750
50 610 630 740 770
51 610 630 760 790
52 620 640 770 790
53 630 650 790 800
54 640 660 800 800
55 640 660
56 650 670
57 660 680
58 670 690
59 680 700
60 690 710
61 700 720
62 710 730
63 720 740
64 740 760
65 770 780
66 790 800
READING AND
WRITING SECTION
MODULE 1
RAW SCORE
(0-27)
READING AND
WRITING SECTION
MODULE 2
RAW SCORE
(0-27)
READING AND
WRITING
RAW SCORE
(0–54)
READING AND
WRITING
SECTION SCORE

MATH SECTION
MODULE 1
RAW SCORE
(0-22)
MATH SECTION
MODULE 2
RAW SCORE
(0-22)
MATH SECTION
RAW SCORE

TOTAL
SAT SCORE

=+
+ =
CONVERT
CONVERT
MATH
SECTION SCORE

+
READING AND WRITING
SECTION SCORE

=
Raw Score Conversion Table: Section Scores
Conversion:
Calculate Your Section and Total Scores
Use the table at the bottom of this page to convert your raw scores from
the previous worksheet into section and test scores.
SCORE YOUR PRACTICE TEST
2
Compare your answers to the answer key on
page 4, and count your total correct answers
for each section. Write the number of correct
answers for each section in the answer key
at the bottom of that section.
4
SAT Practice Test Worksheet:
Answer Key
Mark each of your correct answers below, then add them up to get your raw score on each module.
Math
Module 1 Module 2
Reading and Writing
Module 1 Module 2
MATH SECTION
RAW SCORE
(Total # of Correct Answers)
READING AND WRITING SECTION
RAW SCORE
(Total # of Correct Answers)
QUESTION #
CORRECT
MARK YOUR
CORRECT
ANSWERS
1 B
2 C
3 C
4 B
5 D
6 C
7 B
8 D
9 D
10 A
11 A
12 A
13 A
14 D
15 C
16 B
17 B
18 A
19 A
20 A
21 B
22 A
23 C
24 C
25 C
26 D
27 C
28 C
29 B
30 C
31 C
32 D
33 D
QUESTION #
CORRECT
MARK YOUR
CORRECT
ANSWERS
1 C
2 B
3 D
4 B
5 C
6 C
7 A
8 D
9 B
10 A
11 B
12 A
13 C
14 D
15 C
16 B
17 B
18 B
19 C
20 A
21 D
22 A
23 C
24 C
25 A
26 B
27 A
28 B
29 D
30 D
31 A
32 C
33 B
QUESTION #
CORRECT
MARK YOUR
CORRECT
ANSWERS
1 C
2 C
3 D
4 D
5 C
6 180
7 5
8 A
9 B
10 A
11 B
12 D
13 25/4; 6.25
14 182
15 D
16 A
17 B
18 C
19 A
20 8
21 14.5; 29/2
22 B
23 C
24 A
25 B
26 A
27 9.87; 987/100
QUESTION #
CORRECT
MARK YOUR
CORRECT
ANSWERS
1 A
2 A
3 A
4 D
5 B
6 79
7 55
8 C
9 A
10 C
11 B
12 A
13 240
14 2
15 B
16 C
17 D
18 C
19 B
20 986
21 35
22 D
23 B
24 C
25 C
26 B
27 113
CALCULATE YOUR SCORES
3
Using your marked-up answer key and the
conversion tables, follow the directions on
page 5 to get your section and test scores.
5
SAT Practice Test Worksheet:
Section and Total Scores

RAW SCORE
Reading and Writing
Section Score
Math
Section Score
(# OF CORRECT ANSWERS) LOWE R UPPER LOWER UPPER
0 200 200 200 200
1 200 200 200 200
2 200 200 200 200
3 200 200 200 200
4 200 200 200 200
5 200 200 200 200
6 200 200 200 200
7 200 210 200 220
8 210 230 200 230
9 220 240 220 250
10 230 250 250 280
11 250 270 270 300
12 260 280 290 320
13 280 300 300 330
14 290 310 310 340
15 300 320 320 350
16 330 350 330 360
17 350 370 340 370
18 360 380 350 380
19 370 390 360 390
20 380 400 370 400
21 390 410 380 410
22 400 420 400 430
23 410 430 410 440
24 420 440 420 450
25 430 450 430 460
26 430 450 440 470
27 440 460 450 480
28 450 470 460 490
29 460 480 470 500
30 460 480 480 510
31 470 490 490 520
32 480 500 510 540
33 490 510 520 550
RAW SCORE
Reading and Writing
Section Score
Math
Section Score
(# OF CORRECT ANSWERS) LOWE R UPPER LOWER UPPER
34 490 510 530 560
35 500 520 540 570
36 510 530 550 580
37 520 540 560 590
38 520 540 570 600
39 530 550 590 620
40 540 560 600 630
41 550 570 610 640
42 550 570 620 650
43 560 580 630 660
44 570 590 640 670
45 570 590 660 690
46 580 600 670 700
47 590 610 690 720
48 590 610 700 730
49 600 620 720 750
50 610 630 740 770
51 610 630 760 790
52 620 640 770 790
53 630 650 790 800
54 640 660 800 800
55 640 660
56 650 670
57 660 680
58 670 690
59 680 700
60 690 710
61 700 720
62 710 730
63 720 740
64 740 760
65 770 780
66 790 800
READING AND
WRITING SECTION
MODULE 1
RAW SCORE
(0-27)
READING AND
WRITING SECTION
MODULE 2
RAW SCORE
(0-27)
READING AND
WRITING
RAW SCORE
(0–54)
READING AND
WRITING
SECTION SCORE

MATH SECTION
MODULE 1
RAW SCORE
(0-22)
MATH SECTION
MODULE 2
RAW SCORE
(0-22)
MATH SECTION
RAW SCORE

TOTAL
SAT SCORE

=+
+ =
CONVERT
CONVERT
MATH
SECTION SCORE

+
READING AND WRITING
SECTION SCORE

=
Raw Score Conversion Table: Section Scores
Conversion:
Calculate Your Section and Total Scores
Use the table at the bottom of this page to convert your raw scores from
the previous worksheet into section and test scores.
3
Get Section and
Total Scores
Your total score on an SAT practice test is the sum of your scores in the
modules for both the Reading and Writing and Math sections. To get your
total score, you’ll convert what we call the “raw score” for each section—
the number of questions you got right in that section—into the “scaled
score” for that section, and then calculate the total score.
GET YOUR READING AND WRITING SECTION SCORE
Calculate your SAT Reading and Writing section score
(it’s on a scale of 200–800).
1
Use the answer key on page 4 to count the number of correct
answers you got on module 1 and module 2.
2
To determine your Reading and Writing raw score, add the number
of correct answers you got on module 1 and module 2.
3
Use the Raw Score Conversion Table: Section Scores on page 5
to turn your raw score into your Reading and Writing section score.
GET YOUR MATH SECTION SCORE
Calculate your SAT Math section score (it’s on a scale of 200–800).
1
Use the answer key on page 4 to count the number of correct
answers you got on module 1 and module 2.
2
To determine your Math raw score, add the number of correct
answers you got on module 1 and module 2.
3
Use the Raw Score Conversion Table: Section Scores on page 5
to turn your raw score into your Math section score.
GET YOUR TOTAL SCORE
Add your Reading and Writing section score to your Math section score.
The result is your total score on the SAT practice test, on a scale of 400–1600.
1 Total Score
400–1600 Scale
2 Section
Scores
200–800 Scale
Reading
and Writing
Modules 1 & 2 Modules 1 & 2
Math
Total Score
Your total score on the SAT practice test
is the sum of your Reading and Writing
section score and your Math section score.
Use the worksheets on
pages 4 and 5 to calculate your
section and total scores.
4
SAT Practice Test Worksheet:
Answer Key
Mark each of your correct answers below, then add them up to get your raw score on each module.
Math
Module 1 Module 2
Reading and Writing
Module 1 Module 2
QUESTION #
CORRECT
MARK YOUR
CORRECT
ANSWERS
1 B
2 C
3 C
4 B
5 D
6 C
7 B
8 D
9 D
10 A
11 A
12 A
13 A
14 D
15 C
16 B
17 B
18 A
19 A
20 A
21 B
22 A
23 C
24 C
25 C
26 D
27 C
28 C
29 B
30 C
31 C
32 D
33 D
QUESTION #
CORRECT
MARK YOUR
CORRECT
ANSWERS
1 C
2 B
3 D
4 B
5 C
6 C
7 A
8 D
9 B
10 A
11 B
12 A
13 C
14 D
15 C
16 B
17 B
18 B
19 C
20 A
21 D
22 A
23 C
24 C
25 A
26 B
27 A
28 B
29 D
30 D
31 A
32 C
33 B
READING AND WRITING SECTION
RAW SCORE
(Total # of Correct Answers)
QUESTION #
CORRECT
MARK YOUR
CORRECT
ANSWERS
1 C
2 C
3 D
4 D
5 C
6 180
7 5
8 A
9 B
10 A
11 B
12 D
13 25/4; 6.25
14 182
15 D
16 A
17 B
18 C
19 A
20 8
21 14.5; 29/2
22 B
23 C
24 A
25 B
26 A
27 9.87; 987/100
QUESTION #
CORRECT
MARK YOUR
CORRECT
ANSWERS
1 A
2 A
3 A
4 D
5 B
6 79
7 55
8 C
9 A
10 C
11 B
12 A
13 240
14 2
15 B
16 C
17 D
18 C
19 B
20 986
21 35
22 D
23 B
24 C
25 C
26 B
27 113
MATH SECTION
RAW SCORE
(Total # of Correct Answers)
5
SAT Practice Test Worksheet:
Section and Total Scores
Conversion:
Calculate Your Section and Total Scores
Use the table at the bottom of this page to convert your raw scores from
the previous worksheet into section and test scores.
READING AND
WRITING SECTION
MODULE 1
RAW SCORE
(0–27)
+
READING AND
WRITING SECTION
MODULE 2
RAW SCORE
(0–27)
=
READING AND
WRITING
RAW SCORE
(0–54)
CONVERT
READING AND
WRITING
SECTION SCORE
(200–800)
MATH SECTION
MODULE 1
RA
W SCORE
(0–22)
+
MATH SECTION
MODULE 2
RAW SCORE
(0–22)
=
MATH SECTION
RAW SCORE
(0–44)
CONVERT
MATH
SECTION SCORE
(200–800)
+
READING AND WRITING
SECTION SCORE
(200–800)
=
TOTAL
SAT SCORE
(400–1600)
Raw Score Conversion Table: Section Scores
RAW SCORE
Reading and Writing
Section Score
Math
Section Score
(# OF CORRECT ANSWERS) LOWER UPPER LOWER UPPER
0 200 200 200 200
1 200 200 200 200
2 200 200 200 200
3 200 200 200 200
4 200 200 200 200
5 200 200 200 200
6 200 200 200 200
7 200 210 200 220
8 210 230 200 230
9 220 240 220 250
10 230 250 250 280
11 250 270 270 300
12 260 280 290 320
13 280 300 300 330
14 290 310 310 340
15 300 320 320 350
16 330 350 330 360
17 350 370 340 370
18 360 380 350 380
19 370 390 360 390
20 380 400 370 400
21 390 410 380 410
22 400 420 400 430
23 410 430 410 440
24 420
440 420 450
25 430 450 430 460
26 430 450 440 470
27 440 460 450 480
28 450 470 460 490
29 460 480 470 500
30 460 480 480 510
31 470 490 490 520
32 480 500 510 540
33 490 510 520 550
RAW SCORE
R
eading and Writing
Section Score
Math
Section Score
(# OF CORRECT ANSWERS) LOWER UPPER LOWER UPPER
34 490 510 530 560
35 500 520 540 570
36 510 530 550 580
37 520 540 560 590
38 520 540 570 600
39 530 550 590 620
40 540 560 600 630
41 550 570 610 640
42 550 570 620 650
43 560 580 630 660
44 570 590 640 670
45 570 590 660 690
46 580 600 670 700
47 590 610 690 720
48 590 610 700 730
49 600 620 720 750
50 610 630 740 770
51 610 630 760 790
52 620 640 770 790
53 630 650 790 800
54 640 660 800 800
55 640 660
56 650 670
57 660 680
58 670 690
59 680 700
60 690
710
61 700 720
62 710 730
63 720 740
64 740 760
65 770 780
66 790 800
© 2022 College Board.   2223-BB-682